X



トップページ数学
1002コメント417KB

不等式への招待 第10章

レス数が1000を超えています。これ以上書き込みはできません。
0001不等式ヲタ ( ゚∀゚)
垢版 |
2018/12/18(火) 21:47:07.65ID:e1oKVpnI
ある人は蝶を集め、ある人は切手を収集し、ある人は不等式を集める…
          ___          ----- 参考文献〔3〕 P.65 -----
    |┃三 ./  ≧ \   
    |┃   |::::  \ ./ | 
    |┃ ≡|::::: (● (● |  不等式と聞ゐちゃぁ
____.|ミ\_ヽ::::... .ワ......ノ     黙っちゃゐられねゑ…
    |┃=__    \           ハァハァ
    |┃ ≡ )  人 \ ガラッ

【まとめWiki】 http://wiki.livedoor.jp/loveinequality/

【過去スレ】
・不等式スレッド (第1章) http://science3.2ch.net/test/read.cgi/math/1072510082/
・不等式への招待 第2章 http://science6.2ch.net/test/read.cgi/math/1105911616/
・不等式への招待 第3章 http://science6.2ch.net/test/read.cgi/math/1179000000/
・不等式への招待 第4章 http://science6.2ch.net/test/read.cgi/math/1245060000/
・不等式への招待 第5章 http://uni.2ch.net/test/read.cgi/math/1287932216/
・不等式への招待 第6章 http://uni.2ch.net/test/read.cgi/math/1332950303/
・不等式への招待 第7章 http://rio2016.2ch.net/test/read.cgi/math/1362834879/
・不等式への招待 第8章 http://rio2016.2ch.net/test/read.cgi/math/1498378859/
・不等式への招待 第9章 https://rio2016.5ch.net/test/read.cgi/math/1505269203/
・過去スレのミラー置き場 http://cid-d357afbb34f5b26f.skydrive.live.com/browse.aspx/.Public/

【姉妹サイト】
キャスフィ 高校数学板 不等式スレ  http://www.casphy.com/bbs/test/read.cgi/highmath/1169210077/
キャスフィ 高校数学板 不等式スレ2 http://www.casphy.com/bbs/test/read.cgi/highmath/1359202700/

【wikiなど】
Inequality (mathematics)
https://en.wikipedia.org/wiki/Inequality_(mathematics)
List of inequalities
https://en.wikipedia.org/wiki/List_of_inequalities
List of triangle inequalities
https://en.wikipedia.org/wiki/List_of_triangle_inequalities
Wolfram MathWorld
http://mathworld.wolfram.com/topics/Inequalities.html
0002不等式ヲタ ( ゚∀゚)
垢版 |
2018/12/18(火) 21:47:30.11ID:e1oKVpnI
【不等式の和書】

[1] 不等式(数学クラシックス11),Hardy, Littlewood, Polya,丸善出版,2003年
   http://amazon.jp/o/ASIN/4431710566
[2] 不等式(数学選書),大関信雄・青木雅計,槇書店,1967年(絶版)
[3] 不等式への招待(数学ゼミナール6),大関信雄・大関清太,近代科学社,1987年
   http://amazon.jp/dp/4844372661
[4] 不等式入門(数学のかんどころシリーズ9),大関清太,共立出版,2012年
   http://www.kyoritsu-pub.co.jp/bookdetail/9784320019898
[5] 不等式入門(数学ライブラリー教養篇4),渡部隆一,森北出版,2005年
   http://amazon.jp/o/ASIN/4627010494
[6] 不等式の工学への応用、海津聰、森北出版,2004年
   http://amazon.jp/o/ASIN/4627075812
[7] 不等式(モノグラフ4),染取弘,科学新興新社,1990年
   http://amazon.jp/o/ASIN/4894281740
[8] 不等式 〜 21世紀の代数的不等式論 〜,安藤哲哉,数学書房,2012年
  http://amazon.jp/dp/4903342700、(正誤表+補遺)http://www.math.s.chiba-u.ac.jp/~ando/
[9] 美しい不等式の世界: 数学オリンピックの問題を題材として,佐藤淳郎(訳),朝倉書店,2013年
  http://amazon.jp/dp/4254111371
[10] 思考力を鍛える不等式(大学への数学・別冊)、栗田哲也、東京出版、2014年
  http://www.amazon.co.jp/dp/4887422091

【不等式のpdf】

[1] Vasile Cîrtoaje、Mathematical Inequalities. http://ac.upg-ploiesti.ro/vcirtoaje/vcirtoaje.php
[2] 柳田五夫、初等的な不等式TUVなど. http://izumi-math.jp/I_Yanagita/I_Yanagita.html

【おもな埋蔵地】

[1] JIPAM https://www.emis.de/journals/JIPAM/
[2] MIA http://mia.ele-math.com/volume/20
[3] AoPS https://artofproblemsolving.com/community
[4] Maths problems http://webee.technion.ac.il/people/aditya/www.kalva.demon.co.uk/index.html
[5] IMO http://www.imo-official.org/problems.aspx
[6] American Mathematical Monthly Problems http://www.mat.uniroma2.it/~tauraso/AMM/amm.html

※ その他の参考文献などは、まとめWikiを参照 http://wiki.livedoor.jp/loveinequality/
0003不等式ヲタ ( ゚∀゚)
垢版 |
2018/12/18(火) 21:47:58.79ID:e1oKVpnI
諸君 私は不等式が好きだ
諸君 私は不等式が大好きだ

改造が好きだ 改良が好きだ 拡張が好きだ

AM-GMで Cauchyで Holderで Jensenで Schurで
Chebyshevで rearrangementで Bernoulliで
Muirheadで Karamataで Maclaurinで ぬるぽビッチで

この地上に現れるありとあらゆる不等式が大好きだ

大小順をそろえた歩兵の横隊を 並べ替え不等式で蹂躙するのが好きだ
恐慌状態の新兵が分母にAM-GMを誤用して 不等号の向きを何度も何度も間違えている様など感動すら覚える

糸口の見つからない不等式に滅茶苦茶に悩まされるのが好きだ
必死に悩んだ不等式が成立しない例を挙げられていく様はとても悲しいものだ

君達は一体何を望んでいる? 更なる不等式を望むか?

     『不等式! 不等式! 不等式!』

よろしい ならば証明だ!

              rv―v―、 r-v-v
        r、      ノ     も( ノ ま (      ,ィx
      (\\(^}   ) !! 厳. っ ( ) だ (   /)///7
      {^ヽ^ヽ {   )  し. と ( ) だ (  / 'ヽ /
        \ `Y ノ}_  ハ  く  ノ 乂  ノ  {. 〈 /
           〉,r彡ハ  _>  < /  ま ( 人_ノ〉
           V   ∨ !! 改 も () !! だ( /  7 /
          'v   V  良 っ ().  だ(/    /
             'v   V .を と 人_,ノ〈    /
           V   V rfテ弐ミk /   }'
                'v   ',仔r=r弌リ'   /
                'v   '({ ヾ二フ,j'   /
               }    j个ー‐个ト,  /
             }> / />ュ<ト、\ノ{
                _j/  / | / :| | \\
              _,>、__, イ>\/ _」/\ ̄{_
          /  /:::::| \/__,>|:::::∧  {
            /|  ./:::::/  厂     |::::::::∧ |\
         / :| /:::::/   |o      〔::::::::::::∧.|  \
       / / /:::::/   :|o    丿:|:::::::::::::∧    \
0004132人目の素数さん
垢版 |
2018/12/18(火) 21:55:19.66ID:e1oKVpnI
前スレ967の不等式、(3)が2019年度中国数学オリンピック第一問

(1) a,b,c≧-1, a+b+c=3 のとき、-32≦(a+b)(b+c)(c+a)≦8.
(2) a,b,c,d≧-1, a+b+c+d=4 のとき、-48≦(a+b)(b+c)(c+d)(d+a)≦144.
(3) a,b,c,d,e≧-1, a+b+c+d+e=5 のとき、-512≦(a+b)(b+c)(c+d)(d+e)(e+a)≦288.

---------------------------------------
(1)の証明
a+b, b+c, c+a のうち負は高々1個。

a+b, b+c, c+a ≧0のとき、AM-GMより、
0 ≦ (a+b)(b+c)(c+a) ≦ [ {(a+b)+(b+c)+(c+a)}/3]^3 = 8.

1つだけ負のとき、対称性から a+b < 0 ≦ b+c, c+a としてよい。
このとき、条件より 3<c≦5 で、AM-GMより、
0 ≦ -2(a+b)(b+c)(c+a) ≦ [ {-2(a+b)+(b+c)+(c+a)}/3]^3 = (c-1)^3 ≦ 64.

---------------------------------------
〔予想〕
 a_1, a_2, …, a_n ≧ -1, a_1+a_2+…+a_n = n, のとき

・n:奇数 (n≧5) ならば
 -(2^n)(n-1)^2 ≦ Π(a_j + a_{j+1}) ≦ 2^{n-2} (n-2)^2 (n-1),

・n:偶数 ならば
 -2^{n-2} (n-2)^2 (n-1) ≦ Π(a_j + a_{j+1}) ≦(2^n)(n-1)^2.
0005132人目の素数さん
垢版 |
2018/12/19(水) 01:42:03.32ID:01wf4Nsj
削除依頼を出しました
0006132人目の素数さん
垢版 |
2018/12/19(水) 05:34:28.01ID:5zoTD2o3
>>1
スレ立て乙
ついに2桁に到達したでござるな。

・次スレ用のメモ。

【過去スレ】
 〜.2ch.net/ → 〜.5ch.net/
・過去スレのミラー置き場
   http://onedrive.live.com/?id=D357AFBB34F5B26F%21110&;cid=D357AFBB34F5B26F

【姉妹サイト】
キャスフィ 高校数学板 不等式スレ  http://www.casphy.com/bbs/highmath/471952/
キャスフィ 高校数学板 不等式スレ2 http://www.casphy.com/bbs/highmath/472060/
0007132人目の素数さん
垢版 |
2018/12/19(水) 05:36:05.68ID:5zoTD2o3
>>2
次スレ用のメモ

【不等式の和書】

[1] 不等式 (数学クラシックス11), G.H.Hardy、J.E.Littlewood、G.Polya(著)、細川尋史(訳), 丸善出版, 2012年, 417p.
   http://www.amazon.co.jp/o/ASIN/4621063510
[2] 不等式 (数学選書), 大関信雄・青木雅計, 槇書店, 1967年(絶版), 237p.
   ASIN B000JA494Y,
[3] 不等式への招待 (数学ゼミナール6), 大関信雄・大関清太,近代科学社,1992年, 162p.
   ASIN 4844372661, ISBN 978-4-844-37266-0,
   http://www.kindaikagaku.co.jp/news/20120912/index.html
[4] 不等式入門 (数学のかんどころシリーズ9), 大関清太, 共立出版, 2012年, 186p.
   http://www.kyoritsu-pub.co.jp/bookdetail/9784320019898
[5] 不等式入門 (数学ライブラリー教養篇4), 渡部隆一,森北出版,2005年, 154p.
   ASIN 4627010494, ISBN 978-4-627-01049-9,
   http://www.morikita.co.jp/books/book/84
[6] 不等式の工学への応用, 海津 聰(訳), 森北出版,2004年, 160p.
   ASIN 4627075812, ISBN 978-4-627-07581-8,
   http://www.morikita.co.jp/books/book/437
[7] 不等式 (モノグラフ4), 梁取 弘(著)、矢野健太郎(監修), 科学新興新社, 1998年, 118p.
   http://amazon.jp/o/ASIN/4894281740
[8] 不等式 〜 21世紀の代数的不等式論 〜, 安藤哲哉, 数学書房, 2012年, 280p.
   ASIN 4903342700, ISBN 978-4-903-34270-2,
   http://www.sugakushobo.co.jp/903342_70_mae.html
  (正誤表+補遺) http://www.math.s.chiba-u.ac.jp/~ando/
[9] 美しい不等式の世界: 数学オリンピックの問題を題材として, 佐藤淳郎(訳), 朝倉書店, 2013年, 260p.
   http://www.asakura.co.jp/G_12.php?isbn=ISBN978-4-254-11137-8
[10] 思考力を鍛える不等式 (大学への数学・別冊), 栗田哲也, 東京出版, 2014年, 135p,
   ASIN 4887422091, ISBN 978-4-887-42209-4,
   http://www.tokyo-s.jp/products/d_zoukan/futoushiki/index.html

※ その他の参考文献などは、まとめWikiを参照 https://seesaawiki.jp/w/loveinequality/
0009132人目の素数さん
垢版 |
2018/12/19(水) 09:51:30.85ID:K5b8go44
>>4
> (2) a,b,c,d≧-1, a+b+c+d=4 のとき、-48≦(a+b)(b+c)(c+d)(d+a)≦144.

a+b, b+c, c+d, d+a のうち、3つ以上が負にならない。
また2つが負のとき、隣り合う2つか、一つおきの2つが負だが、後者はありえない。

結局、次の3つを考えればよい。
(i) a+b, b+c, c+d, d+a ≧0.
(ii) a+b, b+c, c+d ≧0 >d+a.
(iii) a+b, b+c ≧0 > c+d, d+a.

(i)のとき、AM-GMより 0≦(a+b)(b+c)(c+d)(d+a)≦16.

(ii)のとき、4<b+c≦6に注意して
0 ≦ -27(a+b)(b+c)(c+d)(d+a) ≦ [ {3(a+b)+(b+c)+3(c+d)-3(d+a)}/4]^4 = (b+c)^4 ≦1296,.
∴ 0≧(a+b)(b+c)(c+d)(d+a)≧-48.

(iii)のとき、-1≦d<b≦7に注意して
0 ≦ 9(a+b)(b+c)(c+d)(d+a) ≦ [ {(a+b)+(b+c)-3(c+d)-3(d+a)}/4]^4 = (b-d-2)^4 ≦1296.
∴ 0≦(a+b)(b+c)(c+d)(d+a)≦144.

( ゚∀゚) こんなものかな。次は(5)か…
0012132人目の素数さん
垢版 |
2018/12/19(水) 20:38:59.29ID:K5b8go44
>>4
> (3) a,b,c,d,e≧-1, a+b+c+d+e=5 のとき、-512≦(a+b)(b+c)(c+d)(d+e)(e+a)≦288.

a+b, b+c, c+d, d+e, e+a のうち、4つ以上が負にならない。
また3つが負の場合に、負でない2つが隣り合わない場合も除外してよい。

結局、次の5つを考えればよい。
(i) a+b, b+c, c+d, d+e, e+a ≧0.
(ii) a+b, b+c, c+d, d+e ≧0 > e+a.
(iii) a+b, b+c, c+d ≧0 > d+e, e+a.
(iv) a+b, b+c, d+e ≧0 > c+d, e+a.
(v) a+b, b+c ≧0 > c+d, d+e, e+a.

(i)のとき、AM-GMより 0≦(a+b)(b+c)(c+d)(d+e)(e+a)≦32.

(ii)のとき、-2≦e+a<0 に注意して
0 ≦ -(a+b)(b+c)(c+d)(d+e)(e+a)
 ≦ [ {(a+b)+(b+c)+(c+d)+(d+e)-(e+a)}/5]^5
 = {2 - (2/5)*(e+a)}^5
 ≦ (14/5)^5
 < 512.
∴ 0≧(a+b)(b+c)(c+d)(d+e)(e+a)>-512.

(iii)のとき、-1≦e≦5, 5<b+c-e≦9 に注意して
0 ≦ 27648(a+b)(b+c)(c+d)(d+e)(e+a)
 ≦ [ {8(a+b)+3(b+c)+8(c+d)-12(d+e)-12(e+a)}/5]^5
 = {3(b+c-e)-e-4}^5
 ≦ 24^5.
∴ 0≦(a+b)(b+c)(c+d)(d+e)(e+a)≦288.

(iv)のとき、5<b≦9 に注意して
0 ≦ (a+b)(b+c)(c+d)(d+e)(e+a)
 ≦ [ {(a+b)+(b+c)+(d+e)}/3]^3 * [ {-(c+d)-(e+a)}/2]^2
 = {(b+5)/3}^3 * {(b-5)/2}^2
 ≦ (14/3)^3 * 2^2
 < 288.
∴ 0≦(a+b)(b+c)(c+d)(d+e)(e+a)<288.

(v)のとき、-1≦d,e<b≦9 に注意して
0 ≦ -64(a+b)(b+c)(c+d)(d+e)(e+a)
 ≦ [ {(a+b)+(b+c)-4(c+d)-4(d+e)-4(e+a)}/5]^5
 = (b-d-e-3)^5
 ≦ 8^5.
∴ 0≧(a+b)(b+c)(c+d)(d+e)(e+a)≧-512.

( ゚∀゚) ウヒョッ! 残りは一般のnの場合だが、こんなやり方じゃ場合分けで死ぬ…
0015132人目の素数さん
垢版 |
2018/12/22(土) 22:59:48.76ID:fXYd5inn
正整数t,k,mがあって、t^2>kmを満たす。
このとき次の不等式が成立
Σ[i=0,m-t-1]C[2m,i]<4^m/2k
0019132人目の素数さん
垢版 |
2018/12/26(水) 09:35:55.89ID:2DcmDNkQ
〔補題〕
自然数mについて
 4^m /√(π(m+1/3)) < C[2m,m] < 4^m /√(π(m+1/4)),

(略解)
 a_m = √{π(m+1/4)} C[2m,m]/(4^m),
 b_m = √{π(m+1/3)} C[2m,m]/(4^m),
とおくと
 a_m < b_m,

(4m+5)(2m+1)^2 - (4m+1)(2m+2)^2 = 1 より
a_{m+1}/a_m = √{(4m+5)/(4m+1)}・{(2m+1)/(2m+2)} > 1,
a_m は単調増加。

(3m+1)(2m+2)^2 - (3m+4)(2m+1)^2 = m より
b_{m+1}/b_m = √{(3m+4)/(3m+1)}・{(2m+1)/(2m+2)} < 1,
b_m は単調減少。

よって
a_1 < a_2 < ・・・・ < a_m < ・・・・ < b_m < ・・・・ < b_2 < b_1
ゆえ収束する。
極限値 1
0020132人目の素数さん
垢版 |
2018/12/26(水) 23:29:50.65ID:2DcmDNkQ
>>19 (続き)

3<π<4  √(π(m+1/4)) < 1 < √(π(m+1/3)) より a_0 < 1 < b_0
3<π<3.2 √(π(m+1/4)) < 2 < √(π(m+1/3)) より a_1 < 1 < b_1
m>1 のときも a_m < 1 < b_m
0021132人目の素数さん
垢版 |
2018/12/28(金) 01:30:14.08ID:NvXV1n10
>>19
極限値はウォリスの公式
  C[2m,m] = (2m)! / (m!)^2 〜 4^m / √(πm),
またはスターリングの公式
 log(n!) = (n+1/2)log(n) - n + (1/2)log(2π) + 1/(12n) + O(1/n^3)
 log{C[2m,m]} = log{(2m)!} - 2log(m!) = m log(4) - (1/2)log{π(m+1/4)} + O(1/m^3)
から出る。
0022132人目の素数さん
垢版 |
2019/01/01(火) 00:06:21.22ID:HV303Epp
      ∧_∧
     ( ´Д` )  新年あけまして
     /     ヽ
     し、__X__,ノJ

      /´⌒⌒ヽ
    l⌒    ⌒l  おめでとうございます。
   ⊂ (   ) ⊃
      V ̄V

正の数 a,b,c に対して
(a^2019 -a^31 +3) (b^2019 -b^31 +3) (c^2019 -c^31 +3) ≧ 9 (abc)^(4/3),

[第9章.395, 397]
0024132人目の素数さん
垢版 |
2019/01/01(火) 11:28:11.24ID:HV303Epp
Inequalitybot [104]
(aa+bb+cc)^2 - (ab+bc+ca)^2 ≧ (√6)|(a-b)(b-c)(c-a)(a+b+c)|,

[前スレ.574, 576]
//www.casphy.com/bbs/highmath/472060 不等式2-196
じゅー君 (作)
0025132人目の素数さん
垢版 |
2019/01/01(火) 12:27:27.17ID:HV303Epp
>>22

x^2019 - x^31 + 3 ≧ 2.08319787624644064040 x^(4/3),
等号成立は x = 0.99794707802373850618 のとき。
 (6053x^2019 - 89x^31 - 12 = 0 の正根)

最良値 9.040481720894526247626

       人
 /⌒\ (__)
 \●/(__)/⌒\
   ∩ (・∀・ )\●/  あけおめでござる。
   Y  ̄ ||y||  ̄`''φ
    Lノ /ニ|| ! ソ >
    乂/ノ ハ ヽー´
    `ー-、__|
0026132人目の素数さん
垢版 |
2019/01/07(月) 14:07:43.34ID:nTHVJaxN
>>22
〔問題390〕
正の数 a,b,c に対して
(a^2019 - a^31 + 3) (b^2019 - b^31 + 3) (c^2019 - c^31 + 3) > 3 (a^4 + b^4 + c^4),

[前スレ.390]
0027132人目の素数さん
垢版 |
2019/01/08(火) 01:02:25.62ID:WdFpB4mR
>>26
x0 = 0.99794707802373850618 とおく。  >>25

(x^2019 - x^31 + 3)^3 ≧ k {(x/x0)^12 + 1 + 1} ≧ 3k (x/x0)^4,
ここに k = 2.98882413327445720383

(左辺) = (a^2019 - a^31 + 3)(b^2019 - b^31 + 3)(c^2019 - c^31 + 3)
≧ k [{(a/x0)^12 + 1 + 1} {1 + (b/x0)^12 + 1} {1 + 1 + (c/x0)^12}]^{1/3}
≧ k {(a/x0)^4 + (b/x0)^4 + (c/x0)^4}     (←コーシー)
= (k/x0^4) (a^4 + b^4 + c^4)
= 3.01349390694842082546 (a^4 + b^4 + c^4)

等号成立は a=b=c=x0.
0029132人目の素数さん
垢版 |
2019/01/09(水) 15:53:33.07ID:Cl/0e+Ah
x はn次元ベクトル
||x||_p はp-norm
p>q>0に対して、||x||_p ≦ ||x||_q ≦ n^(1/q - 1/p)*||x||_p.

今年もよろしくお願いします ( ゚∀゚) ウヒョッ!
0031132人目の素数さん
垢版 |
2019/01/10(木) 06:11:46.62ID:4mlVGqNc
>>27
x ≒ x0 の周りにテイラー展開すれば
(x^2019 - x^31 + 3)^3 = 3k{1 + 4(x/x0 - 1) + 45791.82863314406(x/x0 -1)^2 + ・・・・},
(x/x0)^12 + 1 + 1 = 3{1 + 4(x/x0 - 1) + 22(x/x0 - 1)^2 + ・・・・},
(x/x0)^4 = {1 + 4(x/x0 - 1) + 6(x/x0 - 1)^2 + ・・・・},
0032132人目の素数さん
垢版 |
2019/01/20(日) 05:29:41.07ID:3c2tpFSV
>>13
左:
 0 < t < 1/2 ⇒ 1-t > t,
 1/2 < t < 1 ⇒ t > 1-t,
 (中辺) > (a+b-1)∫[0,1/2] t^{a+b-2} dt + (a+b-1)∫[1/2,1] (1-t)^{a+b-2} dt
 = [ t^{a+b-1} ](0,1/2) + [ (1-t)^{a+b-1} ](1/2,1)
 = (1/2)^{a+b-1} + (1/2)^{a+b-1}
 = (1/2)^{a+b-2},

右:
ヤングの不等式より
 t^{a-1}・(1-t)^{b-1} ≦ [(a-1)t^{a+b-2} + (b-1)(1-t)^{a+b-2}]/(a+b-2),

 (中辺) ≦ [(a-1)t^{a+b-1} - (b-1)(1-t)^{a+b-1}](0,1) /(a+b-2)
 = [(a-1) + (b-1)] / (a+b-2)
 = 1,
0033132人目の素数さん
垢版 |
2019/01/22(火) 05:49:20.92ID:hfoTnJ0x
〔前スレ.950〕
a,b,c > 0 に対して、
(a+b+c)(1/a + 1/b + 1/c) ≧ 9 + 8[(a-b)^2 +(b-c)^2 +(c-a)^2]/(a+b+c)^2,

( //www.casphy.com/bbs/highmath/472060/ 不等式2 - 325, 336 )
( //suseum.jp/gq/question/3013 )
0034132人目の素数さん
垢版 |
2019/01/22(火) 06:04:00.30ID:hfoTnJ0x
>>33
bはaとcの中間にあるとしてよい。
 (a-b)(b-c) ≧ 0,
 (c-a)^2 = (a-b)^2 + 2(a-b)(b-c) + (b-c)^2,

s = a+b+c,u = abc とおく。
(左辺 - 右辺)・ssu
= [c(a-b)^2 + b(c-a)^2 + a(b-c)^2]ss - 8u[(a-b)^2 + (c-a)^2 + (b-c)^2]
= (css -8u)(a-b)^2 + (bss -8u)(c-a)^2 + (ass -8u)(b-c)^2
= [(b+c)ss -16u](a-b)^2 + (bss -8u)・2(a-b)(b-c) + [(a+b)ss -16u](b-c)^2
≧ 4a[(a-b)(b-c)]^2 + 16b[(a-b)(b-c)]^2 + 4c[(a-b)(b-c)]^2
= 4(a+4b+c)[(a-b)(b-c)]^2
≧ 0,

 (b+c)ss - 16u -4a(b-c)^2 = (b+c)[ss - 4a(b+c)] = (b+c)(-a+b+c)^2 ≧ 0,
 bss - 8u -8b(a-b)(b-c) = b[ss -8b(a-b+c)] = b(a-3b+c)^2 ≧ 0,
 (a+b)ss - 16u -4c(a-b)^2 = (a+b)[ss - 4c(a+b)] = (a+b)(a+b-c)^2 ≧ 0,
0035132人目の素数さん
垢版 |
2019/01/25(金) 08:47:28.94ID:dfwh8WQW
a,b,c >0 のとき

(1) (abb)^3 + (bcc)^3 + (caa)^3 + 3(abc)^3 ≧ abc{(ab)^3 + (bc)^3 + (ca)^3} + (abc)^2・(a^3 +b^3 +c^3)
   バルカンMO-2015、P1

(2)  a√(a+3b+c) + b√(a+b+3c) + c√(3a+b+c) ≦ √(a+b+c)・√{a(a+3b+c)+b(a+b+3c)+c(3a+b+c)},
   セルビアMO-2017、P1改

(3)  (a-b-c)^2 /b + (b-c-a)^2 /c + (c-a-b)^2 /a ≧ (aa+bb)/(a+b) + (bb+cc)/(b+c) + (cc+aa)/(c+a),
   クロアチアMO-2018、A1改

ガイシュツかも知れませんが・・・・
0036132人目の素数さん
垢版 |
2019/02/04(月) 16:57:27.17ID:KxBFZY2a
a,b,c>0に対して、1/(a+b) + 1/(b+c) + 1/(c+a) ≧ (3√3)/{2√(aa+bb+cc)}.

( ゚∀゚)ノ ごきげんよう
0037132人目の素数さん
垢版 |
2019/02/05(火) 12:41:14.63ID:xI3EwwZt
とりあえず改造・・・・

a,b,c>0 に対して
 1/a + 1/b + 1/c ≧ 2/(a+b) + 2/(b+c) + 2/(c+a) ≧ 9/(a+b+c) ≧ 9/√{3(aa+bb+cc)}.

( ゚∀゚)ノ ごぶさたでござる。
0039132人目の素数さん
垢版 |
2019/02/11(月) 18:01:27.73ID:dvi1QDx1
等式だけど
(aa+bb-1)^2 + (cc+dd-1)^2 + 2(ac+bd)^2
= (aa+cc-1)^2 + (bb+dd-1)^2 + 2(ab+cd)^2.

これって何か背景あるのかな?
0040132人目の素数さん
垢版 |
2019/02/12(火) 17:06:09.82ID:zTm0tcyX
>>39
行列式を展開して作れるのかなと思ったが、どんな行列式から出てくるか思いつかん。
0041132人目の素数さん
垢版 |
2019/02/16(土) 01:12:40.14ID:fRyCy6GA
>>39
 (aa+bb-1)^2 + (cc+dd-1)^2 - 2(ab±cd)^2
= (aa+cc-1)^2 + (bb+dd-1)^2 - 2(ac±bd)^2.
= (aa+dd-1)^2 + (bb+cc-1)^2 - 2(ad±bc)^2,  (複号同順)
同じことだけど・・・・
0042132人目の素数さん
垢版 |
2019/02/16(土) 01:20:11.40ID:fRyCy6GA
数セミ3月号 NOTE より

n個の正数 x_1, x_2, …, x_n の相加平均を A_n、相乗平均を G_n とする。
それに正数 y を追加した (n+1)個組の相加平均を A_{n+1}、相乗平均を G_{n+1} とする。このとき
 n(A_n - G_n) ≦ (n+1)(A_{n+1} - G_{n+1}),

[前スレ.866, 872]
ニコニコ大百科
http://dic.nicovideo.jp/a/jacobsthalの不等式
0047132人目の素数さん
垢版 |
2019/02/20(水) 07:16:40.17ID:4ATqaV2Y
記事のコメントには、ヤコブスタールの不等式そのものであるとか指摘はなかったけど、ZZZは知らなかったのか?
0048132人目の素数さん
垢版 |
2019/02/20(水) 23:05:39.15ID:u2k/hpyq
学術誌レベルの新規性を要求されちゃカナワンよなぁ。
雑誌を全部サーチしてから出すなんて、どだい無理だし。
0049132人目の素数さん
垢版 |
2019/02/21(木) 00:14:53.49ID:vd6keGE+
ヤコブスタールの不等式は、一般人には有名じゃないの?
コーラを飲めばゲップが出るくらい当たり前のことだと思っていたけど…
0050132人目の素数さん
垢版 |
2019/02/22(金) 02:26:44.49ID:C/jpUJqi
読者(一般人)に有名でなければ 再発見でもOK
ということで願いたい。
0051132人目の素数さん
垢版 |
2019/03/05(火) 01:25:25.02ID:gNDCshUz
〔問題3043〕
A, B, C は正の実数で次式を満たす。
 M = (A+B+C)/3 = 1/(nn+n+1),
 n = 1, 2, ・・・・
このとき次式を示せ。
 {(1-A)/A^p}^p + {(1-B)/B^p}^p + {(1-C)/C^p}^p ≧ 3{(1-M)/M^p}^p,
ただし p = 1/(n+1).
 by K. Chikaya (2019/Feb)

http://suseum.jp/gq/question/3043
0054132人目の素数さん
垢版 |
2019/03/09(土) 06:26:00.80ID:3JHzAJ6Y
〔bot-59〕
x,y,z>0 のとき次を示せ。
 4 + xx + xyy + xyzz ≧ 4xyz,
 カナダMO-2012 A.1


左から AM-GM するだけ....
 4 + xx + xyy + xyzz + xyzww ≧ 4xyzw,
0056132人目の素数さん
垢版 |
2019/03/09(土) 23:16:13.39ID:Vpcwz/ga
a,b,c,d,e>0 に対して、
(abcd + bcde + cdea + deab + eabc)^4 ≧ 125(a+b+c+d+e)(abcde)^3
0057132人目の素数さん
垢版 |
2019/03/11(月) 00:59:41.06ID:gRQ8L5Xj
>>56
 abcd + bcde + cdea + deab + eabc = 5(G^5)/H,
 a+b+c+d+e = 5A,
 abcde = G^5,
だから
 A^(n-1)・H ≧ G^n ≧ A・H^(n-1)  … Sierpinskiの不等式
の右側でござるか。

・文献3 (大関) 2-2 例題1 p.79-80
0058132人目の素数さん
垢版 |
2019/03/11(月) 10:40:22.46ID:gRQ8L5Xj
>>56
1/a=A, 1/b=B, 1/c=C, 1/d=D, 1/e=E
とおく。 与式は
 (A+B+C+D+E)^4 ≧ 125(ABCD + BCDE + CDEA + DEAB + EABC),
となる。  左辺を展開すると、いろいろなパターンの4次項が現れる。
5つから重複を許して4つを取り出す場合は
 (4,1) (3,1,1) (2,2) (2,1,1) (1,1,1,1)
の5パターンがある。
 (4,1) A^4 + B^4 + C^4 + D^4 + E^4,
 (3,1) 4(A^3)(B+C+D+E) + …
 (2,2) 6(AABB+AACC+AADD+AAEE+BBCC+BBDD+BBEE+CCDD+CCEE+DDEE),
 (2,1,1) 12AA(BC+BD+BE+CD+CE+DE) + …
 (1,1,1,1) 24(ABCD + BCDE + CDEA + DEAB + EABC) = 24v,

AM-GM より (A^4 + B^4 + C^4 + D^4)/4 ≧ ABCD,
 (4,1) ≧ v,
同様にして(チョト怪しい…)
 (3,1) ≧ 16v,
 (2,2) ≧ 12v,
 (2,1,1) ≧ 72v,
 (1,1,1,1) = 24v,
となるので、
 (左辺) = (A+B+C+D+E)^4 ≧ (1+16+12+72+24)v = 125v = (右辺),
0059132人目の素数さん
垢版 |
2019/03/11(月) 10:43:09.35ID:gRQ8L5Xj
>>58
 訂正スマソ

5つから重複を許して4つを取り出す場合は
 (4) (3,1) (2,2) (2,1,1) (1,1,1,1)
の5パターンがある。
0060132人目の素数さん
垢版 |
2019/03/12(火) 00:28:15.75ID:iP2fXkuo
>>58
Muirhead から
 (4) ≧ (3,1) ≧ (2.2) ≧ (2,1,1) ≧ (1,1,1,1) = 24v,
ですね。
一般のnについても、同様に成立。
0063132人目の素数さん
垢版 |
2019/03/14(木) 04:36:47.63ID:QG1K7uiM
〔B.5009〕
 Given that xx+yy+zz=3, where x,y,z are positive numbers. Prove that
  2^(1/x) + 2^(1/y) + 2^(1/z) ≧ 6,           (2019/Feb)

(略証)
コーシーで
 (xx+yy+zz)(1/x+1/y+1/z)^2 ≧ (1+1+1)^3 = 27,
あるいは AM-HM で
 1/x + 1/y + 1/z ≧ 9/(x+y+z) ≧ √{27/(xx+yy+zz)} = 3,
AM-GM で
 (左辺) ≧ 3・2^{(1/x +1/y +1/z)/3} ≧ 3・2 = 6,
等号成立は x=y=z=1.
0064132人目の素数さん
垢版 |
2019/03/14(木) 05:04:01.19ID:QG1K7uiM
〔C.1511〕
B and C are interior points of a line segment AD such that AB=CD.
 Prove that PA+PD ≧ PB+PC for any point P on the plane.   (2018/Dec)

(略証)
・Pが直線AD上になく、B≠C の場合。
 ADの中点 = BCの中点 をFとし、PFの延長線上に PF=FQ となる点Qをとる。
 問題図は点Fに関して対称である。
 儕AF ≡ 儔DF、 儕BF ≡ 儔CF
 PA = QD、 PB = QC
 ここで、儕DQ ⊃ 儕CQ だから
 QD + PD ≧ QC + PC,
 ∴ PA + PD ≧ PB + PC,
・B=C の場合は△不等式となる。
・Pが直線AD上の場合は明らか。A以遠またはD以遠のとき等号成立。
0065132人目の素数さん
垢版 |
2019/03/14(木) 05:10:51.33ID:QG1K7uiM
〔B.4980〕
Let n>3 be a positive integer, and let a_1, a_2, ・・・・, a_n be positive real numbers. Prove that
 1 < a_1/(a_n+a_1+a_2) + a_2/(a_1+a_2+a_3) + ・・・・ + a_n/(a_{n-1}+a_n+a_1) < [n/2]
where the left-hand side of the inequality cannot be replaced by a larger number, and the right-hand side cannot be replaced by a smaller number.
( [x] denotes the greatest integer not greater than the number x.)   (2018/Oct)


(左側)
 (分母) < a_1+a_2+・・・・+a_n により成立。
 また ε>0,  a_i = ε^(i-1) とすると、(中辺) < 1 +(n-1)ε,
  ε→0 とすれば1に近づく。

(右側)
nが偶数のとき、隣合うペアについて
 a_i/(a_{i-1}+a_i+a_{i+1}) + a_{i+1}/(a_i+a_{i+1}+a_{i+2}) < a_i/(a_i+a_{i+1}) + a_{i+1}/(a_i+a_{i+1}) = 1,
だから成立。
nが奇数のときは、分母(a_{j-1}+a_j+a_{j+1})が最小となるような j に対して
 a_{j-1}/(a_{j-2}+a_{j-1}+a_j) + a_j/(a_{j-1}+a_j+a_{j+1}) + a_{j+1}/(a_j+a_{j+1}+a_{j+2}) < (a_{j-1}+a_j+a_{j+1})/(a_{j-1}+a_j+a_{j+1}) = 1,
残った偶数項を隣合うペアに分ける。 後略
0066132人目の素数さん
垢版 |
2019/03/14(木) 05:30:26.45ID:QG1K7uiM
〔C.1493〕
A triangle of unit area has sides a,b,c, such that a≧b≧c.
Show that b ≧ √2.               (2018/Sep)

(略解)
 2 = bc・sin(A) ≦ bc ≦ bb,
 b ≧ √(2),
等号成立は直角2等辺
0067132人目の素数さん
垢版 |
2019/03/14(木) 05:33:39.14ID:QG1K7uiM
〔B.4968〕
Solve the following simultaneous equations on the set of positive real numbers:
 1/(1+a+ab+abc) + 1/(1+b+bc+bcd) + 1/(1+c+cd+cda) + 1/(1+d+da+dab) = 1,
 a+b+c+d = 4.                   (2018/Sep)

(略解)
 abcd ≦ {(a+b+c+d)/4}^4 = 1,    (← GM-AM)
 1/(1+b+bc+bcd) = a/(a+ab+abc+abcd) ≧ a/(a+ab+abc+1),
 1/(1+c+cd+cda) = ab/{ab+abc+abcd(1+a)} ≧ ab/(ab+abc+1+a),
 1/(1+d+da+dab) = abc/{abc+abcd(1+a+ab)} ≧ abc/(abc+1+a+ab),
 (左辺) ≧ 1,
 等号条件から a=b=c=d=1.
0068132人目の素数さん
垢版 |
2019/03/15(金) 05:38:56.61ID:dNAYt8q7
>>66

bを底辺としたときの高さ(辺bに下した垂線の長さ)m はc以下だから
 2 = bm ≦ bc ≦ bb,
0071132人目の素数さん
垢版 |
2019/03/25(月) 11:16:31.87ID:PVNNz8L/
n番目の素数をp(n)とおくとき、n≧5に対して、
p(n+1)^3 < Π[k=1 to n] p(k).
0072132人目の素数さん
垢版 |
2019/03/28(木) 19:15:24.33ID:dISuNBxT
Crux (2018年度)から。

4302, 4304, 4308, 4309, 4310
https://cms.math.ca/crux/v44/n1/Problems_44_1.pdf
4311, 4316, 4319, 4320
https://cms.math.ca/crux/v44/n2/Problems_44_2.pdf
4321, 4322, 4327, 4329
https://cms.math.ca/crux/v44/n3/Problems_44_3.pdf
4335, 4336, 4340
https://cms.math.ca/crux/v44/n4/Problems_44_4.pdf
4348, 4349, 4350
https://cms.math.ca/crux/v44/n5/Problems_44_5.pdf
4353, 4359, 4360
https://cms.math.ca/crux/v44/n6/Problems_44_6.pdf
4367
https://cms.math.ca/crux/v44/n7/Problems_44_7.pdf
4376、4377, 4380 ←ハァハァ
https://cms.math.ca/crux/v44/n8/Problems_44_8.pdf
4388, 4389
https://cms.math.ca/crux/v44/n9/Problems_44_9.pdf
4398, 4399
https://cms.math.ca/crux/v44/n10/Problems_44_10.pdf

春ですな ( ゚∀゚) ウヒョッ!
0073132人目の素数さん
垢版 |
2019/03/31(日) 20:44:10.54ID:cXTAnoiE
>>71
nについての帰納法で・・・・

・n=5 のとき
 (左辺) = p(6)^3 = 13^3 = 2197,
 (右辺) = p(1)p(2)p(3)p(4)p(5) = 2・3・5・7・11 = 2310,
により成立。

・n>5 のとき
 ベルトラン予想(チェビシェフの定理)より p(n+2)/p(n+1) < 2,
∴ {p(n+2)/p(n+1)}^3 < 8 < p(n+1),
n について成立すれば n+1 のついても成立する。(終)


さくら、さくら、今 咲き誇る
http://www.youtube.com/watch?v=PB-IdtDRY1I
0074132人目の素数さん
垢版 |
2019/04/01(月) 04:26:16.64ID:Ga8zedWm
>>72
crux/v44/n1/Problems_44_1

4304
Evaluate
cot(π/7) + cot(2π/7) + cot(4π/7) + {cot(π/7)}^3 + {cot(2π/7)}^3 + {cot(4π/7)}^3,

4306-改
Prove that
  √(16n+24) > √n + √(n+1) + √(n+2) + √(n+3) > √(16n+20),

4308.
Let a,b & c be positive real numbers. Prove that
  27abc(aab+bbc+cca) ≦ (a+b+c)^2・(ab+bc+ca)^2,

4309.
Let a,b & c be real numbers such that a+b+c=3. Prove that
  2(a^4 + b^4 + c^4) ≧ ab(ab+1) + bc(bc+1) + ca(ca+1).
0075132人目の素数さん
垢版 |
2019/04/01(月) 04:35:08.54ID:Ga8zedWm
>>74

4304.
cot(π/7) + cot(2π/7) + cot(4π/7) = √7,
{cot(π/7)}^3 + {cot(2π/7)}^3 + {cot(4π/7)}^3 = 18/√7,
∴ 25/√7.


4306-改
右)
 √n + √(n+3) = √{(2n+3) + 2√(n(n+3))} ≧ √{(2n+3) + 2(n+1)} = √(4n+5),

 {√(n+1) - √n} - {√(n+3) - √(n+2)} = 1/{√(n+1) + √n} - 1/{√(n+3) + √(n+2)} > 0,

 √n + √(n+1) + √(n+2) + √(n+3) ≧ 2√(4n+5),

左) GM-AM より
 √n + √(n+1) + √(n+2) + √(n+3) ≦ 4√(n+3/2) = √(16n+24),

4308.
 A = aab+bbc+cca, B = abb+bcc+caa, C = 3abc とおくと与式は
 9CA ≦ (A+B+C)^2,
(u+v+w)^2 ≧ 3(uv+vw+wu) より
 B^2 ≧ 3abc(aab+bbc+cca) = CA,
 A+B+C ≧ A + √(CA) + C ≧ 3√(CA),

4309.
(解1)
 a^4 + b^4 + c^4 ≧ (1/3)(aa+bb+cc)^2 ≧ (1/9)(aa+bb+cc)(a+b+c)^2 = aa+bb+cc ≧ ab+bc+ca,
 a^4 + b^4 + c^4 ≧ (ab)^2 + (bc)^2 + (ca)^2,
 辺々たす。
(解2)
 a^4 + b^4 + c^4 ≧ (1/3)(aa+bb+cc)^2 ≧ (1/27)(a+b+c)^4 = (1/3)(a+b+c)^2 ≧ ab+bc+ca,
 a^4 + b^4 + c^4 ≧ (ab)^2 + (bc)^2 + (ca)^2,
 辺々たす。

http://cms.math.ca/crux/v45/n1/Solutions_45_1.pdf
0076132人目の素数さん
垢版 |
2019/04/01(月) 04:41:05.15ID:Ga8zedWm
>>72
crux/v44/n2/Problems_44_2

4316.
Let f:[0,11] be an integrable and convex function. Prove that
∫[3,5] f(x)dx + ∫[6,8] f(x)dx ≦ ∫[0,2] f(x)dx + ∫[9,11] f(x)dx,

(略解)
下に凸だから
 f(3+t) ≦ {f(t) + f(t) + f(9+t)}/3,
 f(6+t) ≦ {f(t) + f(9+t) + f(9+t)}/3,
辺々たす。
 f(3+t) + f(6+t) ≦ f(t) + f(9+t),
0≦t≦2 で積分する。

4317.
Solve the following system of equations over reals:
a+b+c+d = 4,
abc + bcd+ cda + dab = 2,
abcd = -1/4,

(略解)
 ab+ac+ad+bc+bd+cd = 3√3, 
となるから a〜d は↓の実根。
0 = t^4 -4t^3 +(3√3)t^2 -2t -1/4 = {t-(1+√3)/2}^3 {t-(5-3√3)/2},
∴ a〜d = (1+√3)/2 = 1.36602540378444 (3重根)
      (5-3√3)/2 = -0.0980762113533

4320.
For positive real numbers a,b,c,d, prove that
(a+b)(a+b)(a+c)(b+c)(b+d)(c+d) ≧ (a+b+c+d)(abcd)^(5/4),
0078132人目の素数さん
垢版 |
2019/04/01(月) 21:13:27.29ID:2IqqjnJM
整理して頂き、有難う御座いまする。

最近の数オリに不等式が少ないのは、ネタ切れなのかな?
0079132人目の素数さん
垢版 |
2019/04/03(水) 07:14:05.62ID:/TkvX91f
>>72
crux/v44/n3/Problems_44_3

4321.
Find the greatest positive real number k such that
 (aa + bb + cc + dd + ee)^2 ≧ k(a^4 + b^4 + c^4 + d^4 + e^4)
for all real numbers a,b,c,d & e satisfying a+b+c+d+e=0.

 k = 20/13, 等号成立は {1,1,1,1,-4} のとき。 

4325.
Solve in real numbers the system of equations:
 x^4 -2y^3 -x^2 +2y = -1 +2√5,
 y^4 -2x^3 -y^2 +2x = -1 -2√5,

 x = (1+√5)/2 = φ = 1.61803399
 y = (1-√5)/2 = -1/φ = - 0.61803399
 (x, y) ≒ (1.8 1.5) 付近では接触しないようでござる。

4327.
Prove the following inequality for all x>0:
arctan(x) arctan(1/x) < π/{2(xx+1)}.

4330.
Let a & b be integers such that aa -20b +24 = 0.
Find the complete set of solutions of the following equation over integers:
 5xx + axy + byy = 11.

a = 2(5n+7), b = 5nn+14n+11
 (x, y) = (n, -1) (-n, 1) (3n+4, -3) (-3n-4, 3)

a = 2(5n-7), b = 5nn-14n+11
 (x, y) = (n, -1) (-n, 1) (3n-4, -3) (-3n+4, 3)
0081132人目の素数さん
垢版 |
2019/04/07(日) 06:46:37.06ID:d5M1c3zz
>>79

4327-改
Prove the following inequality for all x>0:
 arctan(x) arctan(1/x) < πx/{2(xx+1)},
(略解)
x⇔1/x としても不変だから、0<x≦1 としてよい。
 arctan(x) < x,
 arctan(1/x) < π/{2(xx+1)},
辺々掛ける。


〔補題〕
0<x≦1 のとき
 arctan(x) > πxx/{2(xx+1)},
 arctan(1/x) < π/{2(xx+1)},
(略証)
・0 < x < 2/π のとき
 arctan(x) = ∫[0,x] 1/(tt+1) dt > x/(xx+1) > πxx/{2(xx+1)},
 arctan(1/x) = (π/2) - arctan(x) < π/{2(xx+1)},

・(4-π)/π < x ≦ 1 のとき
 arctan(x) = (π/4) - ∫[x,1] 1/(tt+1)dt
 ≧ (π/4) - (1-x)/(xx+1)
 = πxx/{2(xx+1)} + (1-x){πx - (4-π)}/{4(xx+1)}
 ≧ πxx/{2(xx+1)},
 arctan(1/x) = (π/2) - arctan(x) ≦ π/{2(xx+1)},
0082132人目の素数さん
垢版 |
2019/04/07(日) 08:12:11.85ID:d5M1c3zz
>>72
crux/v44/n4/Problems_44_4

4335.
Let a & b be fixed positive real numbers and let n≧2 be an integer.
Prove that for any non-negative real numbers x_i, (i=1,2,・・・・,n) such that x1 + x2 + ・・・・ + xn = 1, we have
 (a・x_1 + b)^(1/3) + (a・x_2 + b)^(1/3) + ・・・・ + (a・x_n + b)^(1/3) ≧ (a+b)^(1/3) + (n-1)b^(1/3).

f(x) = (ax+b)^(1/3) は上に凸だから、Jensenで
 f(x) ≧ x・f(0) + (1-x)・f(1),

4336.
For non-negative integers m & n, evaluate in closed form
  Σ[k=0,n] Σ[j=0,m] (j+k+1)C[j+k,j]

 1 + (mn+m+n)(m+n+3)!/{(m+2)! (n+2)!},

4340.
Let a,b,c & d be positive real numbers such that
  a + b + c + d = 1/a + 1/b + 1/c + 1/d.
Show that
  a + b + c + d ≧ max{ 4√(abcd), 4/√(abcd) }.
0083132人目の素数さん
垢版 |
2019/04/07(日) 09:02:34.31ID:d5M1c3zz
>>72
crux/v44/n5/Problems_44_5.pdf

4346.
Find all x,y,z ∈ (0,∞) such that
 64(x+y+z)^2 = 27(xx+1)(yy+1)(zz+1),
 x+y+z = xyz.

 (x+i)(y+i)(z+i) = (xyz-x-y-z) + (xy+yz+zx-1)i
より
 (xx+1)(yy+1)(zz+1) = (xyz-x-y-z)^2 + (xy+yz+zx-1)^2,
与式より
 x+y+z = xyz = ±(3√3)/8・(xy+yz+zx-1)
 ξ^3 -s・ξ^2 +{1±(8/3√3)s}・ξ -s = 0 の3根。
 x = y = z = √3,

4348.
Let p∈[0,1]. Then for each n>1, prove that
 (1-p)^n + p^n ≧ (2pp-2p+1)^n + (2p-2pp)^n.

4349.
Let x,y & z be positive real numbers such that x+y+z = 3.
Find the minimum value of
  (x^3)/{y√(x^3 +8)} + (y^3)/{z√(y^3 +8)} + (z^3)/{x√(z^3 +8)}.

4350.
Let f:[0,1]→R be a decreasing, differentiable and concave function.
Prove that
 f(a) + f(b) + f(c) + f(d) ≦ 3f(0) + f(d-c+b-a),
for any real numbers a,b,c,d such that 0≦a≦b≦c≦d≦1.

単調減少 だから
 f(a) ≦ f(0),
 f(b) ≦ f(b-a),
 f(c) ≦ f(0),
 f(d) ≦ f(d-c),
下に凸 だから
 f(b-a) + f(d-c) ≦ f(0) + f(d-c+b-a),
辺々たす。
0084132人目の素数さん
垢版 |
2019/04/07(日) 11:19:48.06ID:d5M1c3zz
>>72
crux/v44/n6/Problems_44_6

4353.
Evaluate
 lim[n→∞] (1/n)Σ[k=1,∞) Σ[j=1,n] 1/{k C[j+k-1,j]}.

・k=1
 Σ[j=1,n] 1/C[j,j] = n
・k=2
 Σ[j=1,n] 1/{2 C[j+1,j] = Σ[j=1,n] 1/{2(j+1)} 〜 (1/2)log(n),
・k>2
 1/C[j+k-1,j] = ((k-1)/(k-2)){1/C[j+k-1,j] - 1/C[j+k,j+1]},
 Σ[j=1,n] 1/C[j+k-1,j] = ((k-1)/(k-2)){1/k - 1/C[n+k,n+1]}.


4356.
Solve the following system over reals:
 a + b + c + d = 6,
 aa + bb + cc + dd = 12,
 abc + bcd + cda + dab = 8 + abcd.

これらより
 ab + ac + ad + bc + bd + cd = 12,
 abc + bcd + cda + dab = 8,
 abcd = 0,
 0 = t^4 -6t^3 +12t^2 -8t = t(t-2)^3,
 {a,b,c,d} = {0,2,2,2}

4359.
Let a,b & c be positive real numbers.
Prove that
  3 ln(a^b + b^c + c^a) + a/c + b/a + c/b ≧ a+b+c + ln(27).

4360.
Let a,b,c be non-negative real numbers such that a+b+c = 1.
Find the minimum and the maximum values of the expression
  (a+b)/(1+ab) + (b+c)/(1+bc) + (c+a)/(1+ca).
When do these extreme values occur ?

 min. = 9/5,  {a,b,c} = {1/2,1/2,0} {1/3,1/3,1/3}
 max. = 2,  {a,b,c} = {1,0,0}
0085132人目の素数さん
垢版 |
2019/04/07(日) 12:06:02.16ID:d5M1c3zz
>>72
crux/v44/n6/Problems_44_7

4367.
Let a, b & c be distinct complex numbers such that |a| = |b| = |c| = 1 and |a+b+c| ≦ 1.
Prove that
  |(a+b)/(a-b)||(b+c)/(b-c)| + |(b+c)/(b-c)||(c+a)/(c-a)| + |(c+a)/(c-a)||(a+b)/(a-b)| = 1,

4370.
Solve the following system of equations:
 a + b + c + d = 4,
 aa + bb + cc + dd = 7,
 abc + bcd + cda + dab - abcd = 5/16.

これらより
 ab + ac + ad + bc + bd + cd = 9/2,
 abc + bcd + cda + dab = 1,
 abcd = 1/16,
 0 = t^4 -4t^3 +(9/2)t^2 -t +(1/16) = (tt-2t+1/4)^2,
 t = 1 ±(√3)/2,  (重根)
0086132人目の素数さん
垢版 |
2019/04/08(月) 05:42:05.41ID:QMWP0bri
4368.
Calculate
 Σ[n=2,∞) (2^n)[ζ(n) -1 -1/(2^n)]

(与式) = Σ[n=2,∞) Σ[k=3,∞) (2/k)^n
 = Σ[k=3,∞) (2/k)^2 /(1 - 2/k)
 = Σ[k=3,∞) 4/{k(k-2)}
 = Σ[k=3,∞) {2/(k-2) - 2/k}
 = 2/1 + 2/2
 = 3,

-----------------------------------

 crux/v44/n8/Problems_44_8

4377.
Let x≧y≧z >0 such that x+y+z + xy+yz+zx = 1 + xyz.
Find min x.

与式より
 xy+yz+zx -1 = xyz-x-y-z = A,
 (x+i)(y+i)(z+i) = (xyz-x-y-z) + (xy+yz+zx-1)i = A(1+i),
 (x,y,z;A) = (7,3,3;50) (8,5,2;65) (13,4,2;85)

4378.
Find all k such that the following limit exists.
  lim[n→∞) {k・F_(n+1) - Σ[i=0,n] φ^i} = 0,
where F_n is the n-th Fibonacci number and φ is the golden ratio.

 F_n = {φ^n - (-1/φ)^n}/√5,  (Binetの公式)
 k = (√5)φ,
0087132人目の素数さん
垢版 |
2019/04/08(月) 08:43:35.55ID:QMWP0bri
>>72
 crux/v44/n9/Problems_44_9

4383.
Evaluate the inegral
 ∫[0,1] (ln x)・√{x/(1-x)} dx.

4388.
 For positive real numbers a,b & c, prove
  8abc(aa+2ca+bc)(bb+2ab+ca)(cc+2bc+ab) ≦ (27/64){(a+b)(b+c)(c+a)}^3.

4389.
Considerthe real numbers a,b,c & d.
Prove that
 a(c+d) - b(c-d) ≦ √{2(aa+bb)(cc+dd)}.

 {a(c+d) - b(c-d)}^2 + {a(c-d) + b(c+d)}^2 = 2(aa+bb)(cc+dd),
あるいは
 (a+bi)(c-di)(1+i) = {a(c+d) - b(c-d)} + {a(c-d) +b(c+d)}i,
 (a-bi)(c+di)(1-i) = {a(c+d) - b(c-d)} - {a(c-d) +b(c+d)}i,
辺々掛ける。

4390.
Let x,y & z be positive real numbers with x+y+z = m.
Find the minimum value of the expression
  1/(1+xx) + 1/(1+yy) + 1/(1+zz).
0088132人目の素数さん
垢版 |
2019/04/09(火) 00:51:03.91ID:sDGeXCoR
>>87
4383.

(略解)
 x = (sinθ)^2 とおくと
 dx = 2 sinθ cosθ dθ
 (与式) = 4∫[0,π/2] log(sinθ) (sinθ)^2 dθ
 = 2∫[0,π/2] log(sinθ) {1 - cos(2θ)} dθ
 = 2I - ∫[0,π/2] log(sinθ) 2cos(2θ) dθ
 = 2I - [ log(sinθ) sin(2θ) ](0,π/2) + ∫[0,π/2] {cos(2θ)+1} dθ
 = 2I + [ {-log(sinθ) + 1/2} sin(2θ) + θ ](0,π/2)
 = 2I + π/2
 = -π{log(2) - 1/2}       (*)

-----------------------------------------------
*) 次を使った。

[例3]
 ∫[0,π/2] log(sinθ) dθ = - (π/2)log(2).    (Euler)

被積分函数は θ→0 のとき -∞ になるが、θ^a log(sinθ) = (θ^a)logθ + (θ^a)log(sinθ/θ) → 0 (a>0) だから、
積分は収束する。(定理36)
この積分を I とすれば θ を π-θ に,また π/2-θ に変換して
  I = ∫[π/2, π] log(sinθ) dθ,  I = ∫[0,π/2] log(cosθ) dθ.
故に
  2I = ∫[0,π] log(sinθ) dθ.
ここで θ=2φ とすれば
  I = ∫[0,π/2] log(2φ) dφ = ∫[0,π/2] log(2 sinφ cosφ) dφ
   = ∫[0,π/2] log(2) dφ + ∫[0,π/2] log(sinφ) dφ + ∫[0,π/2] log(cosφ) dφ,
   = (π/2)log(2) + I + I.
よって標記の結果を得る。

高木:「解析概論」改訂第三版、岩波書店(1961) p.113
  第3章 積分法、§34. 積分変数の変換、[例3]
0089132人目の素数さん
垢版 |
2019/04/10(水) 11:48:38.47ID:x+zqr5Tw
>>87
4390.
 0 < m < √2 のとき
  1 + 1 + 1/(1+mm) ≦ (与式) ≦ 3/(1+mm/9),
 √2 < m < √3 のとき
  1 + 2/(1+mm/4) ≦ (与式) ≦ 3/(1+mm/9),
 √3 < m < √6 のとき
  1 + 2/(1+mm/4) ≦ (与式) ≦ 1 + 1 + 1/(1+mm),
 √6 < m のとき
  3/(1+mm/9) ≦ (与式) ≦ 1 + 1 + 1/(1+mm),


>>72
 crux/v44/n10/Problems_44_10

4398.
Prove that for n∈N, we have
 1/(2n-1) + ∫[0,1] {sin(x^n)}^2 dx ≧ (2/n){1-cos(1)}.

4399.
Let ABCDE be a pentagon. Prove that
 |AB||EC||ED| + |BC|ED||EA| + |CD||EA||EB| ≧ |AD||EB||EC|.
When does equality hold ?
0090132人目の素数さん
垢版 |
2019/04/10(水) 12:33:36.53ID:x+zqr5Tw
>>.76
 crux/v44/n2/Problems_44_2
4317.
他にもまだあった。
・ab+ac+ad+bc+bd+cd = 3√3 のとき
 a〜d = (1+√3)/2 = 1.36602540378444 (3重根)
     (5-3√3)/2 = -0.09807621135332
・ab+ac+ad+bc+bd+cd = -3√3 のとき
 a〜d = (1-√3)/2 = -0.36602540378444 (3重根)
     (5+3√3)/2 = 5.09807621135332

4320.
 (a+b)(b+c)(c+d)(d+a) - (a+b+c+d)(abc+bcd+cda+dab) = (ac-bd)^2 ≧ 0,
より
 (左辺)^2 ≧ {(a+b+c+d)(abc+bcd+cda+dab)}^3,

 (左辺) ≧ {(a+b+c+d)(abc+bcd+cda+dab)}^(3/2)
  ≧ 8(a+b+c+d)^(3/2)・(abcd)^(9/8)
  ≧ 16(a+b+c+d)・(abcd)^(5/4),
右辺の係数16が抜けてました。スマソ

http://cms.math.ca/crux/v45/n2/Solutions_45_2.pdf
0091132人目の素数さん
垢版 |
2019/04/11(木) 01:59:40.35ID:Ue9ZzVLN
>>90
〔補題〕
(1/16) (a+b+c+d)^4
 ≧ (4/9) (ab+ac+ad+bc+bd+cd)^2
 ≧ { (a+b)(b+c)(c+d)(d+a) + (a+c)(c+d)(d+b)(b+a) + (a+d)(d+b)(b+c)(c+a) } /3
 ≧ (a+b+c+d) (abc+bcd+cda+dab)
 ≧ 16 abcd,

(略証)
 s = a+b+c+d, t = ab+ac+ad+bc+bd+cd, u = abc+bcd+cda+dab, v = abcd とおく。

 3ss - 8t = (a-b)^2 + (a-c)^2 + (a-d)^2 + (b-c)^2 + (b-d)^2 + (c-d)^2 ≧ 0,

 8tt - 6(a+b)(b+c)(c+d)(d+a) - 6(a+c)(c+d)(d+b)(b+a) - 6(a+d)(d+b)(b+c)(c+a)}
 = {(a-b)(c-d)}^2 + {(a-c)(b-d)}^2 + {(a-d)(b-c)}^2 ≧ 0,

 (a+b)(b+c)(c+d)(d+a) - su = (ac-bd)^2 ≧ 0,

 su - 16v = ab(c-d)^2 + ac(b-d)^2 + ad(b-c)^2 + bc(a-d)^2 + bd(a-c)^2 + cd(a-b)^2 ≧ 0,
0092132人目の素数さん
垢版 |
2019/04/19(金) 07:57:02.99ID:mM9/LvYY
>>87
訂正

4388.
For positive real numbers a,b & c, prove
 8abc (aa+2ca+bc)(bb+2ab+ca)(cc+2bc+ab) ≦ {(a+b)(b+c)(c+a)}^3.
0094132人目の素数さん
垢版 |
2019/04/23(火) 17:56:42.13ID:7u2F758f
>>76 (追加)

4319.
Let x_1,x_2,・・・・,x_n ∈ (0,+∞), n≧2, α≧3/2,
such that (x_1)^α + (x_2)^α + ・・・・ +(x_n)^α = n.
Prove the following inequality:
  Π[i=1,n] {1 +x_i + x_i^(α+1)} ≦ 3^n.

(略証)
 x ≦ (α-1 +x^α)/α より
 1 + x + x^(α+1) ≦ 1 + (1 +x^α)(α-1 +x^α)/α
 = 1 + (1+X)(α-1 +X)/α
 = 3 + (1+2/α)(X-1) +(1/α)(X-1)^2
 ≦ 3 + (1+2/α)(X-1) +(1/8)(1+2/α)^2・(X-1)^2  (← α≧3/2)
 = 3{1 +y +(3/8)yy}
 ≦ 3 e^y,           (←補題)
ここに X = x^α, y = (1/3)(1+2/α)(X-1),
題意により
 y_1 +y_2 + ・・・・ +y_n = (1/3)(1+2/α)(X_1 +X_2+・・・・+X_n -n) = 0, 
 (左辺) ≦ (3^n)e^(y_1+y_2+・・・・+y_n) = 3^n.

〔補題〕
 y > -0.9323381774 のとき 1 +y +(3/8)yy < e^y.

 x>0, X>0, α≧3/2 のとき y > -7/9 > -0.9323381774

さて、補題をどう示すか・・・・
0095132人目の素数さん
垢版 |
2019/04/24(水) 00:49:44.98ID:vK+1FJs+
>>85
4367.
O(0), A(a), B(b), C(c) とおく。
 題意より A,B,Cは単位円上にあり、僊BC は鋭角三角形。
 ∠A = α, ∠B = β, ∠C = γ とおくと tanα>0, tanβ>0, tanγ>0,
 (a+b)/(a-b) = -i/tan(∠AOB/2) = -i/tanγ, etc.

 (左辺) = 1/(tanγ・tanα) + 1/(tanα・tanβ) + 1/(tanβ・tanγ)
 = (tanβ + tanγ + tanα)/(tanα・tanβ・tanγ)
 = 1,     (α+β+γ=π より)
0096132人目の素数さん
垢版 |
2019/04/24(水) 13:53:15.76ID:vK+1FJs+
>>93

4325.
 xx-x-1 = 0, yy-y-1 = 0
を元の式に入れて
2√5 = x^4 -2y^3 -x^2 +2y +1 = (xx+x+1)(xx-x-1) -2(y+1)(yy-y-1) +2(x-y) = 2(x-y),
-2√5 = y^4 -2x^3 -y^2 +2x +1 = (yy+y+1)(yy-y-1) -2(x+1)(xx-x-1) +2(y-x) = 2(y-x),

これから x,y が出る。
0097132人目の素数さん
垢版 |
2019/04/25(木) 15:23:38.86ID:4OvWo35u
>>94
 模範解答は・・・・

4319.
X = x^α,
f(X) = log{1 + X^(1/α) + X^(1+1/α)}
とおくと
f "(X)・αα・X^(2-1/α)・exp{2f(X)} = -α(α+1)X^(2+1/α) -2αX^(1+1/α) -αX^(1/α) +(α+1)X -(α-1)
 < -2αX^(1+1/α) + (α+1)X - (α-1)
 < -α[2X^(α+1)]^(1/α) + (α+1)X - 1/2   (← α≧3/2)
 < 0,
より f(X) は X>0で上に凸。

∵ (α/(α+1))・2X^(1+1/α) + (α-1)/(α+1)
 > (α/(α+1))[2X^(α+1)]^(1/α) + (1/(α+1))・(1/2)  (← α≧3/2)
 > X            (← Jensen)

http://cms.math.ca/crux/v45/n2/Solutions_45_2.pdf
0098132人目の素数さん
垢版 |
2019/05/01(水) 06:57:59.14ID:bWsqQfPq
>>79

4321.
|a| ≧ |b|,|c|,|d|,|e| としてもよい。このとき
 5aa - S_2 = 5aa - (aa+bb+cc+dd+ee) ≧ 0,
 aa = (-b-c-d-e)^2 ≦ 4(bb+cc+dd+ee) = 4(S_2-aa),
 ∴ 4S_2 -5aa ≧ 0,  (等号は b=c=d=e のとき)
 2aa -S_2 = (-b-c-d-e)^2 -bb -cc -dd -ee = 2(bc+bd+be+cd+ce+de),
よって
S_4 = a^4 + b^4 + c^4 + d^4 + e^4
  = a^4 + (S_2 -aa)^2 -2(bbcc+bbdd+bbee+ccdd+ccee+ddee)
  ≦ a^4 + (S_2 -aa)^2 -(1/3)(bc+bd+be+cd+ce+de)^2
  = a^4 + (S_2 -aa)^2 -(1/12)(2aa -S_2)^2
  = (13/20)(S_2)^2 - (1/15)(5aa -S_2)(4S_2 -5aa)
  ≦ (13/20)(S_2)^2,

http://cms.math.ca/crux/v45/n3/Solutions_45_3.pdf
0099132人目の素数さん
垢版 |
2019/05/09(木) 01:31:57.01ID:7Q6cd3gq
>>85
4367-改.
Let a,b & c be distinct complex numbers such that |a| = |b| = |c| = 1.
Prove that
 ((a+b)/(a-b))((b+c)/(b-c)) + ((b+c)/(b-c))((c+a)/(c-a)) + ((c+a)/(c-a))((a+b)/(a-b)) = -1.

>>95
(略証)
指数関数の加法公式より
 sin(α+β+γ) = Im{e^(i(α+β+γ))}
 = Im{e^(iα)・e^(iβ)・e^(iγ)}
 = Im{(cosα+i・sinα)(cosβ+i・sinβ)(cosγ+i・sinγ)}
 = cosα・sinβ・cosγ + cosα・cosβ・sinγ + sinα・cosβ・cosγ - sinα・sinβ・sinγ
 = sinα・sinβ・sinγ{1/(tanγ・tanα) + 1/(tanα・tanβ) + 1/(tanβ・tanγ) - 1},
0100132人目の素数さん
垢版 |
2019/05/18(土) 13:58:35.67ID:SPl7kJbB
〔補題〕
0<θ<π/2 のとき
 sinθ < H < θ < G < A < tanθ,

ここで H = 3sinθ/(2+cosθ), G = {(sinθ)^2・tanθ}^(1/3), A = (2sinθ+tanθ)/3 である。
(略証)
 cos(x) < 3{1+2cos(x)}/{2+cos(x)}^2 < 1 < {2cos(x)^2 +1}/{3cos(x)^(4/3)} < {2cos(x) + 1/cos(x)^2}/3 < 1/cos(x)^2,
をxで積分する。(0→θ)
 H < θ を B.C.Carlson と呼び、θ < A を Snellius-Huygens と呼ぶ。

[第2章.196-199,679]
[第3章.565,591]
[第6章.610-613,634,641]
[第7章.156-157,929]
[第9章.762-763]
0101132人目の素数さん
垢版 |
2019/05/18(土) 14:27:46.32ID:SPl7kJbB
θ = π/12 = π/3 - π/4 = π/4 - π/6 とおくと
加法公式により
sinθ = (√6 - √2)/4,
cosθ = (√6 + √2)/4,
tanθ = 2 - √3,
より
 12H = 36(√3 -1)/(1+4√2 +√3) = 3.14150999
 12G = 6(√3 -1)/(1+√3)^(1/3) = 3.141927918
 12A = 2(√6 -√2) + 4(2-√3) = 3.14234913
一方、
 √2 + √3 = 3.1462643699
0103132人目の素数さん
垢版 |
2019/05/19(日) 12:44:34.35ID:9g/K/0vL
〔問題〕
ζ(2) = Σ[k=1,∞] 1/kk = (log 2)^2 + Σ[k=1,∞] 2/(kk・2^k)
を示せ。
 (不等式ぢゃねぇが、バーゼル問題に関連あり)
0104132人目の素数さん
垢版 |
2019/05/19(日) 12:51:11.72ID:9g/K/0vL
>>103
マクローリン展開
 Σ[k=1,∞] (1/k)x^(k-1) = -(1/x)log(1-x),
より
 Σ[k=1,∞] 1/(kk・2^k) = -∫[0〜1/2] (1/x)log(1-x) dx,
 Σ[k=1,∞] {1/kk - 1/(kk・2^k)} = -∫[1/2〜1] (1/y)log(1-y) dy,
辺々引く。
 ζ(2) - Σ[k=1,∞] 2/(kk・2^k)
 = -∫[1/2〜1] log(1-y)/y dy + ∫[0〜1/2] (1/x)log(x) dx,
 = -∫[0〜1/2] log(x)/(1-x) dx + ∫[0〜1/2] (1/x)log(1-x) dx
 = [ log(x)log(1-x) ](x=0,1/2)
 = (log 1/2)^2
 = (log 2)^2
 = 0.4804530139182

http://club.informatix.co.jp/?p=3326

・数列総合スレ
http://rio2016.5ch.net/test/read.cgi/math/1290234907/203-205

・オイラーの贈物スレ
http://rio2016.5ch.net/test/read.cgi/math/1417406099/244-247

・円周率について語り合おう【π】
http://rio2016.5ch.net/test/read.cgi/math/1326599636/304-306
0105132人目の素数さん
垢版 |
2019/05/19(日) 16:33:59.17ID:FEiCVf4L
実数 x,y が x^2 + y^2 = 1 をみたすとき、f(x,y) = 15x^2 + 10xy - 9y^2 の最大値を求めよ。
何通りのやり方があるかな?
0106132人目の素数さん
垢版 |
2019/05/19(日) 19:02:51.22ID:9g/K/0vL
解I.
f(x,y) = 16(xx+yy) - (-x+5y)^2 ≦ 16(xx+yy) = 16,

なお、最小値は
f(x,y) = (5x+y)^2 - 10(xx+yy) ≧ -10(xx+yy) = -10,

解II.
軸を回して
 (5x+y)/√26 = u,
 (-x+5y)/√26 = v,
とおく。
 f(x,y) = {16(5x+y)^2 - 10(-x+5y)^2}/26 = 16uu - 10vv,
0107132人目の素数さん
垢版 |
2019/05/20(月) 00:59:07.00ID:zfRpln9i
解III.
ラグランジュの未定乗数法
 F(x,y;λ) = f(x,y) - λ(xx+yy-1)
とおく。
 ∂F/∂x = ∂f/∂x -2λx = 30x +10y -2λx = 0,
 ∂F/∂y = ∂f/∂y -2λy = 10x -18y -2λy = 0,
これらが自明でない解(x,y)≠(0,0) をもつことから
 (30-2λ)(-18-2λ) = 10^2,
 λ = -10, 16
0110132人目の素数さん
垢版 |
2019/05/20(月) 23:48:57.47ID:zfRpln9i
〔問題〕 2018-09
正の整数nに対し、|xx-yy| がn以下の奇数であるような整数x,yの組の個数をf(n), |xx-yy| がn以下の偶数であるような整数x,yの組の個数をg(n)で表わす。このとき
 | f(n) - g(n) - (4log2)n | < 12√n,
が成り立つことを示せ。

http://www.toshin.com/concours/img/mondai_20180820.jpg


〔問題〕 2018-10
[0,1] で定義された連続な実数値関数fが(0,1)で連続な導関数f' をもち、f(0)=0, f(1)=1 を満たすとき、
 ∫[0,1] {f(x)}^2 dx + ∫[0,1] {f'(x)}^2 dx
のとり得る最小値を求めよ。

http://www.toshin.com/concours/img/mondai_20180920.jpg


〔問題〕 2019-03
nを正の整数、sを1より大きい実数とする。
0以上の実数 a1,a2,・・・・,an が a1+a2+・・・・+an = s を満たすとき、
Σ[k=1,n] (ak)^(k+1) を最大、および最小にする a1,a2,・・・・,an の組(a1,a2,・・・・,an)がそれぞれ一つずつ存在することを示せ。

http://www.toshin.com/concours/img/mondai_20190220.jpg
0111132人目の素数さん
垢版 |
2019/05/21(火) 01:04:52.11ID:GJ4Ie8EK
2018-10

f(x) を微小変化させる。
 f(x) → f(x) + (x)   ただし (0) = (1) = 0,
 f '(x) → f '(x) + '(x)
与式の変化分は
 ∫[0,1] 2f(x)・(x)dx + ∫[0,1] 2f '(x) '(x) dx
 = ∫[0,1] 2f(x)・(x)dx + 2f '(1)(1) - 2f '(0)(0) - ∫[0,1] 2(d/dx)f '(x) (x) dx
 = ∫[0,1] 2{f(x) - (d/dx)f '(x)}(x)dx,
任意の微小変化凾ノ対して非減少だから
 f(x) - (d/dx)f '(x) = 0,     ・・・・ Euler-Lagrange 方程式
これを解くと
 f(x) = a・e^x - b・e^(-x),
f(0)=0, f(1)=1 より a,bを求める。
 f(x) = sinh(x)/sinh(1),
これを与式に入れて
 1/tanh(1) = (e+1/e)/(e-1/e) = 1.3130353
0112132人目の素数さん
垢版 |
2019/05/21(火) 21:17:26.14ID:GJ4Ie8EK
例1
f(x) = tan(πx/4) のとき
 f '(x) = π/{4・cos(πx/4)^2},
 積分値 π/3 - 1 + 4/π = 1.3204371

例2
f(x) = x^c (c>1/2) のとき
 f '(x) = c x^(c-1)
 積分値 1/(2c+1) + cc/(2c-1) ≧ 1.32015717
 等号は c = 1.132557 (4c^4 -7cc +3c -1 = 0 の根) のとき

たしかに 1.3130353 より大きい。
0113132人目の素数さん
垢版 |
2019/05/21(火) 21:23:37.75ID:TfDD7bUD
a,b,c,d >0 に対して、
(a^3+b^3)(a^3+c^3)(a^3+d^3)(b^3+c^3)(b^3+d^3)(c^3+d^3)
≧ {(abc)^2 + (bcd)^2 + (cda)^2 + (dab)^2}^3.
0114132人目の素数さん
垢版 |
2019/05/22(水) 11:07:06.07ID:7SUOfge7
>>111
例3
f(x) = mx + (1-m)x^3 (0<m<1) のとき
 f '(x) = m + 3(1-m)xx,
 積分値 4(51-39m+23mm)/105 ≧ 151/115 = 1.313043478
 等号は m = 39/46 = 0.8478261 のとき。

 m = 1/sinh(1) = 0.8509181 のときは 1.31305185
0115132人目の素数さん
垢版 |
2019/05/23(木) 00:14:12.95ID:pMxXR6IF
例4
f(x) = arcsin(sin(1)・x) のとき 
 f '(x) = sin(1)/√{1 - sin(1)^2・xx},
 積分値 2/tan(1) -1 + sin(1)arctanh(sin(1)) = 1.31599

例5
f(x) = {e^(cx) - 1}/(e^c - 1), (c>0) のとき
 f '(x) = c・e^(cx)/(e^c - 1),
 積分値 {(c+2)cosh(c) + (cc-c-1)sinh(c) -2}/{2c[cosh(c)-1]} ≧ 1.31387
 等号は c = 0.46729 のとき。
0116132人目の素数さん
垢版 |
2019/05/23(木) 01:25:06.19ID:pMxXR6IF
>>113
(a^3+b^3)(c^3+d^3) = (AA+BB)(CC+DD)
 = {(AA+BB)/2}CC + BB{(CC+DD)/2} + AA{(CC+DD)/2} + {(AA+BB)/2}DD
            ≧ ABCC + BBCD + AACD + ABDD,
同様にして
(a^3+c^3)(b^3+d^3) ≧ ACBB + CCBD + ACDD + AABD,
(a^3+d^3)(b^3+c^3) ≧ AABC + DDBC + ADCC + ADBB,
辺々掛けてコーシーで
 (左辺) ≧ {(ABC)^(4/3) + (BCD)^(4/3) + (CDA)^(4/3) + (DAB)^(4/3)}^3
 = {(abc)^2 + (bcd)^2 + (cda)^2 + (dab)^2}^3.
0118132人目の素数さん
垢版 |
2019/05/26(日) 08:56:15.60ID:ijxfgc+2
crux/v45/n1/Problems_45_1

4403.
 Let m be an integer with m>1. Evaluate in closed form
  Σ[k=1,n] (-1)^(k-1) C[n+1,k+1] k/(m+k) = {1 - m!(n+1)!/(m+n)!} /(m-1),

4408.
 Let α∈(0,1]∪[2,∞) be a positive real number and let a,b & c be non-negative real numbers.
 Prove that
  a^α + b^α + c^α + (a+b+c)^α ≧ (a+b)^α + (b+c)^α + (c+a)^α.

4410.
 Prove that
 ∫[0,π/4] √sin(2x) dx < √2 - π/4 = 0.6288154

(解答例)
sin(2x) ≦ min{2x, 1} より
 (与式) < ∫[0,1/2] √(2x) dx + ∫[1/2,π/4] dx = π/4 - 1/6 = 0.6187315
0119132人目の素数さん
垢版 |
2019/05/26(日) 11:11:36.98ID:ijxfgc+2
 //cms.math.ca/crux/v45/n2/Problems_45_2.pdf

OC418.
Three sequences (a_0,a_1,・・・・,a_n), (b_0,b_1,・・・・,b_n), (c_0,c_1,・・・・,c_2n) of non-negative real numbers are given such that for all 0≦i,j≦n we have a_i・b_j≦{c_(i+j)}^2.
Prove that
 Σ[i=0,n] a_i・Σ[j=0,n] b_j ≦ (Σ[k=0,2n] c_k)^2


4417.
Let a,b & c be positive real numbers.
Further, let x,y & z be real numbers such that xy+yz+zx > 0.
Prove that
 (yy+zz)a + (zz+xx)b + (xx+yy)c ≧ 2(xy+yz+zx)(abc)^(1/3).

(解答例) AM-GM で
 (yy+zz)(zz+xx)(xx+yy) = (Y+Z)(Z+X)(X+Y)
 = (X+Y+Z)(XY+YZ+ZX) - XYZ
 ≧ (8/9)(X+Y+Z)(XY+YZ+ZX)
 ≧ {(2/3)(xy+yz+zx)}^3.

4418.
Consider a convex cyclic quadrilateral with sides a,b,c,d & area S.
Prove that
 (a+b)^5 /(c+d) + (b+c)^5 /(d+a) + (c+d)^5 /(a+b) + (d+a)^5 /(b+c) ≧ 64SS.

(解答例)
 (ab+cd)/2 ≧ S,
 (bc+da)/2 ≧ S,
 (左辺) ≧ (a+b)^4 + (b+c)^4 + (c+d)^4 + (d+a)^2
 ≧ (4ab)^2 + (4bc)^2 + (4cd)^2 + (4da)^2
 ≧ 8(ab+cd)^2 + 8(bc+da)^2
 ≧ 8(2S)^2 + 8(2S)^2
 = 64SS.
0120132人目の素数さん
垢版 |
2019/05/26(日) 15:48:57.31ID:ijxfgc+2
 //cms.math.ca/crux/v45/n3/Problems_45_3.pdf

4429.
Let a,b,c be positive real numbers. Prove that
 √{(aa+bb+cc)/2(ab+bc+ca)} ≧ (a+b+c)/{√(a(b+c)) + √(b(c+a)) + √(c(a+b))}.
0121132人目の素数さん
垢版 |
2019/05/26(日) 16:13:40.74ID:ijxfgc+2
 //cms.math.ca/crux/v45/n4/Problems_45_4.pdf

4431.
Let x,y≧0 and x+y=2. Prove that
 √(xx+8) + √(yy+8) + √(xy+8) ≧ 9.

(解答例)
 {√(xx+8) + √(yy+8)}^2 = (xx) + (yy+8) + 2√(xx+8)√(yy+8)
 = 14 + (x+y)^2 + 2(1-xy) + 2√{49 + 8(x+y)^2 + 14(1-xy) + (1-xy)^2}
 = 18 + 2(1-xy) + 2√{81 + 14(1-xy) + (1-xy)^2}
 ≧ 18 + 2(1-xy) + 2{9 + (7/9)(1-xy)}
 = 36 + (32/9)(1-xy),
∴ √(xx+8) + √(yy+8) ≧ 6 + (2/9)(1-xy)
 (xy+8) = 9 - (1-xy) ≧ {3 - (2/9)(1-xy)}^2,
よって
 (左辺) ≧ {6 + (2/9)(1-xy)} + {3 - (2/9)(1-xy)} = 9,
0122132人目の素数さん
垢版 |
2019/05/26(日) 19:12:28.07ID:ijxfgc+2
〔類題〕
x, y≧0 かつ x+y = 2 のとき
 √(xx+8) + √(yy+8) + 2√(xy+8) ≦ 12,
0123132人目の素数さん
垢版 |
2019/05/27(月) 00:02:09.94ID:EyPYWN4T
>>114
例6
f(x) = mx + nx^3 + (1-m-n)x^5 (0<m, 0<n, m+n<1) のとき
 f '(x) = m + 3nxx + 5(1-m-n)x^4,
 積分値 (4/3465)(1660mm +1164mn +263nn -2990m -1065n +2485) ≧ 15331/11676 = 1.31303528606
 等号は m = 3785/4448 = 0.850944245  n = 630/4448 = 0.14163669  1-m-n = 33/4448 = 0.007419065 のとき。

これは 1/tanh(1) = (ee+1)/(ee-1) = 1.31303528550 より大きい。

なお m = 1/sinh(1) = 0.850918128  n = 1/{6sinh(1)} = 0.14181969  1-m-n = 0.0072621837 のとき 1.31303529111
0125132人目の素数さん
垢版 |
2019/05/28(火) 06:11:55.41ID:xWwuUG0H
>>113
(a^3+b^3)(b^3+c^3)(c^3+d^3)(d^3+a^3) = (A+B)(B+C)(C+D)(D+A)
 = (A+B+C+D)(ABC+BCD+CDA+DAB) + (AC-BD)^2
 ≧ (A+B+C+D)(ABC+BCD+CDA+DAB),            >>90

(左辺) ≧ (A+B+C+D)^(3/2)・(ABC+BCD+CDA+DAB)^(3/2)
 ≧ 4 (ABC + BCD + CDA + DAB)^2
 = 4 {(abc)^3 + (bcd)^3 + (cda)^3 + (dab)^3}^2
 ≧ {(abc)^2 + (bcd)^2 + (cda)^2 + (dab)^2}^3,
0126132人目の素数さん
垢版 |
2019/05/29(水) 14:50:44.53ID:pVbDo3+L
>>118
4408.
α-1 ≦ 0 または α-1 ≧ 1 ゆえ
x^(α-1) は下に凸。
 a^(α-1) + (a+b+c)^(α-1) ≧ (a+b)^(α-1) + (c+a)^(α-1),
 a^α + a(a+b+c)^(α-1) ≧ a(a+b)^(α-1) + a(c+a)^(α-1),
循環的にたす。

α≧2 のとき
(左辺) - (右辺) = α(α-1)(α-2)∫[0,a] ∫[0,b] ∫[0,c] (x+y+z)^(α-3) dx dy dz ≧ 0,

α≦1 のとき
(左辺) - (右辺) = (-α)(1-α)(2-α)∫[a,∞] ∫[b,∞] ∫[c,∞] (x+y+z)^(α-3) dx dy dz ≧ 0,

α≧2 のとき
(左辺) - (右辺) = α(α-1)(α-2)∫[0,a] ∫[0,b] ∫[0,c] (x+y+z)^(α-3) dx dy dz ≧ 0,
0127132人目の素数さん
垢版 |
2019/05/30(木) 00:23:28.75ID:S7fbSkoD
>>92
4388.
u = abc,
p = aab +bbc +cca,
q = abb +bcc +caa,
とおくと
 p+q+2u = (a+b)(b+c)(c+a),
 q-p = (a-b)(b-c)(c-a) = ,
 4u ≦ p+u, 4u ≦ q+u,
また
 b(aa+2ca+bc) = p+2u -cca,
 c(bb+2ab+ca) = p+2u -aab,
 a(cc+2bc+ab) = p+2u -bbc,
より
 (左辺) = 8(p+2u -cca)(p+2u -aab)(p+2u -bbc)
 = 8{(p+2u)^3 -p(p+2u)^2 +(p+2u)qu -u^3}
 = 8u{2(p+u)^2 + (p+u)(q+u) + 3(p+u)u + (q+u)u}
 ≦ 8u{2(p+u)^2 + (p+u)(q+u) + (p+u)(q+u)}
 ≦ 16u(p+u)(p+q+2u)
 = 4(p+u)(q+u)(p+q+2u)
 = (p+q+2u)^3 - (p+q+2u)(p-q)^2
 = {(a+b)(b+c)(c+a)}^3 - (a+b)(b+c)(c+a)刧,
0128132人目の素数さん
垢版 |
2019/05/30(木) 11:42:57.13ID:S7fbSkoD
>>83
4348.
 q = 2p(1-p) とおくと
1/2 - q = 2(1/2 - p)^2 ≦ |1/2 - p|,
∴ 1-q, q は 1-pとpの間にある。Jensen より
 (1-p)^n + p^n ≧ (1-q)^n + q^n,

4349.
 t/√(t+8) は単調増加だからチェビシェフにより
 (与式) ≧ xx/√(x^3 +8) + yy/√(y^3 +8) + zz/√(z^3 +8),

Max{x,y,z} = X ≧2 のとき
 X^4 - (X^3 +8) = (X-2)(X^3 +XX+2X+4) ≧ 0,
 (与式) > 1,
x,y,z ≦ 2.7945 のとき
 xx/√(x^3 +8) ≧ (11x-5)/18, etc.
 (与式) ≧ {11(x+y+z)-15}/18 = 1,
 等号は x=y=z=1 のとき。
0129132人目の素数さん
垢版 |
2019/06/01(土) 02:44:07.56ID:6xVHiY/M
〔問題074〕
△ABCが鋭角三角形のとき
 {sin(A)+sin(B)+sin(C)} / {cos(A)+cos(B)+cos(C)}
の取りうる値の範囲を求めよ。

 大学への数学 2012年/Dec. 宿題
 [第6章.872-873]
 Inequalitybot [074]
0130132人目の素数さん
垢版 |
2019/06/01(土) 03:20:03.90ID:6xVHiY/M
古い問題だ・・・・

(略解)
 x = cos((A-B)/2) ∈ [cos(C/2), 1] とおく。

(与式) = {2cos(C/2)・x + sin(C)} / {2sin(C/2)・x + cos(C)}
 = tan(C) + 2cos(3C/2)・x / {[2sin(C/2)・x + cos(C)]cos(C)} = g(x),

g(cos(C/2)) = 1 + 1/{sin(C)+cos(C)} ∈ [1 + 1/√2, 2)

A+B+C=180゚ ゆえ (0, 60゚] の角と [60゚, 90゚] の角がある。

0<C≦60゚ とすれば cos(3C/2) ≧ 0, g(x)は単調増加,
 g(x) ≧ g(cos(C/2)) ≧ 1 + 1/√2, (最小)
 等号は x = cos(C/2) = 1, {A, B} = {45゚, 90゚}, C=45゚ のとき

60゚≦C≦90゚ とすれば cos(3C/2) ≦ 0, g(x)は単調減少,
 g(x) ≦ g(cos(C/2)) < 2, (上限)
 等号は x = cos(C/2) = 1, A=B→90゚, C→0゚ のとき
0131132人目の素数さん
垢版 |
2019/06/01(土) 03:27:51.68ID:6xVHiY/M
最後は
 g(x) ≦ g(cos(C/2)) < 2, (上限)
 等号は x = cos(C/2) = 1/√2, {A, B}→{0゚,90゚}, C=90゚ のとき
でした。
0132132人目の素数さん
垢版 |
2019/06/05(水) 03:41:46.20ID:+pVPgegT
〔補題555〕
X,Y,Z ≧ 0, X+Y+Z = S のとき
 S/(1 + S/3) ≧ X/(1+X) + Y/(1+Y) + Z/(1+Z) ≧ S/(1 + S),

分かスレ478-555
0134132人目の素数さん
垢版 |
2019/06/10(月) 06:36:27.58ID:0ZLkhJ7v
>>83

4346.
 x = tan(A), y = tan(B), z = tan(C) とおくと
sin(A+B+C) = cos(A)cos(B)cos(C){tan(A) + tan(B) + tan(C) - tan(A)tan(B)tan(C)}
 = cos(A)cos(B)cos(C)(x+y+z-xyz) = 0   (← 与式)
∴ A+B+C = π,
{A,B,C} は三角形の頂角をなす。
フランダースの不等式から
 sin(A)sin(B)sin(C) ≦ {(√3)/2}^3,

一方、与式から
 8tan(A)tan(B)tan(C) = 8xyz = 8(x+y+z) = (3√3)/{cos(A)cos(B)cos(C),
より
 sin(A)sin(B)sin(C) = {(√3)/2}^3,
等号成立条件から A=B=C =π/3, x=y=z = √3,

>>128

4349.
コーシーで
(左辺) ≧ {f(x)+f(y)+f(z)}^2 /(xy+yz+zx) ≧ (1/3){f(x)+f(y)+f(z)}^2,  (←題意)
ここに
f(t) = tt/(t^3 +8)^(1/4)
f "(t) = (1/32){9t^6 + (t^3 -64)^2}/(t^3 +8)^(9/4) > 0,  (t>-2)
f(t) は t>-2 で下に凸。
f(t) ≧ (23t-11)/(12√3),
f(x)+f(y)+f(z) ≧ 3f((x+y+z)/3) = 3f(1) = √3,
(左辺) ≧ 1,

http://cms.math.ca/crux/v45/n4/Solutions_45_4.pdf
0135132人目の素数さん
垢版 |
2019/06/10(月) 06:42:18.15ID:0ZLkhJ7v
>>118

4410.
 y = √x は上に凸だから
 1 + √sin(2x) ≦ (√2)√{1+sin(2x)} = (√2)[sin(x)+cos(x)],
よって
 (与式) ≦ ∫[0,π/4] {(√2)[sin(x)+cos(x)] - 1} dx = √2 - π/4,

なお、 (与式) = Γ(3/4)^2 / √(2π) = 0.5990701173678

>>134
フランダースぢゃなかった・・・
0136132人目の素数さん
垢版 |
2019/06/10(月) 07:17:01.72ID:0ZLkhJ7v
>>118 >>135

4410.
√sin(2x) < √(2x)              (0<x<π/12)
√sin(2x) < (√2)[sin(x)+cos(x)] - 1   (π/12<x<π/4)
とすると
(与式) < ∫[0,π/12] √(2x) dx + ∫[π/12,π/4] {(√2)[cos(x)+sin(x)] -1} dx
= (1/3)(π/6)^(3/2) + (1 - π/6)
= 0.602693468

(与式) = 0.599070117
0137132人目の素数さん
垢版 |
2019/06/10(月) 17:22:10.92ID:0ZLkhJ7v
>>118
4410.
ジョルダンの不等式
 (√2)|sin(x -π/4)| ≧ |1 - 4x/π|    (0≦x≦π/2)
から
 yy = sin(2x) = cos(2x-π/2) = 1 - 2sin(x-π/4)^2 ≦ 1 - (1-4x/π)^2,
これは楕円の1/4
・中心 (π/4, 0)
・長半径1, 短半径π/4
・面積 (π/4)^2 = 0.616850275
0138132人目の素数さん
垢版 |
2019/06/11(火) 00:54:55.96ID:a3rUuuK+
>>118 >>137
4410.
ジョルダンの不等式
 |sin(π/4 - x)| ≧ (3/π)|π/4 - x|  (π/12 < x < 5π/12)
から
 yy = sin(2x) = cos(π/2 - 2x) = 1 - 2sin(π/4 - x)^2 ≧ 1 - (1/bb)(π/4 - x)^2,
これは楕円の一部
・中心 (π/4,0)
・長半径 a=1, 短半径 b=π/(3√2),
・面積 ab(π+2)/8,

(与式) < ∫[0,π/12] √(2x) dx + ∫[π/12,π/4] a√{1 - (1/bb)(π/4 - x)^2} dx
 = (1/3)(π/6)^(3/2) + ab・(π+2)/8
 = (1/3)(π/6)^(3/2) + π(π+2)/(24√2)
 = 0.12629224364 + 0.47590613074
 = 0.60219837438
0139132人目の素数さん
垢版 |
2019/06/11(火) 07:27:30.94ID:a3rUuuK+
>>118 >>137 >>138
4410.
 √sin(2x) < √(2x),     (0<x<π/12)
 √sin(2x) < {1 + sin(2x)}/2, (π/12<x<π/4)  GM-AM
とすると
 (与式) < ∫[0,π/12] √(2x) dx + ∫[π/12,π/4] {1+sin(2x)}/2 dx
 = (1/3)(π/6)^(3/2) + [ x/2 - cos(2x)/4 ](x=π/12,π/4)
 = (1/3)(π/6)^(3/2) + π/12 + (1/8)√3
 = 0.1262922436 + 0.2617993878 + 0.2165063510
 = 0.6045979824
0140132人目の素数さん
垢版 |
2019/06/14(金) 22:03:00.11ID:SNdqfAoO
a,b,c >0 に対して、
Σ[cyc] (a+b)^2/{(b+c)^2 + (c+a)^2} ≦ 3(a+b)(b+c)(c+a)/(16abc).

既出でおぢゃるかな?
0144132人目の素数さん
垢版 |
2019/06/14(金) 22:12:01.88ID:SNdqfAoO
三角形の辺長 a,b,c に対して、
a^3/(b+c-a) + b^3/(c+a-b) + c^3/(a+b-c) ≧ 4*(2R-r)^2
0147132人目の素数さん
垢版 |
2019/06/15(土) 08:30:02.99ID:d+NNwnLK
>>145
 (aab+bbc+cca +1) - (aa+bb+cc) = 1 - aa(1-b) - bb(1-c) - cc(1-a)
 ≧ 1 - a(1-b) - b(1-c) - c(1-a)
 = (1-a)(1-b)(1-c) + abc
 ≧ 0,

>>146
そりゃ、KoMaL
C.1552
 Prove that if 0<a<1 and 0<b<1 then log_a{2ab/(a+b)}・log_b{2ab/(a+b)} ≧ 1,

 A = log(a) < 0, B = log(b) < 0,
 2ab/(a+b) ≦ √(ab),
 log_a{2ab/(a+b)} = log{2ab/(a+b)}/ log(a) ≧ log(ab)/{2log(a)} = (A+B)/(2A) > 0,
 log_b{2ab/(a+b)} ≧ (A+B)/(2B) > 0,
辺々掛けて
 (左辺) ≧ (A+B)^2 /(4AB) ≧ 1,
0149132人目の素数さん
垢版 |
2019/06/16(日) 07:07:43.58ID:HwIhVDIU
>>110 (下)

〔問題〕 2019-03
・最大は a_1 = ・・・・ = a_(n-1) = 0, a_n = s のときで  (s>1)
 最大値 s^(n+1).
・極小点では
 (k+1)(a_k)^k = 2t  (未定乗数)
より
 a_1 = t, a_k = {2t/(k+1)}^(1/k),
このとき
 f(t) = t + Σ[k=2,n] {2t/(k+1)}^(1/k)
は単調増加で f(0)=0, f(s) > s だから
f(t)=s は 0<t<s にただ一つの解をもつ。

n=2のとき t = s - {√(1+6s) - 1}/3,
0150132人目の素数さん
垢版 |
2019/06/16(日) 10:38:35.53ID:HwIhVDIU
〔補題554〕
0<k<1 のとき
(1) √{1 - (k・sinφ)^2} ≧ (cosφ)^2 + (sinφ)^2・√(1-kk),
(2) E(k) = ∫[0,π/2] √{1 - (k・sinφ)^2} dφ ≧ (π/4){1 + √(1-kk)},
     E(k)は第二種の完全楕円積分

分かスレ453−554,555
0151132人目の素数さん
垢版 |
2019/06/16(日) 20:46:24.95ID:HwIhVDIU
〔補題559〕
0<k<1 のとき
(1')  2√(1-kk/2) ≧ √{1 - (k・sinφ)^2} + √{1 - (k・cosφ)^2} ≧ 1 + √(1-kk),
(2')  (π/2)√(1-kk/2) ≧ E(k) ≧ (π/4){1 + √(1-kk)},
0152132人目の素数さん
垢版 |
2019/06/18(火) 06:37:15.80ID:1unLBUnb
〔問題541〕
長半径a, 短半径a√(1-kk) である楕円の周長を K,
半径 a の円周の長さを L,
半径 a√(1-kk) の円周の長さを M とする。すなわち
 K = 4a E(k),
 L = 2πa,
 M = 2πa√(1-kk),
とするとき
 √{(LL+MM)/2} ≧ K ≧ (L+M)/2,
 等号成立は K=L=M (k=0) のとき。
分かスレ453-541

〔問題537〕
zは複素数で |z|≦1 のとき
 |(1+z)exp(-z) - 1| ≦ |z|^2
 等号成立は z=-1 のとき。
分かスレ453-537
0154132人目の素数さん
垢版 |
2019/06/20(木) 01:05:37.07ID:WxweZeE5
>>152 (下)

(左辺) = |∫[0,z] (-z') exp(-z') dz'|
≦ ∫(0,|z|) |z'| exp(|z'|) |dz'|
= ∫(0,|z|) r exp(r) dr    (r=|z'|)
= 1 - (1-|z|) exp(|z|)
≦ 1 - (1-|z|) (1+|z|)     (|z|≦1)
= |z|^2,
0155132人目の素数さん
垢版 |
2019/06/20(木) 03:01:03.41ID:WxweZeE5
>>141 >>142
(左辺) - (右辺) = 9abc + (1/a+1/b+1/c) -7 - 7(ab+bc+ca)
 ≧ 9abc + 9/(a+b+c) -7 - 7(ab+bc+ca)    (AM-HM)
 = 9abc + 2(a+b+c)^3 - 7(a+b+c)(ab+bc+ca)
 = (a+b+c){(a+b+c)^2 - 3(ab+bc+ca)} + {(a+b+c)^3 - 4(a+b+c)(ab+bc+ca) + 9abc}
 = (a+b+c)F_0(a,b,c) + F_1(a,b,c)
 ≧ 0,

〔Schurの不等式〕
a,b,c≧0 または n:偶数のとき
F_n(a,b,c) = (a^n)(a-b)(a-c) + (b^n)(b-c)(b-a) + (c^n)(c-a)(c-b) ≧ 0,
(略証)
bはaとcの中間にあるとして
 (a-b)(b-c) ≧ 0,  a^n - b^n + c^n ≧ 0,
F_n(a,b,c) = (a^n)(a-b)^2 + (a^n - b^n + c^n)(a-b)(b-c) + (c^n)(b-c)^2 ≧ 0,
0156132人目の素数さん
垢版 |
2019/06/20(木) 04:06:49.82ID:WxweZeE5
>>143

a/(b+c) + b/(c+a) + c/(a+b) ≧ (a+b+c)^2 /{a(b+c) + b(c+a) + c(a+b)}   (コーシー)
 = 1 + (1/2) (aa+bb+cc)/(ab+bc+ca)
 = 1 + (1/2)XX
 ≧ (1/2) + X,

∴ (左辺) ≧ (1/2) + X + 1/X ≧ 5/2,
0157132人目の素数さん
垢版 |
2019/06/25(火) 08:56:15.44ID:4AX2BJg5
>>118
4408.  a,b & c がベクトルのとき

α=2
 Sq.= |a|^2 + |b|^2 + |c|^2 + |a+b+c|^2 - |a+b|^2 - |b+c|^2 - |c+a|^2 = 0,   (← 内積)

α=1  (Hlawka)
  |a| + |b| + |c| + |a+b+c| - |a+b| - |b+c| - |c+a| ≧ 0,
(略証)
 (|a|+|b|+|c|+|a+b+c|) (左辺)
 = Sq. + (|b|+|c|-|b+c|)(|a|-|b+c|+|a+b+c|) + (|c|+|a|-|c+a|)(|b|-|c+a|+|a+b+c|) + (|a|+|b|-|a+b|)(|c|-|a+b|+|a+b+c|)
 ≧ 0.
 [初代スレ.354-360, 364]
 [第8章.388 (5), 450, 708, 795]
 文献[3] 大関、p.33-34 例題8.
0158132人目の素数さん
垢版 |
2019/06/26(水) 07:40:18.60ID:jPrPUfqH
>>70 >>140
コーシーで
 (a+b)^2 / {(a+c)^2 + (b+c)^2} ≦ aa/(a+c)^2 + bb/(b+c)^2,
よって
(左辺) ≦ (aa+bb)/(a+b)^2 + cyclic
 = 3/2 + (1/2){(a-b)/(a+b)}^2 + cyclic
 ≦ 3/2 + (a-b)^2 /(8ab) + cyclic
 = 3/2 + {c(a-b)^2 + a(b-c)^2 + b(c-a)^2}/(8abc)
 = 1/2 + (a+b)(b+c)(c+a)/(8abc),
0164132人目の素数さん
垢版 |
2019/07/04(木) 23:03:38.04ID:6zAehVNK
>>159

〔類題〕
 a,b,c は0でない実数で 1/a + 1/b + 1/c = -2 を満たす。 次を示せ。
 32(a^4 + b^4 + c^4 + a^3 + b^3 + c^3) - 8(8k-1)・(aa + bb + cc) - 64k・(a+b+c) + 16k(4k-3) - 16kk・(a+b+c)/abc ≧ 0,

 k≧0 とする。等号が成立するのはいつか ?
0165132人目の素数さん
垢版 |
2019/07/05(金) 00:07:26.17ID:g7vswyLm
>>163

左辺をSとおく。
 a/(b+c) +2b/(c+d) > (a+b)/(b+c) + (b+c)/(c+d) -1,
循環的に加えて AM-GM を使えば
 3 S > 2Σ[k=1,n] {a_k + a_(k+1)}/{a_(k+1) + a_(k+2)} - n > 2n - n = n.
ただし a_{n+1}=a_1, a_{n+2}=a_2 とした。
∴ S ≧ n/3.

[初代.497, 501-502] [第2章.284]
0166132人目の素数さん
垢版 |
2019/07/05(金) 02:15:12.85ID:g7vswyLm
>>163

S > (√2 -1)n

(略証)
 (a+b+c)(b+c+d) > (b+c)(a+b+c+d),
より
 {a/(b+c) + 1}{b/(c+d) + 1} > (a+b+c+d)/(c+d) = (a+b)/(c+d) + 1 ≧ 2√{(a+b)/(c+d)},
AM-GM より
 {a/(b+c) + b/(c+d)}/2 > (√2){(a+b)/(c+d)}^(1/4) - 1,
循環的に加えて AM-GM を使えば
 S > (√2 -1)n,
  (mahanmath および abch42 による。2011/May/08)

http://artofproblemsolving.com/community/c6h271096p1468831
0167132人目の素数さん
垢版 |
2019/07/06(土) 12:04:07.65ID:oiEnJ4mP
>>163

S_n > (γ/2)n = 0.494566817223496526 n
  γ は Drinfel'd 定数。

(文献)
・V. G. Drinfel'd: Math. Zametki, 9 (2), p.113-119 (1971) "A cyclic inequality"
・安藤哲哉: 「不等式」 数学書房 (2012) §5.2.8
0168132人目の素数さん
垢版 |
2019/07/07(日) 04:01:49.63ID:NQih2PzA
>>164

(略解)
 (与式) = 8{(2a+1)(aa-k)/a}^2 + 8{(2b+1)(bb-k)/b}^2 + 8{(2c+1)(cc-k)/c}^2 - (1/2)(1/a+1/b+1/c +2)^2
  = 8{(2a+1)(aa-k)/a}^2 + 8{(2b+1)(bb-k)/b}^2 + 8{(2c+1)(cc-k)/c}^2
  ≧ 0,
 等号成立は k>0 かつ {a,b,c} = { -1/2, -√k, √k} のとき。
0169132人目の素数さん
垢版 |
2019/07/07(日) 18:48:35.43ID:NQih2PzA
>>167

f(x) = e^(-x),
g(x) = 2/{exp(x) + exp(x/2)},
は下に凸である。
y=f(x) のグラフと y=g(x) のグラフの共通接線は1本だけ存在する。それを
 y = m・x + γ
とおく。、
 m = -0.8562482144492661168
 γ = (-m){1 - log(-m)},
0170132人目の素数さん
垢版 |
2019/07/09(火) 02:35:03.06ID:96Oo9tpn
>>167
nを固定して考える。
 S_n < (λ_n)・n,

n≦13 または n=15,17,19,21,23 のとき λ_n = 1/2,

一方、n=14 {0, 1+4δ, 2δ, 1+4δ, 4δ, 1+3δ, 5δ, 1+δ, 4δ, 1, 2δ, 1, 0, 1+2δ}
  δ=1/60 のとき 0.4999880721 n

λ_14 < 0.4999880721

λ_24 < 0.499197
  A.Zulauf: Math. Gazette, 43, p.182-184 (1959) "On a conjecture of L.J.Mordell II!

λ_111 < 0.49656
  D.E.Daykin: J. London Math. Soc.(2), 3, p.453-462 (1971) "Inequalities for functions of cyclic nature"

γ/2 = 0.4945668172235
  V.G.Drinfel'd: Math Notes, 9, p.68-71 (1971) "A cyclic inequality" (>>167 の英訳)

λ_{n+2} ≦ λ_n (?)
0171132人目の素数さん
垢版 |
2019/07/09(火) 22:57:04.96ID:96Oo9tpn
|a| = √{(a_1)^2 + (a_2)^2 + ・・・・ + (a_n)^2} とおく。

〔補題〕
a_1 ≧ a_2 ≧ ・・・・ ≧ a_n ≧ 0 のとき
 Σ[k=1,n] {√(n+1-k) - √(n-k)} a_k ≦ |a| ≦ Σ[k=1,n] {√k - √(k-1)} a_k,
等号成立は a_1 = a_2 = ・・・・・ = a_n のとき。

http://www.casphy.com/bbs/highmath/472060/ 不等式2-357
0173132人目の素数さん
垢版 |
2019/07/14(日) 15:04:25.43ID:Xfj84fYJ
>>84

4353.
 S(j) = (1/j)Σ[k=1,∞] 1/{k・C(j+k-1,j)}
  = (j-1)!Σ[k=1,∞] (k-1)!/{(k+j-1)!・k},
とおくと階差は
 S(j) - S(j+1) = 1/jj,
また
 S(1) = Σ[k=1,∞] 1/kk = ζ(2),
より
 S(j) = ζ(2) - Σ[k=1,j-1] 1/kk 〜 1/(j - 1/2),
 lim[n→∞] (1/n)Σ[j=1,n] j・S(j) = lim[n→∞] n・S(n) = 1,

4359.
 log(x) は上に凸だから Jensen で
 3log((a^b+b^c+c^a)/3) ≧ b・log(a) + c・log(b) + a・log(c),

 log(x) は上に凸だから
 log(x) = - log(1/x) ≧ - (1/x - 1) = 1 - 1/x,
0174132人目の素数さん
垢版 |
2019/07/14(日) 15:09:52.59ID:Xfj84fYJ
〔問題〕
 f(x) = e^(-x) とおくとき次を示せ。
 f(f(f(1))) > 1/2,

( //www.casphy.com/bbs/highmath/472060/ casphy!-高校数学-不等式2-359)
0175132人目の素数さん
垢版 |
2019/07/15(月) 12:26:35.29ID:hmERs5gh
π ≒ 22/7    (約率)

7π/2 = 11 - 0.0044257124

nが奇数のとき
 |sin(11n)| > 1 - (1/2)(0.0044257124 n)^2 = 1 - 0.0000098nn,

nが偶数のとき
 |sin(11n)| < 0.0044257124 n,

 (分かスレ454-178,188)
0176132人目の素数さん
垢版 |
2019/07/20(土) 11:09:50.76ID:bSAoQnjE
1000
ふうL@Fu_L12345654321
学コン1傑いただきました!
とても嬉しいです!

https://pbs.twimg.com/media/D-IuUuqVUAALnAB.jpg
https://twitter.com/Fu_L12345654321/status/1144528199654633477
https://twitter.com/5chan_nel (5ch newer account)
0177132人目の素数さん
垢版 |
2019/07/20(土) 17:29:12.98ID:E2uDcqfM
(1)
a,b,c≧0 に対して、
√(1+aa+bb+cc-ab-bc-ca) ≧ (a+b+c-abc)/2.

(2)
a,b,c>0 に対して、
{(aaa+bbb+ccc)/3}^(1/3) + (a+b+c)/9 ≧ 4/3.

(3)
a,b,c>0 に対して、
5(a/b + b/c + c/a)^2 ≧ 2(a+b+c)(1/a + 1/b + 1/c) + 27.

(4)
a,b,c>0, a+b+c=3 に対して、
(a^3+b^3)/(1+a) + (b^3+c^3)/(1+b) + (c^3+a^3)/(1+c) ≧ a^(5/2) + b^(5/2) + c^(5/2).

     /////
    /////_________
    /////  ̄ ̄ ̄ ̄ ̄ ̄| ̄ ̄
   /////   ___   (~) チリンチリン
   /////  /  ≧ \ ノ,,
  /////  |::::: (● (● |    
  /////   ヽ::::... .ワ.....ノ  お久しぶりです
 /////     ( つ且 ~ つ  不等式の夏ですね
0178132人目の素数さん
垢版 |
2019/07/21(日) 05:49:41.51ID:4k/Gtesi
>>175
π ≒ 355/113   (密率)
355 = 113π + 0.00003014435

|sin(355n)| < 0.00003014435・n

|cos(355n)| > 1 - (1/2)(0.00003014435・n)^2 = 1 - 4.5434102×10^(-10) nn
0179132人目の素数さん
垢版 |
2019/07/21(日) 06:49:52.30ID:4k/Gtesi
>>177

(2)
 A = (a+b+c)/3,
 Q = √{(aa+bb+cc)/3},
 T = {(aaa+bbb+ccc)/3}^(1/3),
とおく。
 T + (1/3)A
 = (T+T+T+A)/3
 ≧ (4/3)(TTTA)^(1/4)  (AM-GM)
 ≧ (4/3)Q,     (コーシー)

(3)
 (a/b+b/c+c/a)^2 = 2(a+b+c)(1/a+1/b+1/c) - 9 + (a/b-1)^2 + (b/c-1)^2 + (c/a-1)^2
  ≧ 2(a+b+c)(1/a+1/b+1/c) - 9,
 両辺に 36 をたす。
 等号は a=b=c
0180132人目の素数さん
垢版 |
2019/07/22(月) 05:15:00.06ID:vDQA99OD
>>177
(1)
s = a+b+c, t = ab+bc+ca, u = abc とおく。
(右辺) = (s-u)/2 ≦ 1 のときは明らか。
よって s > 2+u とする。

(左辺)^2 = 1 + ss -3t
 = 1 + ss/4 + (3/4s)(F_1 - 9u)
 ≧ 1 + ss/4 - 27u/(4s)
 ≧ 1 + {(s-u)/2}^2 + su/2 -uu/4 - 27u/(4s)  (← sに関して単調増加)
 ≧ 1 + {(s-u)/2}^2 + (2+u)u/2 -uu/4 -27u/[4(2+u)]   (← s>2+u)
 = {(s-u)/2}^2 + (8+u)(1-u)^2 /[4(2+u)]
 ≧ {(s-u)/2}^2,

F_1(a,b,c) = s^3 -4st +9u ≧ 0,     (Schur-1)
0183132人目の素数さん
垢版 |
2019/08/10(土) 15:56:05.98ID:oX3OQU5P
[Reverse triangle inequality]
If x,y,z >0 & y is between x and z, then
(x/y + y/x -2) + (y/z + z/y -2) ≦ (x/z + z/x -2),

(short proof)
(x-y)(y-z) ≧ 0,
RHS - LHS = (x-y)(y-z) (x+z)/(xyz) ≧ 0,
0184132人目の素数さん
垢版 |
2019/08/11(日) 23:03:44.95ID:GctloD3X
>>181

 d(x,y) := x/y + y/x -2 ≧ 0,
とおくと、
 LHS(n) = nn + Σ(i<j) d(x_i,x_j)
 RHS(n) = nn + [nn/4] d(a,b),
また、a≦y≦b ならば
 d(a,y) + d(y,b) ≦ d(a,b)  >>183

(略証)
nについての帰納法による。
n=2 のとき
 d(x_1, x_2) ≦ d(a,b).
ゆえ成立する。

n>2 のとき
 m := [n/2]
とおく。メジアン y := x_{m+1} を除く n-1 変数に対しては、帰納法の仮定より
 LHS(n-1) ≦ RHS(n-1) = (n-1)^2 + [(n-1)^2 /4] d(a,b).
また
 LHS(n) - LHS(n-1) = (2n-1) + Σ(i=1,m) d(x_i,y) + Σ(j=m+2,n) d(y,x_j)
 ≦ (2n-1) + Σ(i=1,m) d(a,y) + Σ(j=m+2,n) d(y,b)
 ≦ (2n-1) + m (d(a,y) + d(y,b))
 ≦ (2n-1) + m d(a,b).
したがって
 LHS(n) ≦ nn + ([(n-1)^2 /4] + [n/2])d(a,b) = nn + [nn/4]d(a,b) = RHS(n).
0185132人目の素数さん
垢版 |
2019/08/12(月) 16:47:05.09ID:uLwjs1DH
[ (n-1)^2 /4 ] + [ n/2 ] = [ nn/4 ]

(short proof)
δ = mod(n, 2)
δ = 0  (n:even)
δ = 1  (n:odd)
then
[ (n-1)^2 /4 ] = ((n-1)^2 -1+δ)/4,
[ n/2 ] = (n-δ)/2,
[ nn/4 ] = (nn-δ)/4,
0186132人目の素数さん
垢版 |
2019/08/13(火) 11:27:05.56ID:Tk2MgydX
任意の実数xに対して次の不等式が成立
sinx+sin(√2x)≦2-1/(100(1+xx))

出典 ピーター・フランクルの中学生でも分かる大人でも解けない問題集代数編
0188132人目の素数さん
垢版 |
2019/08/15(木) 00:49:47.21ID:RxBWT0Y0
 y = x - (π/2) - 2π[ (x+π/2) / 2π ],
とおくと
 -π ≦ y < π,
 1 - sin(x) = 1 - cos(y) ≧ 2(y/π)^2,
0189132人目の素数さん
垢版 |
2019/08/16(金) 01:59:00.40ID:oNtuWoss
 1 - cos(y) ≧ (2y/π)^2 = 0.4052847 yy, (|y|≦π/2)
 1 - cos(y) ≧ 1,      (|y|≧π/2)

 1 - cos(y) ≧ (1/2)(3y/π)^2 = 0.45594533 yy, (|y|≦π/3)
 1 - cos(y) ≧ 1/2,     (|y|≧π/3)

 1 - cos(y) ≧ {1 - cos(π/6)}(6y/π)^2 = 0.4886807 yy, (|y|≦π/6)
 1 - cos(y) ≧ 1 - cos(π/6) = (2-√3)/2 = 0.1339746,  (|y|≧π/6)

 1 - cos(y) ≧ {1 - cos(π/12)}(12y/π)^2 = 0.4971507 yy,  (|y|≦π/12)
 1 - cos(y) ≧ 1 - cos(π/12) = 1 - (1+√3)/(2√2) = 0.0340742  (|y|≧π/12)
0190132人目の素数さん
垢版 |
2019/08/21(水) 19:59:33.25ID:a2bL2fVn
△ABCの辺長 a,b,c、外接円、内接円、傍接円の半径 R, r, r[a], r[b], r[c] に対して、
1/(2R^3) ≦ r[a]/(a^4) + r[b]/(b^4) + r[c]/(c^4) + ≦ 1/(16r^3).
0191132人目の素数さん
垢版 |
2019/08/26(月) 23:23:29.14ID:ywejxerG
>>190
Aに対する傍接円の中心をOとすると、
  △ABC = △OAB + △OAC - △OBC.
  ∴ r[a] = 2S/(b+c-a).

示すべき不等式は a,b,c のみで表せるから、伝家の宝刀 "ぬるぽ変換"でなんとかなりそう。
※ ぬるぽ変換とは、x = (b+c-a)/2、y = (c+a-b)/2、z = (a+b-c)/2.


傍接円の半径なんて初めて求めたでござるよ。( ゚∀゚) スリスリ スリットォ!
0194132人目の素数さん
垢版 |
2019/08/28(水) 10:14:07.85ID:641rcCLM
B.5017.
 Is there a function f:R→R with the following properties:
 (1) if x1≠x2 then f(x1)≠f(x2),
 (2) there exist appropriate constants a,b > 0 such that
     f(xx) - {f(ax+b))}^2 > 1/4.
 for all x∈R ?

C.1532
 Show that if a,b,c are positive numbers and
   a+b+c ≧ 1/ab + 1/bc + 1/ca,
 then one of them is at least 1.

C.1552.
 Show that if 0<a<1 and 0<b<1 then
 log_a{2ab/(a+b)}・log_b{2ab/(a+b)} ≧ 1.
0195132人目の素数さん
垢版 |
2019/08/28(水) 10:31:15.35ID:641rcCLM
B.5017.
a,b > 0 とする。
2次方程式 xx = ax+b は相異なる2実根 x_1≠x_2 をもつ。
 f(x_i^2) = f(a・x_i+b) = y_i
とおくと、性質(2)から
 y_i - (y_i)^2 ≧ 1/4.
∴ 0 ≧ (y_i - 1/2)^2
∴ y_i = 1/2,
∴ f(a・x_1+b) = 1/2 = f(a・x_2+b),
性質(1) (fは単射) から
 a・x_1 + b = a・x_2 + b,
a>0 から
 x1 = x2   (矛盾)

C.1532.
 a+b+c ≧ 1/ab + 1/ca + 1/ab = (a+b+c)/abc,
 a+b+c>0 で両辺を割ると
 1 ≧ 1/abc,
 abc ≧ 1.

C.1552.
 log(a) < 0, log(b) < 0 より log(a)・log(b) > 0,
HM-GM より
 2ab/(a+b) ≦ √(ab),
 log(2ab/(a+b)) ≦ (1/2){log(a)+log(b)} < 0,
したがって
 (左辺)・log(a)・log(b) = {log(2ab/(a+b))}^2
  ≧ (1/4){log(a)+log(b)}^2
  ≧ log(a)・log(b),
これを log(a)・log(b) >0 で割る。
0196132人目の素数さん
垢版 |
2019/08/28(水) 14:17:25.93ID:641rcCLM
>>192
 同感でござる。
(右)は
 aa ≧ aa - (b-c)^2 = (a-b+c)(a+b-c),
 16SS = (a+b+c)(-a+b+c)(a-b+c)(a+b-c),  … ヘロン
 1/r[a] + 1/r[b] + 1/r[c] = 1/r,
より
 r[a]/aa = 2S/{(-a+b+c)aa} ≦ 2S/{(-a+b+c)(a-b+c)(a+b-c)}
 = (a+b+c)/8S = 1/(4r),

∴ r[a]/(a^4) + r[b]/(b^4) + r[c]/(c^4)
  ≦ (1/r[a] + 1/r[b] + 1/r[c])/(4r)^2 = 1/{r(4r)^2},
0197132人目の素数さん
垢版 |
2019/08/29(木) 12:56:45.26ID:V/0HLVAJ
>>190 >>192

(左)は
 a+b+c = 2S/r,
 abc = ab・2R sin(C) = 4SR,
 1/r[a] + 1/r[b] + 1/r[c] = 1/r,
より
 1/aa + 1/bb + 1/cc ≧ 1/ab + 1/bc + 1/ca
 = (a+b+c)/abc = (2S/r)/(4RS) = 1/(2Rr),

∴ コーシーで
 r[a]/(a^4) + r[b]/(b^4) + r[c]/(c^4)
 ≧ (1/aa + 1/bb + 1/cc)^2 / (1/r[a] + 1/r[b] + 1/r[c])
 = r (1/aa + 1/bb + 1/cc)^2
 ≧ r/(2Rr)^2
 = 1/(4RRr),
0200132人目の素数さん
垢版 |
2019/08/30(金) 08:39:13.74ID:3MesnOrF
お前らがここまで一生懸命書き込んで来たのに....
俺なんかがこんなに簡単に 200get していいの?😜
 (分かスレ455-200)
0201132人目の素数さん
垢版 |
2019/08/30(金) 08:53:58.84ID:3MesnOrF
>>199
 |x+y| ≦ |x|+|y| 等号成立は xy≧0 (同符号)

(1)
証:依題、x,y,z 中必有2↑同号。
 不妨設 x,y 同号, 則 |x+y|/(|x|+|y|) = 1.
 又 0≦|y+z|/(|y|+|z|)≦1, 0≦|z+x|/(|z|+|x|)≦1, 故 〜〜

(2) Let x,y & z are arbitrary real numbers (not all zero).
Prove that
  2/3 ≦ (|x+y|+|y+z|+|z+x|)/(|x|+|y|+|z|) ≦ 2,

(証) 題意より、x,y,z の中に必ず同符号の2つがある。
 xy≧0 と置くことを妨げないから
 (分子) = |x+y| + |y+z| + |z+x|
  = |x| + |y| + (2Max{|y|,|z|} -|y| -|z|) + (2Max{|x|,|z|} -|z| -|x|)
  = 2 Max{|y|,|z|} + 2 Max{|x|,|z|} -2|z|
  = 2 Max{M,|z|} + 2(Max{m,|z|} -|z|)
  ≧ 2 Max{|x|,|y|,|z|}
  ≧ (2/3)(|x|+|y|+|z|),
 ここに M = Max{|x|,|y|}, m = min{|x|,|y|} とおいた。
0202132人目の素数さん
垢版 |
2019/08/31(土) 06:25:54.80ID:45aPYUp8
〔補題〕
|x+z| + |x-z| = 2 Max{|x|, |z|},
|y+z| + |y-z| = 2 Max{|y|, |z|},

…とやるより、場合分けした方が簡単かな。
0203132人目の素数さん
垢版 |
2019/09/08(日) 05:19:16.22ID:BBoFSmJW
ジュニア数檻から。

(1) x,y∈R に対して、(xy+x+y-1)^2 / {(x^2+1)(y^2+1)} の取りうる値の範囲。

(2) a,b,c,d∈R に対して、(aa+ac+cc)(bb+bd+dd) の取りうる値の範囲。

(3) a,b∈N に対して、min{gcd(a,b+1), gcd(a+1,b)} ≦{√(4a+4b+5) - 1}/2
0204132人目の素数さん
垢版 |
2019/09/08(日) 15:16:47.18ID:NPxrtGxy
(1)
 (x+i)(y+i) = (xy-1) + (x+y)i,
∴ {(xy-1) + (x+y)}^2 + {(xy-1) - (x+y)}^2 = 2{(xy-1)^2 + (x+y)^2}
 = 2 |(xy-1) + (x+y)i|^2
 = 2 |(x+i)(y+i)|^2
 = 2 (xx+1)(yy+1),
 より [0,2]
 等号成立は (x-1)(y-1)=2  (直角双曲線)
(2)
 aa+ac+cc = (3/4)(a+c)^2 + (1/4)(a-c)^2 ≧ 0,
 bb+bd+dd = (3/4)(b+d)^2 + (1/4)(b-d)^2 ≧ 0,
 より [0,∞)
 等号成立は a=b=c=d=0
(3)
左辺をLとおく。
 Max{gcd(a,b+1), gcd(a+1,b)} ≧ L+1,  (← 互いに素)
 L(L+1) ≦ gcd((a+1)(b+1), ab) = gcd(a+b+1, ab) ≦ a+b+1,
0205132人目の素数さん
垢版 |
2019/09/08(日) 15:54:03.88ID:NPxrtGxy
訂正スマソ
(2)
 等号成立は a=c=0 または b=d=0,

(3)
 gcd(a,b+1)・gcd(a+1,b) = gcd((a+1)(b+1), ab) を使った。
0207132人目の素数さん
垢版 |
2019/09/11(水) 03:41:17.00ID:1IJ7FHVd
A1.
Let a,b,c be positive real numbers such that min(ab,bc,ca)≧1.
Prove that
 {(aa+1)(bb+1)(cc+1)}^(1/3) ≦ {(a+b+c)/3}^2 + 1.

A8.
Find the largest real constant a_n such that
for all positive real numbers x_1, x_2, ・・・・, x_n satisfying 0 < x_1 < x_2 < ・・・・ < x_n,
we have
 Σ[j=0,n-1] 1/{x_(j+1) - x_j} ≧ a_n・{Σ[k=0,n-1] k / x_(k+1) },
(x_0 = 0.)

答だけ書くと
a_1 = 1/2 = 0.5
a_2 = 12/25 = 0.48
a_3 = 0.4701514765959817784543884・・・・
  (190t^4 - 6561t^3 + 574t^2 + 329t + 391 = 0 の正根 )
a_4 = 0.4643963253583455727840309・・・・
  (489t^5 + 1965t^4 - 71t^3 + 602t^2 - 613t + 60 = 0 の正根 )
  ・・・・・・
a_n → 4/9 = 0.44444・・・・  (n→∞)
0208132人目の素数さん
垢版 |
2019/09/11(水) 03:52:19.18ID:1IJ7FHVd
A8.
(略解)
 3/{x_(j+1) - x_j} … 3個
 j / x_j      … j個
の(j+3)個で AM-HM すると
 9/{x_(j+1) - x_j} + jj/x_j ≧ (j+3)^2 /x_(j+1)
 9/{x_(j+1) - x_j} ≧ (j+3)^2 /x_(j+1) - jj/x_j,
j=0…n-1 でたして 9で割る。
 Σ[j=0,n-1] 1/{x_(j+1) - x_j} ≧ (4/9)Σ[k=0,n-1] (k+2) / x_(k+1),

A8. の右辺は (k+2)/ x_(k+1) と訂正....orz >>207
0210132人目の素数さん
垢版 |
2019/09/11(水) 23:38:19.31ID:1IJ7FHVd
>>207
A1.
 0≦k≦1,
 a ' = (1-k)a + kb,
 b ' = ka + (1-k)b,
とする。
 (a 'a '+1)(b 'b '+1) - (aa+1)(bb+1)
 = (D+ab)^2 + (-2D+aa+bb) +1 - (aa+1)(bb+1)
 = D{D + 2(ab-1)}
 ≧ 0   (← ab≧1)
ここに D = k(1-k)(a-b)^2 とおいた。

∴ F(a,b,c) = (aa+1)(bb+1)(cc+1)
は (a,b,c) について上に凸。
 F(a,b,c) ≦ F(a',b',c')
 ≦ ・・・・・
 ≦ F((a+b+c)/3, (a+b+c)/3, (a+b+c)/3)
0211132人目の素数さん
垢版 |
2019/09/12(木) 00:38:37.35ID:q8BKz8lq
>>206
A4.
Find all functions f:(0,∞) → (0,∞) such that
 for any x,y ∈ (0,∞),
  x・f(xx)・f(f(y)) + f(y・f(x)) = f(xy)・{f(f(xx)) + f(f(yy))}.

A7.
Find all functions f:R→R such that f(0)≠0 and
 for all x,y∈R,
  f(x+y)^2 = 2f(x)f(y) + max{f(xx+yy), f(xx)+f(yy)}.

例  f(x) = -1, f(x) = x-1,
0212132人目の素数さん
垢版 |
2019/09/12(木) 01:07:17.91ID:q8BKz8lq
>>210 (補足)
 0≦k≦1 より
 D = k(1-k)(a-b)^2 ≧ 0,
∴ a'b' = D + ab ≧ ab ≧ 1

〔別法〕
 d = (a+b+c)/3 とおく。
(aa+1)(bb+1) ≦ {[(a+b)/2]^2 + 1}^2,
(cc+1)(dd+1) ≦ {[(c+d)/2]^2 + 1}^2,
辺々掛けて
(aa+1)(bb+1)(cc+1)(dd+1) ≦ {[(a+b+c+d)/4]^2 + 1}^4
 = (dd+1)^4,
0214132人目の素数さん
垢版 |
2019/09/16(月) 01:34:11.70ID:FF+PWEgn
>>213
すべての実数xに対して定義された関数
 f(x) = cos(x) + cos((√2)x)
について、

(1) f(x) = 2 を満たすxの値をすべて求めよ。
また、f(x) は周期関数ではないことを証明せよ。

(2) t = 6726π のとき、すべてのxに対して 不等式
  | f(x+t) - f(x)| < 0.002
が成り立つことを証明せよ。
ただし、√2 = 1.41421356…… とする。
 (1986 山梨医科大)
0215132人目の素数さん
垢版 |
2019/09/16(月) 01:44:55.63ID:FF+PWEgn
>>214
(1)
f(x) = 2 ⇔
 cos(x) = cos((√2)x) = 1, ⇔
 x = 2mπ, (√2)x = 2nπ, (m,n∈Z) ⇒
 m√2 = n (m,n∈Z) ⇔
 m = n = 0,
よって x=0 のみ。
f(x)=2 となるxは0以外にないから、周期関数ではない。
0216132人目の素数さん
垢版 |
2019/09/16(月) 02:00:54.51ID:FF+PWEgn
>>214
(2)
t は 2π の整数倍だから
 cos(x+t) - cos(x) = 0,
また和積公式から
 |cos((√2)(x+t) - cos((√2)x)| = 2|sin((√2)(x+t/2) sin(t/√2)|
  ≦ 2|sin(t/√2)|
  = 0.001321107
0217132人目の素数さん
垢版 |
2019/09/16(月) 02:03:03.70ID:FF+PWEgn
>>187
p,q が自然数のとき  |√2 - (p/q)| ≧ (6-4√2)/qq,

(略証)
・q=1 のとき
 (左辺) ≧ √2 -1 = 0.41421356… > 0.34314575… = 6-4√2,
・q≧2, p/q ≧ 3/2 のとき
 (左辺) ≧ 3/2 - √2 = (6-4√2)/4 ≧ (6-4√2)/qq,
・p/q < 3/2 のとき
 1/qq ≦ |2qq - pp| = (√2 + p/q)|√2 - (p/q)| < (√2 + 3/2)|√2 - (p/q)|
 (左辺) > (6-4√2)/qq,

 ピーター・フランクルすれ-039
0218132人目の素数さん
垢版 |
2019/09/16(月) 16:29:27.78ID:FF+PWEgn
>>187
αは整数係数のn次方程式 f(x)=0 の根だから
 f(α) = 0,
因数定理より
 f(x) = f(x) - f(α) = (x-α)g(x,α)
p,q を整数(q≠0) とすれば
 (q^n)f(p/q) は0でない整数。
 1 ≦ |(q^n)f(p/q)| = |q|^n・|f(p/q)|
 = |q|^n・|p/q - α| g(p/q,α)
 ≦ |q|^n・|p/q - α| / c(α),
0219132人目の素数さん
垢版 |
2019/09/16(月) 16:49:30.42ID:FF+PWEgn
>>187
・n=2 の例
c(√2) = 2(√2 -1)^2 = 0.34314575  (3/2)
c(√3) = (1/2)(√3 -1)^2 = 0.26794919 (2/1)
c(√5) = 4(√5 -2)^2 = 0.22291236  (9/4)
c(√6) = (√6 -2)^2 = 0.20204103   (5/2)
c(√7) = (3/2)(3-√7)^2 = 0.18823820 (8/3)
c(√8) = (1/4)(√8 -2)^2 = 0.17157288 (3/1)
c(√10) = 6(√10 -3)^2 = 0.15800423  (19/6)
c(√11) = (3/2)(√11 -3)^2 = 0.15037689 (10/3)
c(√12) = (1/2)(4-√12)^2 = 0.14359354 (7/2)
c(√13) = 180(5√13 -18)^2 = 0.13867497 (649/180)
c(√14) = 2(4-√14)^2 = 0.13348181   (15/4)
c(√15) = (1/6)(√15 -3)^2 = 0.12701665 (4/1)
c(√17) = 8(√17 -4)^2 = 0.12123996  (33/8)
c(√18) = (4/9)(9-2√18)^2 = 0.117749006 (9/2)
c(√19) = (39/2)(3√19 -13)^2 = 0.11470688 (170/39)
c(√20) = (√20 -4)^2 = 0.11145618   (9/2)
c(√50) = 14(√50 -7)^2 = 0.070708874  (99/14)
c(√99) = (1/18)(√99 -9)^2 = 0.050125629 (10/1)
c(√200) = (7/4)(√200 -14)^2 = 0.35354437 (99/7)

c(√n) ≦ 1/(2√n),
0220132人目の素数さん
垢版 |
2019/09/16(月) 17:37:47.01ID:FF+PWEgn
>>218
ここに 1/c(α) は α-1≦x≦α+1 における |g(x,α)| の最大値。
(なお、 |(p/q) - α| >1 のときは明らか。)
0221132人目の素数さん
垢版 |
2019/09/20(金) 13:27:55.61ID:KyAOfC1j
2800
かずきち@dy_dt_dt_dx 8月28日
学コン8月号Sコース1等賞1位とれました!
マジで嬉しいです!
来月からも理系に負けず頑張りたいと思います!
https://twitter.com/dy_dt_dt_dx
https://twitter.com/5chan_nel (5ch newer account)
0223132人目の素数さん
垢版 |
2019/09/23(月) 18:13:40.99ID:2PqEJji0
 x_k >0, Σ[k=1,n] x_k = S のとき
 (S/n)^S ≦ Π[k=1,n] (x^k)^(x_k),
(略証)
 {x・log(x)}" = 1/x > 0, (下に凸)
Jensenより
 S・log(S/n) ≦ Σ[k=1,n] (x_k)log(x_k),
 等号成立は x_1=x_2=…=x_n
0224132人目の素数さん
垢版 |
2019/09/24(火) 06:33:10.11ID:CUDTSBu2
〔Problem6〕
Construct a bounded infinite sequence x_0,x_1,x_2,…… such that
 |x_i - x_j||i - j| > 1
for every pair of distinct i,j.

次の不等式をみたす有界無限実数列: x_0,x_1,x_2,… を1つ与えよ。
 i≠j ⇒ |x_i - x_j||i - j| > 1,

IMO-1991(32nd,Sweden)問題6-改.
数セミ、1991年10月号
0225132人目の素数さん
垢版 |
2019/09/24(火) 06:37:31.33ID:CUDTSBu2
>>224
 x_j = k { j √m - 1/2 }, k=1+2√m,
ここに m は平方数でない自然数。{ a } はaの小数部分
(富蘭平太氏の解)
0227132人目の素数さん
垢版 |
2019/09/26(木) 08:01:36.36ID:C1ckjksZ
〔問題〕
Prove that, for a,b,c,・・・・ > 0,
Σ[cycl] (ab)^3 /c^5 ≧ Σ[cycl] ab/c,

コーシーで
 (c+d+・・・・+a+b)^2 (左辺) ≧ (右辺)^3,
は出るけど、
 (右辺) ≧ (a+b+c+ ・・・・)
は成り立つのかな??
0228132人目の素数さん
垢版 |
2019/09/26(木) 23:50:54.84ID:C1ckjksZ
3文字のときはコーシーで簡単だが・・・・
(右辺)^2 = (ab/c+bc/a+ca/b) (ca/b+ab/c+bc/a) ≧ (a+b+c)^2,

文献[8] 安藤(2012) p.124 例題3.1.3(1) および p.144 例題3.2.2(1)

・3文字の別解
 ab/c + bc/a + ca/b ≧ √{3(aa+bb+cc)} を使う。
(右辺)^2 ≧ 3{(ca/b)(ab/c) + (ab/c)(bc/a) + (bc/a)(ca/b)} = 3(aa+bb+cc),

アイルランドMO (2007)
文献[8] 安藤(2012) p.162 例題3.3.5(2)
文献[9] 佐藤(2013) p.56 演習問題 1.113
0230132人目の素数さん
垢版 |
2019/09/27(金) 17:30:14.59ID:ncViLEfF
>>227
4文字以上では不成立かな。

a,b,c,d >0
 ab/c + bc/d + cd/a + da/b ≧ a+b+c+d,
の凡例
a,b,cを固定する。
(左辺) - (右辺) = (ab/c -a-b-c) + (bc/d) + (c/a + a/b -1)d,
c/a + a/b <1 ならば d→∞ で負になる。
a=m, b=(m+1)m, c=1, d≧(m+1)m^3 のとき負。
0231132人目の素数さん
垢版 |
2019/10/10(木) 09:05:52.04ID:4MNDsrsX
>>225
i≠j とする。
 |i-j| ≧ 1,
 | {i√m} - {j√m} | < 1,
さて、
 (i-j)√m - {i√m} + {j√m} = [ i√m ] - [ j√m ] = n,
の両辺を2乗して移行すれば
 (i-j)^2・m - nn = ({i√m}-{j√m})(2|i-j|√m -{i√m} +{j√m})
m≠平方数 ゆえ、左辺は0でない整数。
 1 ≦ |(i-j)^2・m - nn| ≦ |{i√m}-{j√m}|(1+2√m)|i-j| = |x_i-x_j||i-j|
0232132人目の素数さん
垢版 |
2019/10/14(月) 21:12:53.88ID:OfKxP42X
ここの住人は積分不等式とかは興味ないかな?

f,gを[0,1]上滑らか、かつ(f(0),g(0))=(f(1),g(1))となる関数としたとき

2π∫_0^1 {f(x)g’(x)-f’(x)g(x)}dx≦[∫_0^1 √{(f’(x))^2+(g’(x))^2} dx]^2
0233132人目の素数さん
垢版 |
2019/10/15(火) 00:45:39.69ID:eja156vF
(u, v) = (f(x), g(x)) とおく。
x∈[0,1] で (u, v) は閉曲線Cを描く。
∫[0,1] {f(x)g’(x)- f’(x)g(x)}dx = ∫_C (udv-vdu) = {Cの内部の有向面積(反時計周り→正)},
∫[0,1] √{(f’(x))^2 + (g’(x))^2}dx = ∫_C √{(du)^2 + (dv)^2} = (Cの長さ),
2π(面積) ≦ (長さ)^2
等号はCが円周 f(x)^2 + g(x)^2 = c^2 のとき。
0234132人目の素数さん
垢版 |
2019/10/15(火) 01:20:58.22ID:eja156vF
訂正
4π(面積) ≦ (長さ)^2

・参考書
数セミ増刊「数学100の問題」 日本評論社 (1984)
 「等周問題」 p.176-177
 香具師が変分法を作り、シャボン玉がコンパクト概念
を生んだ。                      (森 毅)
0235132人目の素数さん
垢版 |
2019/10/15(火) 02:06:55.09ID:O93uxOk1
>>233
>>234
正解です
いわゆる等周不等式

想定していた解法はfとgをフーリエ級数展開する方法でした
0236132人目の素数さん
垢版 |
2019/10/17(木) 05:36:05.11ID:eT2GFlgw
>>234
参考書の方法の概要

長さが一定(L=2π)で面積Sが最大の閉曲線をCとする。
もしCが凹ならば、鏡映により凸に変更すればもっと広くなる。(矛盾)
∴ Cは凸閉曲線である。
Cの二等分点をA,Bとする。
Cの内部を線分ABによって分割し、面積をS1, S2とする。
もし S1>S2 ならば S1を2つ接いだ方が広くなる。(矛盾)
∴ S1=S2
よって S1を最大にすればよい。
C上に一点Pをとる。
APより外側の部分はAPに固定し、BPより外側の部分はBPに固定し、∠APBを変える。
ΔAPBの面積だけが変わり、∠APB=90゚のとき最大になる。
(A,B以外の) C上の任意の点Pについても同様だから、
CはABを直径とする円周である。(S=π)     (終)
0238132人目の素数さん
垢版 |
2019/10/17(木) 21:33:31.34ID:MQ0StxZa
>>237
実は曲線が極座標表示可能だとしたらイェンゼン使えばすぐ分かります

曲線をr=r(θ) (0≦θ≦2π)とすれば

4π×面積=2π∫_0^(2π) r^2 dθ=4π^2(1/(2π)) ∫_0^(2π) r^2 dθ
≦ 4π^2{(1/(2π)) ∫_0^(2π) |r| dθ}^2 (∵イェンゼン)
≦ {∫_0^(2π) √(r^2+(r’)^2) dθ}^2
=周長^2

さらに等号成立はr’≡0からrは定数⇒曲線は円
ということもすぐにわかります
0239132人目の素数さん
垢版 |
2019/10/18(金) 08:22:53.35ID:nO1XpZx3
4π(面積) = 2π∫[0,2π] r^2 dθ
 = ∫[0,2π] dθ ∫[0,2π] r^2 dθ
 ≧ {∫[0,2π] r dθ}^2   (←シュワルツ)

う〜む。
0240132人目の素数さん
垢版 |
2019/10/18(金) 14:30:07.06ID:nO1XpZx3
(例) 辺の長さ L/4 の正方形

∫[0,2π] r dθ = 8∫[0, π/4] L/(8cosθ) dθ
 = ∫[0, π/4] L/cosθ dθ
 = ∫[0, π/4] L/(cosθ)^2 cosθdθ
 = ∫[0, 1/√2] L/(1-ss) ds
 = ∫[0, 1/√2] (L/2){1/(1+s) + 1/(1-s)} ds
 = [ (L/2)log{(1+s)/(1-s)} ](s:0→1/√2)
 = L log(1+√2)
 = 0.881373587 L
よって
4π×(面積) = 4π(L/4)^2
 = (π/4)L^2
 = 0.785398163 L^2
 > 0.77681940 L^2
 = (0.881373587 L)^2
 = (∫[0,2π] r dθ)^2
0243132人目の素数さん
垢版 |
2019/10/26(土) 07:51:32.79ID:S8xxgIdK
〔問題921〕
1/3 < ∫[0→∞] {sin(x)/(1+x)}^2 dx < 1/2
を示せ。

[分かスレ456脇-921]
0244132人目の素数さん
垢版 |
2019/10/26(土) 09:07:33.72ID:S8xxgIdK
I_k = ∫[kπ,(k+1)π] {sin(x)/(1+x)}^2 dx
  = ∫[0,π] {sin(t)/(1+kπ+t)}^2 dt とおく。
I = Σ[k=0,∞] I_k,

(下限)
I_k = ∫[0,π] sin(t)^2 {1/(1+kπ+t)^2 + 1/(1+(k+1)π-t)^2}/2 dt
  > ∫[0,π] sin(t)^2 dt /(1+(k+1/2)π)^2
  = 2π/(2+(2k+1)π)^2,

I_0 = 2π/(2+π)^2 = 0.2376755653426
I_1 = 2π/(2+3π)^2 = 0.0481375886732
k≧2 のとき
I_k > 2π/(2+(2k+1)π)^2
  > 2π/{(2+(2k+1)π)(2+(2k+3)π)}
  = 1/(2+(2k+1)π) - 1/(2+(2k+3)π),
Σ[k=2,∞] I_k > 1/(2+5π) = 0.056471768
これらより、I > 0.342284922 > 1/3,

(上限)
I_k = ∫[0,π] sin(t)^2 {1/(1+kπ+t)^2 + 1/(1+(k+1)π-t)^2}/2 dt
  < ∫[0,π] sin(t)^2 dt {1/(1+kπ)^2 + 1/(1+(k+1)π)^2}/2 dt
  = (π/4){1/(1+kπ)^2 + 1/(1+(k+1)π)^2}
  = π{1/(2+2kπ)^2 + 1/(2+(2k+2)π)^2},

∴ Σ[k=1,∞] I_k = π/{4(1+π)^2} + 2πΣ[k=2,∞] 1/(2+2kπ)^2
  < π/{4(1+π)^2} + 2πΣ[k=2,∞] 1/{(2+(2k-1)π)(2+(2k+1)π)}
  = π/{4(1+π)^2} + Σ[k=2,∞] {1/(2+(2k-1)π) - 1/(2+(2k+1)π)}
  = π/{4(1+π)^2} + 1/(2+3π)
  = 0.04578836 + 0.087529053
  = 0.133317413
  < 2/15,

0<x<π では sin(x) < (4/ππ)x(π-x),
I_0 < (2/π)^4 ∫[0,π] {x(π-x)/(1+x)}^2 dx
  = (2/π)^4 ・ 1.8581544248371
  = 0.30521248563
  < 1/3,
∴ I < 7/15

なお、実際の値は
 I_0 = 0.28136039736534
 I_1 = 0.0496240021299
 I = 0.3990209885942
0245132人目の素数さん
垢版 |
2019/10/26(土) 09:18:45.67ID:S8xxgIdK
>>244
I_0 > 2π/(2+π)^2 = 0.2376755653426
I_1 > 2π/(2+3π)^2 = 0.0481375886732

∫{x(π-x)/(1+x)}^2 dx = (1/3)(1+x)^3 - (2+π)(1+x)^2 + (6+6π+π^2)(1+x) - (1+π)^2/(1+x) - 2(2+3π+π^2)log(1+x),
0246132人目の素数さん
垢版 |
2019/10/28(月) 13:09:51.57ID:M55VqgNP
>>244

|y| ≦ π/2 のとき
 ((√2)/π) |y| ≦ |sin(y/2)| ≦ |y/2|,  ・・・・ Jordanの不等式
 1 - (1/2)yy ≦ cos(y) = 1- 2sin(y/2)^2 ≦ 1 - (2y/π)^2,

π/2 ずらすと
0≦x≦π のとき  sin(x) ≦ (4/π^2)・x(π-x),
0247132人目の素数さん
垢版 |
2019/11/11(月) 01:05:02.45ID:RnIwgTT0
a,b,c>0 に対して、
1/a + 1/b + 1/c ≦ (a^8 + b^8 + c^8)/(a^3 b^3 c^3).

IMO 1967 らしい
0248132人目の素数さん
垢版 |
2019/11/11(月) 23:31:55.51ID:uIUz6082
 a^8 (2個), b^8 (3個), c^8 (3個) の8個で AM-GM する。
 (2a^8 + 3b^8 + 3c^8) /8 ≧ a^2・b^3・c^3 = (abc)^3 /a,
循環的にたす。
0249132人目の素数さん
垢版 |
2019/11/12(火) 02:21:11.77ID:dGHte+xL
問題
x>-2,y>0として
ye^x>log(yx+2y)
0251132人目の素数さん
垢版 |
2019/11/12(火) 06:15:32.35ID:XcoIZ5AW
>>249
e^t >= 1+t, 1+t >= log(t+2)より
ye^x = e^{(log y) + x} >= log y + (x+1)
>= log y + log(x+2) = log(yx+2y)
二つの等号を同時に成立させるx, yはない。
0252132人目の素数さん
垢版 |
2019/11/13(水) 01:43:38.75ID:HL1mwdTs
>>249
題意より y(x+2) > 0,
(1/e)t ≧ log(t) より、
y e^x = (1/e)y (1/e)e^(x+2) ≧ (1/e)y(x+2) ≧ log{y(x+2)},
等号成立は x=-1, y=e のとき。
0253132人目の素数さん
垢版 |
2019/11/15(金) 10:38:04.46ID:co/VrloJ
Σ[n=1->∞](1/{(n+1)(n!)^2})^(1/n) < e
(オリジナル)

飛び道具を使わずに示したいんだけど、どうもうまくいかない。
0254132人目の素数さん
垢版 |
2019/11/15(金) 13:32:49.92ID:+e/x8O7I
というか、上の級数は1.93あたりに収束するっぽいんだけど、
明示的に極限を書けないかしら。
0255132人目の素数さん
垢版 |
2019/11/16(土) 04:10:15.50ID:iMDULalJ
>>253
たとえば
 (1+1) < (1+1/2)^2 < (1+1/3)^3 < ・・・・・ < (1+1/n)^n < e,
すなわち
 2 < (3/2)^2 < (4/3)^3 < ・・・・ < {(n+1)/n}^n < e,
最右辺を除くn項を掛け合わせて
 (n+1)^n /n! < e^n,
 n! > {(n+1)/e}^n,
 1/n! < {e/(n+1)}^n,
と評価する。

Σ[n=3,∞) (1/{(n+1)!(n!)})^(1/n) < Σ[n=3,∞] {e/(n+2)}{e/(n+1)}
 = Σ[n=3,∞] ee{1/(n+1) - 1/(n+2)} = ee/4,

(左辺) < 1/2 + 1/√12 + ee/4 = 2.63593916 < e,
0256132人目の素数さん
垢版 |
2019/11/16(土) 05:33:37.76ID:cdgu8qg6
>>255
おお…ありがとう
ちなみに飛び道具とはcarlemanの不等式でした。
その証明にも(n+1)^n /n! < e^nが使われるという
0258132人目の素数さん
垢版 |
2019/11/18(月) 15:47:33.56ID:W9Q6monY
〔補題〕
 (1) (1 +1/n)^(n+1/2) は単調減少でeに収束
 (2) n! < n^(n+1/2) / e^(n-1),
 (3) (2n)! / n! < (√2)(4n/e)^n,

(1)は一般化二項公式を使うらしい。
0260132人目の素数さん
垢版 |
2019/11/25(月) 17:59:50.61ID:earBUlUp
 A = a^4/(a^4+b^4+c^4),
 B = b^4/(a^4+b^4+c^4),
 C = c^4/(a^4+b^4+c^4),
とおくと
 A+B+C = 1.
また
 f(x) = √x - [x(1-x)/2]^(1/4),
とおくと
 f '(x) = 1/(2√x) - ((1-2x)/8)[x(1-x)/2]^(-3/4),
 f "(x) = -1/(4x√x) - (1/8)[x(1-x)/2]^(-3/4) + (3/64)[x(1-x)/2]^(-7/4) > 0,
∴ y=f(x) は下に凸。
∴ Jensen より
 (LHS - RHS) 〜 f(A) + f(B) + f(C)
  ≧ 3f((A+B+C)/3)
  = 3f(1/3)
  = 0,

微分のことは微分でせよ?
0261132人目の素数さん
垢版 |
2019/11/26(火) 05:01:43.80ID:/rJkC9KX
>>236-237
・参考文献
 J. Steiner: J. reine Angew. Math., 24, p.93-152 (1842)
 浦川 肇:「等周不等式」 数理科学(サイエンス社) No.386, p.20-24 (1995/Aug)
0264132人目の素数さん
垢版 |
2019/11/26(火) 12:13:55.79ID:GwDHuLDD
問題

1≦pとする
ある定数C=C(p)>0が存在し、
(0,1)上の任意のC^1級関数fに対して
∫_0^1 |f(x)-∫_0^1 f(y) dy|^p dx≦C∫_0^1 |f’(x)|^p dx

またこの不等式が成立する最小のC(p)を求めよ
0267132人目の素数さん
垢版 |
2019/11/28(木) 22:48:14.44ID:ghZZAPQ9
>>263 を改良

S_n = Σ[k=1→n] 1/{(k+1)(k!)^2}^(1/k)
  ≒ 1.99877613 - (ee/(n+2)){1 - (1/n)log(n)}

S_1 = 0.50
S_2 = 0.78867513459481
S_4 = 1.11596688482249
S_8 = 1.41825957672665
S_16 = 1.6498309820817
S_32 = 1.80276021419195
S_64 = 1.8936289850894
S_128 = 1.9439982730789
S_256 = 1.9707380873724
S_512 = 1.9845718842414
S_1024 = 1.99162226380515
S_2048 = 1.9951849538552
S_4096 = 1.9969766630793
S_8192 = 1.9978753488909
S = 1.99877613
0270132人目の素数さん
垢版 |
2019/11/30(土) 06:31:05.40ID:EU1tlCDO
>>268
〔定理〕
x_1 >0, x_2 >0, ・・・・・・, x_n >0 のとき
 (1/n)Σ[k=1,n] (x_1・x_2…x_k)^(1/k) ≦ {Π[k=1,n] (x_1+x_2+…+x_k)/k}^(1/n),
 等号成立は x_1 = x_2 = ・・・・ = x_n.

[前スレ.037(1)〜044, 051-053, 097] の辺り
K.Kedlaya: Amer.Math.Monthly, Vol.101, No.4, p.355-357 (1994/Apr)
 "Proof of a mixed Arithmetic-mean, Geometric-mean inequality"
0271132人目の素数さん
垢版 |
2019/12/01(日) 04:00:04.45ID:oC7hjXGF
右辺を変形してコーシーに持ち込みたいが・・・・

 G1 = a, G2 = √(ab), G3 =(abc)^(1/3), ・・・・ とおく。
n=2
 (a+a)(a+b) ≧ (G1+G2)^2,
n=3
 (a+a+a)(a+G2+b)(a+b+c) ≧ (G1+G2+G3)^3,
n=4
 (a+a+a+a)(a+B1+B2+b)(a+B2+C+c)(a+b+c+d) ≧ (G1+G2+G3+G4)^4,
  B_1 = (aab)^(1/3)
  B_2 = (abb)^(1/3),
  B_k = {a^(n-1-k)・b^k}^(1/(n-1)) 
  C = (bbc)^(1/3),
n=5
 (a+a+a+a+a)(a+B1+G2+B3+b)(a+G2+C'+C"+c)(a+B3+C"+D'+d)(a+b+c+d+e)≧(G1+G2+G3+G4+G5)^5,
  B_1 = (aaab)^(1/4),
  B_2 = √(ab) = G2,
  B_3 = (abbb)^(1/4),
  C ' = (ab^4・c)^(1/6),
  C " = √(bc),
  D ' = (cccd)^(1/4),
0272132人目の素数さん
垢版 |
2019/12/01(日) 16:15:48.98ID:oC7hjXGF
まづ AM-GM で右辺を
 (x1 + x2 + ・・・・ + xk)/k ≧ Σ[j=1,n] Π[i=1,k] (x_i)^e(i;j,k)
として、コーシーを使うのでござる。
ここに
e(i;j,k) = (n-j)! (j-1)! (n-k)! (k-1)! /{(n-1)! (i-1)! (j-i)! (k-i)! (n+i-j-k)!}
     (i≦j≦n, i≦k≦n & j+k-i≦n)
    = 0  (otherwise)
チト面倒でござるが・・・・
0273132人目の素数さん
垢版 |
2019/12/01(日) 20:39:28.33ID:oC7hjXGF
 (x_1+x_2+・・・・+x_k)/k ≧ (1/n)Σ[j=1,n] Π[i=1,k] (x_i)^e(i;j,k)
でござった。

対称性
 e(i;j,k) = e(i;k,j)
斉次性
 Σ[i=1,k] e(i;j,k) = 1,
総和則
 Σ[j=i,n] e(i;j,k) = n/k  (1≦i≦k)
           = 0,   (k<i≦n)
 Σ[k=i,n] e(i;j,k) = n/j  (1≦i≦j)
           = 0,   (j<i≦n)
から出るか。
0274132人目の素数さん
垢版 |
2019/12/02(月) 19:46:04.02ID:a5zqFxLP
>>270
(略証)
AM-GM より
 Σ[i=1,k] e(i;j,k) x_i
 ≧ {Σ[i=1,k] e^(i;j,k)}・{Π[i=1,k] (x_i)^e(i;j,k)}
 = Π[i=1,k] (x_i)^e(i;j,k)     (←i斉次性)
これを Σ[j=1,n] で加えると
 (x_1+x_2+・・・・ +x_k)/k
 ≧ Σ[j=1,n] Π[i=1,k] (x_i)^e(i;j,k) (←j総和則)
次に Π[k=1,n] で掛け合わせ、コーシーを使う。
n乗の中の第j項は
 Π[i=1,n] (x_i)^{(1/n)Σ[k=1,n] e(i;j,k)}
 = Π[i=1,j] (x_i)^(n/j)     (←k総和則)
 = G_j,   (終)
0275132人目の素数さん
垢版 |
2019/12/03(火) 08:05:57.48ID:tkRoCzDX
最後は
 Π[i=1,n] (x_i)^{(1/n)Σ[k=i,n] e(i;j,k)}
 = Π[i=1,j] (x_i)^(1/j)     (←k総和則)
 = G_j,   (終)
0276132人目の素数さん
垢版 |
2019/12/03(火) 09:41:52.21ID:tkRoCzDX
e(i;j,k) = C[j-1,i-1] C[k-1,i-1] (n-j)! (n-k)! / {C[n-1,i-1] (n-i)! (n+i-j-k)!},

のように組合せを使って表わすのが 貞三流でござるか? (山梨大)
0277132人目の素数さん
垢版 |
2019/12/05(木) 01:06:46.33ID:+ttUgWiw
>>269

〔例題4'〕
 x_1≧1, x_2≧1, ・・・・, x_n ≧1 のとき
 A - G ≧ G/H - 1,
ここに
 A = (x_1+x_2+・・・・+x_n)/n,
 G = (x1・x2・・・・x_n)^(1/n),
 H = n/(1/x_1+1/x_2+ ・・・ +1/x_n),

(略証)
G = (x_1・x_2・・・・x_n)^(1/n) を固定して考える。
F(x) = (左辺) - (右辺)
   = (x_1+x_2+・・・・+x_n)/n - G − (1/n)(1/x_1+1/x_2+・・・・+1/x_n)G + 1,
とおく。
(x_1, x_2, ・・・・, x_n) = (G, G, ・・・・, G) ならば F(x) = 0.
そうでないとき
 x_i > G > x_j ≧ 1,
となる i≠j がある。 (x_i・x_j > G・1)
それらを
 x_i’= G, x_j’=x_i・x_j/G,
に変更すると
 x_i’+ x_j’- (x_i + x_j) = −(x_i -G)(G -x_j)/(n・G),
 1/x_i’+ 1/x_j’- (1/x_i + 1/x_j) = (x_i -G)(G -x_j)/(n・x_i・x_j),
よって
 F(x’) − F(x) = −{(x_i -G)(G -x_j)/(n・G)} {1 - G/(x_i・x_j)} < 0,
すなわち F(x) は減少する。

上記の操作を行なうたびにGの個数が1つ増えるから、
n回以内に (G,G,・・・・,G) となり、F=0 に至る。 (終)
0278132人目の素数さん
垢版 |
2019/12/05(木) 02:40:54.44ID:+ttUgWiw
〔系〕
 x_1≦1, x_2≦1, ・・・・, x_n≦1 のとき
 A - G ≦ G/H - 1,

(略証)
x_i = 1/x’_i とおくと、
 A = 1/H’ G = 1/G’ H = 1/A’
0279132人目の素数さん
垢版 |
2019/12/09(月) 04:36:54.15ID:3RsZZfph
>>270
 nについての帰納法による。
 A_k = (x_1+x_2+・・・・+x_k)/k,
 G_k = (x_1・x_2・・・・x_k)^(1/k),
とおく。

n=1 は明らか。
あるnについて成立つとする。
 (A_1・A_2・・・・A_n)^(1/n) ≧ (G_1+G_2+・・・・+G_n)/n
  = (g_1+g_2+・・・・・+g_{n+1})/(n+1),
ここに
g_k = [(k-1)G_{k-1} + (n+1-k)G_k]/n
  ≧ [G_{k-1}^(k-1)・(G_k)^(n+1-k)]^(1/n)  (AM-GM)
  = {(G_k)^(n+1) / x_k}^(1/n),
とおいた。また
 A_{n+1} = (x_1+x_2+・・・・+x_{n+1})/(n+1),

ここでコーシーを使う。
 (g_k)^n・x_k ≧ (G_k)^(n+1),
より
 (A_1・A_2・・・・・A_{n+1})^{1/(n+1)} ≧ (G_1+G_2+・・・・+G_{n+1})/(n+1)
n+1 についても成り立つ。
0280132人目の素数さん
垢版 |
2019/12/15(日) 01:02:39.88ID:Nfo6ujPm
0<x<2n で f "(x) > 0 のとき、
 f(x) は下に凸で
 nΣ[k=0,n] f(2k) > (n+1)Σ[k=1,n] f(2k-1)

[分かスレ456.720-722]
0281132人目の素数さん
垢版 |
2019/12/17(火) 14:19:03.72ID:/04vhOiY
凸不等式より
 {f(2k-2) + f(2k)}/2 > f(2k-1)   ・・・・ (1)
また 0<k<n に対して
 {(n-k)f(0) + k・f(2n)} /n > f(2k),
 {k・f(0) + (n-k)f(2n)} /n > f(2n-2k),
辺々たすと
 f(0) + f(2n) > f(2k) + f(2n-2k)   ・・・・ (2)
これらにより
 (左辺) - (右辺)
 > nΣ[k=0,n] f(2k) - (n+1){f(0)+f(2n)}/2 - (n+1)Σ[k=1,n-1] f(2k)
 = (n-1){f(0) + f(2n)}/2 - Σ[k=1,n-1] f(2k)
 = (1/2)Σ[k=1,n-1] {f(0) + f(2n) - f(2k) - f(2n-2k)}
 > 0,
0282132人目の素数さん
垢版 |
2019/12/17(火) 14:43:09.39ID:/04vhOiY
〔問題132〕
 A = (x_1+x_2+・・・・・+x_n)/n,
 G = (x_1・x_2・・・・・x_n)^(1/n),
 L = (A_1・A_2・・・・・A_n)^(1/n)  ただし A_k = (x_1+x_2+・・・・・+x_k)/k,
とおく。
 (n+1)(G/A)^{1/(n+1)} ≦ n(L/A)^(1/n) + G/L ≦ n+1,

 IMO-2004 short list A.7
 Inequalitybot [132]  ☆12
0283132人目の素数さん
垢版 |
2019/12/17(火) 14:58:04.88ID:/04vhOiY
(左) GM-AM で
(右)も GM-AM で
 (L/A)^(1/n)
 = {1^((n+1)/2n)} Π[k=2,n] {A_(k-1)/A_k}^((k-1)/nn)
 ≦ (n+1)/2n + (1/nn)Σ[k=1,n] (k-1)・A_(k-1)/A_k,
また
 G/L = (Π[k=1,n] x_k/A_k)^(1/n) ≦ (1/n)Σ[k=1,n] x_k/A_k,
したがって
 (中辺) ≦ (n+1)/2 + (1/n)Σ[k=1,n] {(k-1)A_(k-1) + x_k}/A_k
  = (n+1)/2 + (1/n)Σ[k=1,n] k
  = (n+1)/2 + (n+1)/2
  = n+1,
0285132人目の素数さん
垢版 |
2019/12/19(木) 13:28:01.97ID:8YaA1ba7
ab+bc+ca = t とする。

1/aa + 1/bb + 1/cc ≧ 1/ab + 1/bc + 1/ca
 = (a+b+c)/(abc)
 ≧ √(3t) /(abc),

∵ xx+yy+zz ≧ (1/3)(x+y+z)^2 ≧ xy+yz+zx,
0290132人目の素数さん
垢版 |
2019/12/29(日) 03:42:26.63ID:ktrDgrgt
問 題
 任意の相異なる正の数 a,b に対し、不等式
  √(ab) < (a-b)/{log(a)-log(b)} < (a+b)/2 ・・・… (*)
が成立することが知られている。
 この不等式を相異なる3つの正の数 a,b,c に関する不等式に拡張したものを一つ見つけて、それを証明せよ。ただし、ここでの拡張した不等式とは
 (abc)^(1/3) < F(a,b,c) < (a+b+c)/3
  ( F(a,b,c) は log(a), log(b), log(c) を含む a,b,c の対称式 )
であるとする。
 なお、F(a,b,c)について上記以外の仮定は定めないが、できる限り(*)から自明に得られるものでない方が望ましいものとする。
(ただし、(*)からの変形が不可というわけではない。今回はかなり自由に考えてほしい。)

 東進 数学コンクール
0293132人目の素数さん
垢版 |
2019/12/31(火) 02:29:47.93ID:CIMjjWYH
>>292
f(x) を [0,1] 上で非負の単調減少関数とするとき、
∫[0,1] x f(x)^2 dx /∫[0,1] x f(x) dx ≦ ∫[0,1] f(x)^2 dx /∫[0,1] f(x) dx,
を示せ。

(略証)
題意より
 0 ≦ f(x),
 0 ≦ f(y),
 0 ≧ (x-y) {f(x)-f(y)},
よって
 0 ≧ ∬(x-y){f(x)-f(y)}f(x)f(y) dxdy
  = ∬{xf(x)^2 f(y) + yf(y)^2 f(x) - xf(x)f(y)^2 - yf(y)f(x)^2} dxdy
  = 2∫xf(x)^2 dx・∫f(y) dy - 2∫f(x)^2 dx・∫yf(y) dy

∴ ∫xf(x)^2 dx /∫yf(y)dy ≦ ∫f(x)^2 dx /∫f(y)dy,
0294132人目の素数さん
垢版 |
2020/01/01(水) 03:51:45.32ID:F81QwpXb
      ∧_∧
     ( ´Д` )  新年あけまして
     /     ヽ
     し、__X__,ノJ

      /´⌒⌒ヽ
    l⌒    ⌒l  おめでとうございます。
   ⊂ (   ) ⊃
      V ̄V

正の数 a,b,c に対して
(a^2020 -a^2 +2^2)(b^2020 -b^2 +2^2)(c^2020 -c^2 +2^2) > (a^2+b^2+c^2)^3,
>>22
>>26
0295132人目の素数さん
垢版 |
2020/01/01(水) 13:16:53.96ID:vZvCSbvi
exradii って傍接円の半径であってるかな?
共立の赤い辞書にも載ってないし、web検索でもhitしないけど…
0299132人目の素数さん
垢版 |
2020/01/04(土) 21:55:59.92ID:2AvOoxci
>>183
Reverse triangle inequalityと聞くと違う不等式を想像する
何か別の名前はついてないのかな
0301132人目の素数さん
垢版 |
2020/01/05(日) 16:43:59.29ID:M9rUjvu0
正の数a, b, cに対して
(a^1010-a+4)(b^1010-b+4)(c^1010-c+4)>(a+b+c)^3
が成り立つことを示す.
Σa/3=Mとおく.

M≧1のとき
(LHS)
≧Π(a^1009+3)
>9Σa^1009
≧27M^1009
≧27M^3
=(RHS)

M<1のとき
(LHS)
≧Π(a^1009+3)
>3^3
>27M^3
=(RHS)
0302132人目の素数さん
垢版 |
2020/01/05(日) 17:02:53.16ID:M9rUjvu0
>>181
Kantorovich
>>227 >>247
Muirhead
>>280
Karamata
細かいけど(0,2n)ではなく[0,2n]でf">0だと思う
>>300
ttps://en.wikipedia.org/wiki/Triangle_inequality
ttps://proofwiki.org/wiki/Reverse_Triangle_Inequality
0303132人目の素数さん
垢版 |
2020/01/05(日) 18:28:42.28ID:PT3ra9AO
>>298
サノバビッチの不等式、糞ビッチの不等式、ぬるぽビッチの不等式…、いろいろあるなぁ… (錯乱)
0304132人目の素数さん
垢版 |
2020/01/06(月) 03:47:32.05ID:iay27LR5
>>301
正解です!!

 (y^1010 +1) - (y^1009 + y)
 = (y^1009 -1)(y-1)
 ≧ 0,
を使ったでござるか。さらに
 y^1009 + 3 ≧ y^1006 + y^3 +2 ≧ y^3 +2,
とすれば、コーシーで
 (a^3+1+1)(1+b^3+1)(1+1+c^3) ≧ (a+b+c)^3,

なお、最良係数は
 y^1010 -y +4 ≧ 1.008619375112(y^3 +2),
 等号は y = 0.994531163783 のとき

>>295
 傍接円(excircle) の半径(radii) ですね。
0305132人目の素数さん
垢版 |
2020/01/06(月) 19:06:36.22ID:iay27LR5
>>301
(a+b+c)/3 = M とおく。

(a^1009 +3)(b^1009 +3)(c^1009 +3)
 > 27{(a^1009 + b^1009 + c^1009)/3 + 1}
 ≧ 27(M^1009 + 1)
 > 27max{M^1009, 1}
 ≧ 27(max{M, 1})^3
 = max{3M, 3}^3
 = max{a+b+c, 3}^3.
0308132人目の素数さん
垢版 |
2020/01/17(金) 00:16:00.50ID:1crWIv5/
>>307
Sは三角形の面積だと勝手に解釈しました. 出典知りたいです.

a≧c≧bの場合を証明すればよい.
まず補題, a^2+b^2+c^2-4S≧(a+c-b)^2 を示す.
これは -1≧1/sinB-1/sinA-1/sinC と同値.
Bを固定する. f(C)=1/sinB-1/sin(π-B-C)-1/sinC とする.
g(X)=-cosX/(sinX)^2 とする. (定義域は(0,π))
g'>0 だからgは単調増加なので f'(C)=-g(π-B-C)+g(C)≦0 だからfは単調減少.
よって f(C)≦f(B)=-1/sin(π-2B)≦-1 なので補題は示された.
題意の不等式は a^2+b^2+c^2-4S=1 の場合を証明すればよい.
(0,1]を定義域とする h(x)=-sqrt(1-x^2) を考える. hは凸関数.
(a+c-b,b) は (a,c) をマジョライズするからKaramataの不等式より
h(a+c-b)+h(b)≧h(a)+h(c) なので h(b)≧h(a)+h(c)
よって sqrt(1-aa)+sqrt(1-cc)≧sqrt(1-bb)
題意の不等式は示された.
0309132人目の素数さん
垢版 |
2020/01/17(金) 16:03:09.19ID:8XEo1F0J
>>291
 k ≧ (1+√5)/2 = φ = 1.618034・・・ より
 kk-k-1 = (k-φ)(k+φ-1) ≧ 0,
より
 (左辺) - (右辺) = (kk-k-1){(a-1)^2 + (b-1)^2 + (c-1)^2}
 + k{(ab-1)^2 + (bc-1)^2 + (ca-1)^2} + (abc-1)^2
 + 2(a-1)(b-1)(c-1),
う〜む、場合分けでござるか・・・・


k=2 の場合は
 a,b,c>0 のとき (aa+2)(bb+2)(cc+2) ≧ 3(a+b+c)^2.

 APMO-2004 A.5,
 文献[9] 佐藤(訳) 問題3.85, 朝倉 (2013)
 Inequalitybot [20]
 [前スレ. 070[2]、084]  [第8章.456、469]
0310132人目の素数さん
垢版 |
2020/01/20(月) 13:00:34.19ID:RONGOZUM
>>309
k ≧ φ ならば, どの文字に注目しても
f(a,b,c,k) = (aa+k)(bb+k)(cc+k) - (k+1)(a+b+c+k-2)^2
は下に凸の二次関数, さらに(1,1,1,φ)で極小値0.

>>291 のステートメントが適当すぎる.
0311132人目の素数さん
垢版 |
2020/01/21(火) 00:47:02.56ID:NlSt5Qji
〔問題〕
実数a,b,cが
 a < b < c, a+b+c = 6, ab+bc+ca = 9
を満たしている。
 0 < a < 1 < b < 3 < c < 4
を証明せよ。
 高校数学問題bot (@7k_x)

- - - - - - - - - - - - - - -
3a(4-a) = (c-b)^2 >0,
3c(4-c) = (b-a)^2 >0,
(c-a)^2 -9 = (c-b)(b-a) >0,
3(3-b)(b-1) = (c-b)(b-a) >0,
らしい。
0314132人目の素数さん
垢版 |
2020/02/01(土) 04:59:30.88ID:7zqqjjoe
>>312
第7問
実数kは、任意の2以上の整数nと正の実数 a_0,a_1,・・・・,a_n に対して
 1/(a_0+a_1) + 1/(a_0+a_1+a_2) + ・・・・ + 1/(a_0+a_1+・・・・+a_n) < k (1/a_0 + 1/a_1 + ・・・・ +1/a_n)
を満たす。このようなkとしてあり得る最小の値を求めよ。

>>313
第8問
正の整数からなる数列a1,a2,・・・・があり、任意の正の整数nについて
 a_n > (a_{n+1} + a_{n+2} + ・・・・ + a_{2n}) / (n+2016)
を満たしている。このとき、ある正の実数Cが存在し、
任意の正の整数nについて a_n < C が成り立つことを示せ。

第10問
3以上の正の整数nであり、次を満たすものをすべて求めよ。
|a_k| + |b_k| = 1 (k=1,2,・・・・,n) を満たすような任意の2n個の実数 a_1,a_2,・・・・,a_n, b_1,b_2,・・・・,b_n に対して、
実数 x_1,x2,・・・・,x_n を、次を満たすように選ぶことができる。
 |x_k| = 1 (k=1,2,・・・・,n),
 |Σ[k=1,n] x_k・a_k | + |Σ[k=1,n] x_k・b_k | ≦ 1.
0315132人目の素数さん
垢版 |
2020/02/01(土) 07:31:35.29ID:7zqqjjoe
第7問
 k = 1/3
 (a_0 = a_1 = 1, a_k = 2^(k-1), n→∞ のとき)

第10問
 v_k = (a_k,b_k) は正方形 |X| + |Y| = 1 の辺上にある。
 Σ[k=1,n] (±v_k) が正方形の内側に落ちる (ように各符号x_kを選ぶ)
0317132人目の素数さん
垢版 |
2020/02/05(水) 07:10:07.69ID:AQM1KB8L
>>312 >>315
第7問
 S_j = a_0 + a_1 + ・・・・ + a_j,
とおくと HM-AM で
 4/S_j = 4/{S_(j-1) + a_j} ≦ 1/S_(j-1) + 1/a_j,
 4/S_j - 1/S_(j-1) ≦ 1/a_j,
j=1〜n の和をとる。
 3Σ[j=1,n] 1/S_j + 1/S_n ≦ Σ[j=0,n] 1/a_j,
 Σ[j=1,n] 1/S_j < (1/3)Σ[j=0,n] 1/a_j,
0321132人目の素数さん
垢版 |
2020/02/12(水) 07:39:21.58ID:uWBQqkSN
 2020年 日本数学オリンピック 本選
      (C)(公財) 数学オリンピック財団

      問   題^1

2020年2月11日 試験時間4時間 5題

1. (n^2 +1)/(2m) と √{2^(n-1) + m + 4} がともに整数となるような正の整数の組 (m,n) をすべて求めよ。

 (m,n) = (1,3) (61,11) ?

2. BC < AB, BC < AC なる三角形ABCの辺AB,AC上にそれぞれ点D,Eがあり、BD=CE=BC を満たしている。
 直線BEと直線CDの交点をPとする。
 三角形ABEの外接円と三角形ACDの外接円の交点のうちAでない方をQとしたとき、直線PQと直線BCは垂直に交わることを示せ。
 ただし、XYで線分XYの長さを表わすものとする。

3. 正の整数に対して定義され正の整数値をとる関数fであって、任意の正の整数m,nに対して
   m^2 + f(n)^2 + (m-f(n))^2 ≧ f(m)^2 + n^2,
 を満たすものをすべて求めよ。
0322132人目の素数さん
垢版 |
2020/02/12(水) 08:06:10.32ID:uWBQqkSN
4. nを2以上の整数とする。
 円周上に相異なる3n個の点があり、これらを特別な点とよぶことにする。
 A君とB君が以下の操作をn回行なう。

  まず、A君が線分で直接結ばれていない2つの特別な点を結んで線分で結ぶ。
  次に、B君が駒の置かれていない特別な点を1つ選んで駒を置く。

 A君はB君の駒の置き方にかかわらず、n回の操作が終わったときに駒の置かれている特別な点と駒の置かれていない特別な点を結ぶ線分の数を (n-1)/6 以上にできることを示せ。

5. ある正の実数cに対して以下が成立するような、正の整数からなる数列 a_1, a_2, ・・・・ をすべて求めよ。

  任意の正の整数m,nに対して gcd(a_m + n, a_n + m) > c (m+n) となる。

 ただし、正の整数x,yに対し、xとyの最大公約数を gcd(x,y) で表わす。
                                以 上
0323132人目の素数さん
垢版 |
2020/02/13(木) 08:18:39.16ID:8bKSb4oB
〔問題〕
f(x) = Σ[k=0,n] c_k x^k (c_k ≧0) のとき、次は成り立つか?
(1) {u f '(x)/f(x)} ' ≧ 0,
(2) log{f(e^x)} は下に凸
(3) f(x)f(y) ≧ f(√(xy))^2.

分かスレ458.054-063
0324132人目の素数さん
垢版 |
2020/02/16(日) 01:35:24.83ID:N9QZtxQk
>>311
 f(x) = (x-a)(x-b)(x-c) = x^3 -6x^2 +9x -abc
とおく。
 (4-abc) - f(x) = (4-x)(1-x)^2,
から x=1 で極大 f(1) = 4-abc = f(4),
 f(x) + abc =x(x-3)^2,
より x=3 で極小 f(3) = -abc = f(0),
または 微分して増減表を書けばx=1で極大値 4-abc,x=3で極小値 -abc をとる。
題意により、f(x)=0 は 3実根a<b<cをもつから
 -abc < 0 < 4-abc
∴ 0<a<1<b<3<c<4
(tenさん)

http://suseum.jp/gq/question/3132
0325132人目の素数さん
垢版 |
2020/02/19(水) 13:19:05.46ID:WOZU/4dA
n>1 に対して
 ζ(n) = 1 + 1/(2^n) + 1/(3^n) + ・・・・
とおく。このとき
 ζ(n) > e^{1/(2^n)} > 1 + 1/(2^n)
か?
0326132人目の素数さん
垢版 |
2020/02/20(木) 08:57:27.26ID:ZWVgPXIY
ζ(n) = Σ[k=1,∞] 1/(k^n)
 > Σ[j=0,∞] 1/(2^j)^n
 = Σ[j=0,∞] 1/N^j
 = 1/(1 - 1/N),
ここに、N=2^n とおいた。
1/N^j > 1/(j!・N^j) より
ζ(n) > 1/(1 - 1/N) > exp(1/N) > 1 + 1/N,
0328132人目の素数さん
垢版 |
2020/02/27(木) 03:16:30.37ID:6SmBw6gg
>>327
   第 1 問

a,b,c,p を実数とする。不等式
  ax^2 + bx + c > 0
  bx^2 + cx + a > 0
  cx^2 + ax + b > 0
をすべて満たす実数xの集合と、x>p を満たす実数xの集合が一致しているとする。

(1) a,b,c はすべて0以上であることを示せ。

(2) a,b,c のうち少なくとも1個は0であることを示せ。

(3) p=0 であることを示せ。
0329132人目の素数さん
垢版 |
2020/02/27(木) 04:06:30.88ID:6SmBw6gg
>>328
(1) 背理法による。
 a<0 と仮定すると f(x) = ax^2 +bx +c は上に凸な放物線。
 f(x)>0 を満たすxの範囲は (もし有っても) 有限の範囲内。
 これは題意の「x>p ⇒ 3式すべてを満たす」に反する。
 b,cについても同様。

(2) 背理法による。
 a>0 と仮定すると f(x) = ax^2+bx+c は下に凸な放物線。
 |x| > 2|b/a| + √|2c/a| ⇒ f(x) >0 を満たす。
 -x がじゅうぶん大きいときにも f(x) >0 を満たす。
 a,b,c>0 と仮定すると、
 -x がじゅうぶん大きいときにも 3式すべてを満たす。
 これは題意の「3式すべてを満たす ⇒ x>p」に反する。
 ∴ abc=0.
0330132人目の素数さん
垢版 |
2020/02/28(金) 16:07:45.47ID:b8YXVJTs
>>328
(3)
a>0, b≧0, c=0 のとき
 (ax+b)x >0 ・・・・ x>0 または ax+b<0
 bxx+a > 0 ・・・・ すべての実数
 ax+b > 0 ・・・・ (x>0を含む)
 このすべてを満たす実数xは x>0.

a=b=c=0 のとき
 3式を満たす実数xはない。(不適)

以上により p=0.
0331132人目の素数さん
垢版 |
2020/02/28(金) 16:57:20.02ID:b8YXVJTs
・分野・テーマ
 2次関数、集合と命題、極限

・設問内容
係数に対称性のある3つの2次以下の不等式をすべて満たす実数の集合の形から、不等式の係数についての条件や、不等式の解を決定する問題である。

・解答のポイント
結論は直感的には明らかであるが、それをきちんと証明するのは難しい。
(1) グラフの概形をイメージすることが大きな手掛かりとなる。
  十分大きなxについて考えると良い。
(2) (1)と同様に極限を考えると良いが、ここが難所である。
(3) 前問が示せていなくても、取り組みたい。
  係数すべてが0となることはなく、1つは0であるから、この条件の下で3つの不等式を解くことで解決する。

http://www.yomiuri.co.jp/nyushi/sokuho/k_mondaitokaitou/tokyo/1313355_5408.html
http://hayabusa9.5ch.net/test/read.cgi/news/1582861742/
0333132人目の素数さん
垢版 |
2020/03/09(月) 23:24:26.98ID:V6IMEB5h
〔問題332〕
0以上1以下の実数p,q,r,sにおいて、
 √(p^2 + r^2) + √{p^2 + (r-1)^2} + √{(1-q)^2 + s^2} + √{(1-q)^2+(1-s)^2} + √{(p-q)^2 + (r-s)^2}
の最小値を求めよ。

-------------------------------------------------------------

O(0,0) A(0,1) B(1,1) C(1,0) P(p,r) Q(q,s)
とおくと与式は
 OP + AP + CQ + BQ + PQ

P は △OCQ のフェルマー点
Q は △ABP のフェルマー点
0334132人目の素数さん
垢版 |
2020/03/10(火) 09:32:03.67ID:Ct1vj+NA
∠AQB = 120゚ より 点Qは円周
 {1+1/(2√3) -x}^2 + (y - 1/2)^2 = 1/3,
上にある。
∴ ∠AQB の二等分線は (1+1/√3, 1/2) を通る。

∴ P,Q は y=1/2 上に有る。r=s=1/2,
∴ P(p,r) = (1/(2√3), 1/2)  Q(q,s) = (1 - 1/(2√3), 1/2)

与式 = OP + AP + CQ + BQ + PQ
 ≧ 1/√3 + 1/√3 + 1/√3 + 1/√3 + (q-p)
 = 4/√3 + (1 - 1/√3)
 = 1 + √3.
0335132人目の素数さん
垢版 |
2020/03/10(火) 09:37:26.87ID:Ct1vj+NA
修正・・・
∠OPC = 120゚ より 点Pは円周
 {x + 1/(2√3)}^2 + (y - 1/2)^2 = 1/3,
上にある。
∴ ∠OPC の二等分線は (-1/√3, 1/2) を通る。

∠AQB = 120゚ より 点Qは円周
 {1+1/(2√3) -x}^2 + (y - 1/2)^2 = 1/3,
上にある。
∴ ∠AQB の二等分線は (1+1/√3, 1/2) を通る。

上記のことから、点P, Q は直線 y=1/2 上に有る。
∴ P(p,r) = (1/(2√3), 1/2)  Q(q,s) = (1 - 1/(2√3), 1/2)

与式 = OP + AP + CQ + BQ + PQ
 ≧ 1/√3 + 1/√3 + 1/√3 + 1/√3 + (q-p)
 = 4/√3 + (1 - 1/√3)
 = 1 + √3.
0336132人目の素数さん
垢版 |
2020/03/10(火) 10:03:31.24ID:Ct1vj+NA
また間違えた・・・・

∴ ∠OPC の二等分線は (-(√3)/2, 1/2) を通る。
∴ ∠AQB の二等分線は (1+(√3)/2, 1/2) を通る。

イナに改名しようかな・・・・
0338132人目の素数さん
垢版 |
2020/03/13(金) 16:13:22.75ID:HFBlyies
>>337
問1を判別式を使って答えようとすると、問2を先に解答しなくちゃならなく
あれっ?ってなって、それが問3のヒントになるのか
うまく作ってあるね
0339132人目の素数さん
垢版 |
2020/03/13(金) 19:09:48.28ID:l20VjRfO
2020年 大阪市立大
 ̄ ̄ ̄ ̄ ̄ ̄ ̄ ̄ ̄
 a,b,c, x,y,z を実数とする。次の問いに答えよ。
問1 a^2 - b^2 > 0 のとき、tについての2次方程式
 (at+x)^2 - (bt+y)^2 = 0
 は実数解をもつことを示せ。
問2 a^2 - b^2 > 0 のとき、
 (ax-by)^2 ≧ (a^2 -b^2)(x^2 -y^2)
 が成り立つことを示せ。
問3 a^2 - b^2 - c^2 > 0 のとき、
 (ax-by-cz)^2 ≧ (a^2 -b^2 -c^2)(x^2 -y^2 -z^2)
 が成り立つことを示せ。
0340132人目の素数さん
垢版 |
2020/03/13(金) 23:26:59.05ID:l20VjRfO
問1
 f(t) = (at+x)^2 - (bt+y)^2
  = (aa-bb)tt + 2(ax-by)t + (xx-yy)
とおく。
aa-bb>0 だから 下に凸な放物線。  
|t| がじゅうぶん大きいとき f(t)>0,
一方、|a|>|b|≧0, a≠0
 f(-x/a) = - {b(-x/a)+y}^2 ≦ 0,
中間値の定理より f(t)=0 は実数解をもつ。
 t = -(x+y)/(a+b), -(x-y)/(a-b)

問2
aa-bb > 0 のとき、
 (判別式) = (ax-by)^2 - (aa-bb)(xx-yy) ≧ 0,
(別解) ラグランジュの恒等式から
 (ax-by)^2 - (aa-bb)(xx-yy) = (ay-bx)^2 ≧ 0

問3
 g(t) = (at+x)^2 - (bt+y)^2 - (ct+z)^2
  = (aa-bb-cc)tt -2(ax-by-cz)t + (xx-yy-zz)
とおく。
aa-bb-cc > 0 だから 下に凸な放物線。
|t| がじゅうぶん大きいとき g(t) >0,
|a|> |b|,|c| ≧0 より a≠0,
 g(-x/a) = - {(-b/a)t +y}^2 - {(-c/a)t +z}^2 ≦ 0,
中間値の定理より g(t)=0 は実数解をもつ。
 (判別式) = (ax-by-cz)^2 - (aa-bb-cc)(xx-yy-zz) ≧ 0.
0341132人目の素数さん
垢版 |
2020/03/14(土) 10:34:57.13ID:iH59lf4s
このスレの解答は・・・・

問1
 (at+x)^2 - (bt+y)^2 = {(a+b)t + (x+y)}{(a-b)t + (x-y)} = 0,
ところで (a+b)(a-b) = aa-bb ≠ 0,
∴ t = -(x+y)/(a+b), -(x-y)/(a-b),

問2
 (ax-by)^2 - (aa-bb)(xx-yy) = (ay-bx)^2 ≧ 0   (*)
 この場合は aa-bb>0 は不要

問3
 xで平方完成する。
 (ax-by-cz)^2 - (aa-bb-cc)(xx-yy-zz)
 = {(bb+cc)xx -2ax(by+cz) + aa(yy+zz)} - (bz-cy)^2  (*)
 = {[(bb+cc)x -a(by+cz)]^2 + (aa-bb-cc)(bz-cy)^2}/(bb+cc)
 ≧ 0,
 等号成立は x/a = y/b = z/c のとき。
 b=c=0 のときは明らか。

*) ラグランジュの恒等式
 (yy+zz) = {(by+cz)^2 + (bz-by)^2}/(bb+cc) を使った。
0342132人目の素数さん
垢版 |
2020/03/16(月) 13:55:03.33ID:bNeBdUF1
〔問題669〕
x>0, y>0 のとき
x^(2x) - 2(x^y)(y^x) + y^(2y) ≧ 0,

(略証)
log は単調増加だから
 (x-y){log(x)-log(y)} ≧ 0
 (x/y)^(x-y) ≧ 1,
 (x^x)(y^y) ≧ (x^y)(y^x),
よって
(左辺) ≧ x^(2x) -2(x^x)(y^y) + y^(2y)
 = (x^x - y^y)^2
 ≧ 0,

[分かスレ458.669]
0343132人目の素数さん
垢版 |
2020/03/16(月) 16:52:52.52ID:bNeBdUF1
〔補題〕
x>0, y>0 とする。
(1) x^x - x^y - y^x + y^y ≧ 0,
(2) x^(2x) - x^(2y) - y^(2x) + y^(2y) ≧ 0,
(3) 0<a≦e のとき、
 x^(ax) - x^(ay) - y^(ax) + y^(ay) ≧ 0,
等号成立は x=y のとき。
0347132人目の素数さん
垢版 |
2020/03/17(火) 05:32:50.08ID:CmDsCyUw
>>346

〔問題5〕
nを2以上の整数とする。
実数 a_1, a_2, ・・・・, a_n が (a_1)^2 + (a_2)^2 + ・・・・ + (a_n)^2 = n を満たすなら
 Σ[1≦i<j≦n] 1/(n−a_i・a_j) ≦ n/2,
が成り立つことを示せ。

Problem 5.
Let n be an integer greater than or equal to 2.
Prove that if the real numbers a_1, a_2, ・・・・, a_n satisfy (a_1)^2 + (a_2)^2 + ・・・・ + (a_n)^2 = n, then
Σ[1≦i<j≦n] 1/(n−a_i・a_j) ≦ n/2,
must hold.

APMO-2012
http://cms.math.ca/Competitions/APMO/ → 2012
Inequalitybot [87] ☆10
0348132人目の素数さん
垢版 |
2020/03/17(火) 06:34:23.26ID:CmDsCyUw
>>347
HM-AM より
 2xy/(n-xy) ≦ (x+y)^2 /{(n-xx)+(n-yy)}
  ≦ yy/(n-xx) + xx/(n-yy),
x=a_i, y=a_j とおく。
 Σ[x≠y] xy/(n-xy) ≦ Σ[x≠y] yy/(n-xx)
 = Σ[i=1,n] Σ[j≠i] (a_j)^2 /{n-(a_i)^2}
 = Σ[i=1,n] 1
 = n,
両辺に n(n-1) をたすと
 Σ[1≦i≠j≦n] n/(n−a_i・a_j) ≦ nn,
 Σ[1≦i<j≦n] 1/(n−a_i・a_j) ≦ n/2,
0350132人目の素数さん
垢版 |
2020/03/17(火) 15:21:04.67ID:CmDsCyUw
(上)
f(x)は、区間[0,1]で定義された連続な実数値関数とする。次を示せ。
 ∫[0,1] ∫[0,1] |f(x)+f(y)| dx dy ≧ ∫[0,1] |f(x)| dx.

Let f(x) be a continuous real-valued function defined on the interval [0,1].
Show that
 ∫[0,1] ∫[0,1] |f(x)+f(y)| dx dy ≧ ∫[0,1] |f(x)| dx.
0351132人目の素数さん
垢版 |
2020/03/17(火) 15:43:05.43ID:CmDsCyUw
(下)
a_1, a_2, ・・・・ は実数とする。
すべてのnについて
 ∫[-∞,∞] {Σ[i=1,n] 1/(1+(x-a_i)^2) }^2 dx ≦ A n.
となる定数Aがあるとする。
すべてのnについて
 Σ[i=1,n] Σ[j=1,n] {1 + (a_i-a_j)^2} ≧ B n^3.
となる定数B >0 があることを示せ。

Let a_1, a_2, ・・・・, be real numbers.
Suppose there is a constant A such that for all n,
 ∫[-∞,∞] {Σ[i=1,n] 1/(1+(x-a_i)^2) }^2 dx ≦ A n.
Prove there is a constant B > 0 such that for all n,
 Σ[i=1,n] Σ[j=1,n] {1 + (a_i-a_j)^2} ≧ B n^3.
0353132人目の素数さん
垢版 |
2020/03/17(火) 17:25:15.41ID:CmDsCyUw
>>348
 2xy ≦ (1/2)(x+y)^2 ≦ xx+yy,  (GM-AM)
より
 2xy/(n-xy) ≦ (x+y)^2 /{2(n-xy)}
 ≦ (x+y)^2 /{(n-xx)+(n-yy)}
 ≦ yy/(n-xx) + xx/(n-yy),  (←コーシー)
x=a_i, y=a_j とおき、1≦i<j≦n でたす。
0354132人目の素数さん
垢版 |
2020/03/17(火) 18:07:30.59ID:CmDsCyUw
>>351
>>352
ついでに・・・
∫ 1/{1+(x-a)^2}*1/{1+(x-b)^2} dx
 = {(1/(a-b))log((1+(x-b)^2)/(1+(x-a)^2)) + arctan(x-a) + arctan(x-b)}/{4+(a-b)^2},

ローレンツ形関数の畳み込み・・・
0355132人目の素数さん
垢版 |
2020/03/17(火) 19:23:46.26ID:CmDsCyUw
>>344
>>345
・2n-1<x<2n のとき
 f(x+2) - f(x) = 1/(x+2) + 1/(2018-x),
 g(x+2) - g(x) = 1/(x+1) + 1/(2017-x),
∴ g(x+2) - f(x+2) > g(x) - f(x) > ・・・・
∴ 1<x<2 について示せば十分。
 g(x) - f(x)
 = - 1/x + 1/(x-1) + 1/(2-x) - 1/(3-x) + Σ[i=2,1008] {1/(2i-x) - 1/(2i+1-x)} + 1/(2018-x)
 > - 1/x + 1/(x-1) + 1/(2-x) - 1/(3-x)
 = 1/{(2-x)(x-1)} - 3/{x(3-x)}
 > 4 - 3/2
 = 5/2.
∵ 1/4 - (2-x)(x-1) = (3/2 -x)^2 ≧ 0,
  x(3-x) - 2 = (2-x)(x-1) > 0,

・2n<x<2n+1 のとき
 f(x-1009), g(x-1009) は 奇関数。
 f(x) = - f(2018-x),
 g(x) = - g(2018-x),
ところで、2n<x<2n+1 ゆえ
 2(1009-n) -1 < 2018-x < 2(1009-n),
∴上記により
 f(x) - g(x) = - f(2018-x) + g(2018-x) > 5/2,

APMO-2018
http://cms.math.ca/Competitions/APMO/ → 2018
0357132人目の素数さん
垢版 |
2020/03/18(水) 05:54:49.50ID:LbXnfiiv
(上)
a,b,c は負でない実数とする。次を示せ。
 a(a-b)(a-2b) + b(b-c)(b-2c) + c(c-a)(c-2a) ≧ 0.

Let a,b,c be non-negative real numbers.
Prove that
 a(a-b)(a-2b) + b(b-c)(b-2c) + c(c-a)(c-2a) ≧ 0.

Wenyu Cao
USA.ELMO-2009 day1-Q3
Inequalitybot [111] ☆5
安藤哲哉「不等式」数学書房 (2012) の p.55, 例題2.2.12(3)
 S_3 + 2S_{2,1} ≧ 3S_{1,2}
0358132人目の素数さん
垢版 |
2020/03/18(水) 06:09:44.78ID:LbXnfiiv
(下)
a,b,c は正の実数で
 a + b + c = a^(1/7) + b^(1/7) + c^(1/7)
を満たすとする。
  (a^a)(b^b)(c^c) ≧ 1
を証明せよ。

Let a,b,c be positive reals satisfying
 a + b + c = a^(1/7) + b^(1/7) + c^(1/7).
Prove that (a^a)(b^b)(c^c) ≧ 1.
 (Evan Chen)
0359132人目の素数さん
垢版 |
2020/03/18(水) 06:25:25.87ID:LbXnfiiv
>>357
 min{a,b,c} = m,
 {a,b,c} = {m,m+x,m+y} とする。(x≧0, y≧0)
 (与式) = 2m(xx-xy+yy) + x(x-2y)^2 + y(x-y)^2 ≧ 0.
0360132人目の素数さん
垢版 |
2020/03/18(水) 06:36:15.76ID:LbXnfiiv
>>352
>>354
対角項 (a=bの場合) は
∫[-∞,∞] 1/(1+(x-a)^2)^2 dx = ∫[-∞,∞] 1/(1+xx)^2 dx = π/2,

∵∫ 1/(1+xx)^2 dx
 = (1/2)∫ {(1-xx)/(1+xx)^2 + 1/(1+xx)} dx
 = x/(2(1+xx)) + (1/2)arctan(x),
0361132人目の素数さん
垢版 |
2020/03/19(木) 13:09:23.34ID:mXsnD9nM
>>343 (1)

(x-1)log(x) ≧ 0 より
 x^x - x ≧ 0,  y^y - y ≧ 0,  ・・・・ (A)

・x≧1 のとき
 x^x -x^y -y^x +y^y
 = {x^(x-y) -1}(x^y - y^y) + {(x^x)(y^y) - (x^y)(y^x)}/(x^y)
 ≧ {x^(x-y) -1}(x^y - y^y)   (>>342 より)
 ≧ 0.

・(x-1)(y-1)≦0 のとき
 x^y ≦ x,  y^x ≦ y,
これと (A) から出る。

・0 < y ≦ x ≦1 のとき
 d = (x-y)/2 ≧ 0, とおく。
 x^x -x^y -y^x +y^y
 = - x^((x+y)/2) {x^(-d) - x^d} + y^((x+y)/2) {y^(-d) - y^d},
(sinhθ)/θ は |θ| について単調増加ゆえ
 {y^(-d) - y^d}/(-log(y)) ≧ {x^(-d) - x^d}/(-log(x)) ≧ 0,
また
 y^((x+y)/2)(-log(y)) ≧ x^((x+y)/2)(-log(x)) ≧ 0,
辺々掛ける。
0362132人目の素数さん
垢版 |
2020/03/19(木) 17:26:22.21ID:mXsnD9nM
>>350
区間[0,1] を
 P = { x∈[0,1] | f(x)≧0 }
 N = { x∈[0,1] | f(x)<0 }
に分ける。
 I_P = ∫_P |f(x)| dx,
 I_N = ∫_N |f(x)| dx,
 p = ∫_P dx,
 n = ∫_N dx,
とすると
 p + n = 1,

(左辺) = ∫PP (|f(x)|+|f(y)|) dxdy
 + ∫NN (|f(x)|+|f(y)|) dxdy
 + ∫PN ||f(x)|-|f(y)|| dxdy
 + ∫NP ||f(x)|-|f(y)|| dxdy
 = ∫PP (|f(x)|+|f(y)|) dxdy
 + ∫NN (|f(x)|+|f(y)|) dxdy
 + |∫PN (|f(x)|-|f(y)|) dxdy |
 + |∫NP (|f(x)|-|f(y)|) dxdy |
 = 2p・I_P + 2n・I_N + 2|n・I_P - p・I_N|
 ≧ 2p・I_P + 2n・I_N + (n-p)(I_P - I_N)  (*)
 = (p+n)(I_P + I_N)
 = I_P + I_N
 = ∫[0,1] |f(x)| dx,

(*)
(n-p)(I_P-I_N) ≦ 0 のときは明らか。
n≧p, I_P≧I_N のとき
 n・I_P - p・I_N = (n-p)I_N + n(I_P-I_N) ≧ (n-p)(I_P-I_N),
p≧n, I_N≧I_P のとき
 p・I_N - n・I_P = (p-n)I_P + p(I_N-I_P) ≧ (n-p)(I_P-I_N).
0366132人目の素数さん
垢版 |
2020/03/20(金) 00:31:37.70ID:lC3HBZ24
>>363

[1]
すべての自然数nについて
 sin(π/4n) ≧ (√2)/(2n),

 sin(π/4n) ≧ (√2)/(2n), ∀n∈N

[2]
自然数 n≧2 と n個の正の実数 a_1, a_2, ・・・・, a_n が
次の不等式を満たすとする。
 Σ[j=1,i] a_j ≦ a_{i+1},  ∀i∈{1,2,・・・・,n-1}
このとき
 Σ[k=1,n-1] a_k/a_{k+1} ≦ n/2.
を証明せよ。

Let be a natural number n≧2 and n positive real numbers
a_1, a_2, ・・・・, a_n that satisfy the inequalities
 Σ[j=1,i] a_j ≦ a_{i+1},  ∀i∈{1,2,・・・・,n-1}
Prove that
 Σ[k=1,n-1] a_k/a_{k+1} ≦ n/2.

[3]
1/2 ≦ a,b,c ≦ 1 とする。
 2 ≦ (a+b)/(1+c) + (b+c)/(1+a) + (c+a)/(1+b) ≦ 3.
を証明せよ。

Let a,b,c ∈ [1/2,1].
Prove that
 2 ≦ (a+b)/(1+c) + (b+c)/(1+a) + (c+a)/(1+b) ≦ 3.
 (selected by Mircea Lascu)

[4]
n∈N゚ とし、
v1, v2, ・・・・, vn は平面内ヴェクトルで、長さは1以下とする。
このとき
 |ξ1・v1 + ξ2・v2 + ・・・・ + ξn・vn | ≦ √2.
となるような ξ1, ξ2, ・・・・, ξn ∈ {-1,1} が存在することを示せ。

Let n∈N゚ and v1, v2, ・・・・, vn be vectors in the plane
 with lengths less than or equal to 1.
Prove that there exists ξ1, ξ2, ・・・・, ξn ∈ {-1,1} such that
 |ξ1・v1 + ξ2・v2 + ・・・・ + ξn・vn | ≦ √2.
0367132人目の素数さん
垢版 |
2020/03/20(金) 04:13:12.05ID:lC3HBZ24
[1]
〔ジョルダンの不等式〕
sinθは上に凸だから
 sin(aθ) ≧ a・sinθ,  (0≦a≦1、0≦θ≦1.43π)
 文献[3] 大関(1987)、p.38 例題2


[3]
(左)
s = a+b+c とおくと
(与式) = (a+b)/(1+c) + (b+c)/(1+a) + (c+a)/(1+b)
 = (1+a+b+c){1/(1+c) + 1/(1+a) + 1/(1+b)} - 3
 ≧ (1+s)・9/(3+s) - 3  (AM-HM)
 = 6 - 18/(3+s)
 ≧ 6 - 4    (s≧3/2)
 = 2,
(右)
(与式) = {(a+b)(1+a)(1+b) + (b+c)(1+b)(1+c) + (c+a)(1+c)(1+a)}/{(1+c)(1+a)(1+b)}
 = 3 - {3(1+c)(1+a)(1+b) - (a+b)(1+a)(1+b) - (b+c)(1+b)(1+c) - (c+a)(1+c)(1+a)}/{(1+c)(1+a)(1+b)}
 = 3 - {(1-a)[4a+(b-c)^2] +(1-b)[4b+(c-a)^2] +(1-c)[4c+(a-b)^2] +3(1-a)(1-b)(1-c)}/{(1+c)(1+a)(1+b)}
 ≦ 3  (0≦a,b,c≦1)
0368132人目の素数さん
垢版 |
2020/03/20(金) 16:48:31.71ID:lC3HBZ24
>>364

[5]
n≧2 は自然数, a_i,b_i (1≦i≦n) は実数で、
 Σ[i=1,n] (a_i)^2 = Σ[j=1,n] (b_j)^2 = 1,
 Σ[i=1,n] (a_i)(b_i) = 0.
のとき
 (Σ[i=1,n] a_i)^2 + (Σ[j=1,n] b_j)^2 ≦ n.
を証明せよ。

For n∈N, n≧2, a_i,b_i∈R, 1≦i≦n, such that
 Σ[i=1,n] (a_i)^2 = Σ[j=1,n] (b_j)^2 = 1,
 Σ[i=1,n] (a_i)(b_i) = 0.
Prove that
 (Σ[i=1,n] a_i)^2 + (Σ[j=1,n] b_j)^2 ≦ n.


[6]
a_1, a_2, a_3, a_4 を任意の4角形の辺とし、周長を 2s とする。
 Σ[i=1,4] 1/(a_i+s) ≦ (2/9)Σ[1≦i<j≦4] 1/√{(s-a_i)(s-a_j)}.
を証明せよ。
等号が成立つのはいつか?

Let a_1, a_2, a_3, a_4 be the sides of an arbitrary quadrilateral
of perimeter 2s. Prove that
 Σ[i=1,4] 1/(a_i+s) ≦ (2/9)Σ[1≦i<j≦4] 1/√{(s-a_i)(s-a_j)}.
When does the equality hold ?


[7]
n≧2 を整数とし、a_1, a_2, ・・・・, a_n を実数とする。
任意の空でない部分集合S ⊂ {1,2,・・・・,n} について
 (Σ[i∈S] a_i)^2 ≦ Σ[1≦i≦j≦n] (a_i+・・・・+a_j)^2.
を証明せよ。

Let n≧2 be an integer and let a_1, a_2, ・・・・, a_n be
real numbers.
Prove that for any non-empty subset S ⊂ {1,2,・・・・,n}
we have
 (Σ[i∈S] a_i)^2 ≦ Σ[1≦i≦j≦n] (a_i+・・・・+a_j)^2.
 (Gabriel Dospinescu)
0369132人目の素数さん
垢版 |
2020/03/21(土) 01:09:55.99ID:a/9U1hEf
[5]
n次元空間で考える。
n個のヴェクトル {a,b,c, ・・・・ } が規格化直交系をなす、とする。
 t = a(a・t) + b(b・t) + c(c・t) + ・・・・,
  (t・t) = (a・t)^2 + (b・t)^2 + (c・t)^2 + ・・・・
   ≧ (a・t)^2 + (b・t)^2.
ここで t = (1,1,・・・・,1) とおく。
0370132人目の素数さん
垢版 |
2020/03/21(土) 04:24:54.60ID:a/9U1hEf
>>359 を改良

 x(x-2y)^2 + y(x-y)^2 ≧ (K-5/2)|處,
ここに
  = (a-b)(b-c)(c-a) = xy(x-y),
 K = √(13/4 + 4√2) = 2.984435331765856875

(略証)
x(x-2y)^2 + y(x-y)^2 - (K-5/2)xy(x-y)
 = x^3 - (K+1/2)x^2・y + (K-1/2)xy^2 + y^3
 = (x+0.2819716800612y)(x-1.8832・・・・y)^2,
 ≧ 0,
(x/y)。 = {1 +√2 +√(2√2 -1)}/2
  = 1.8832035059135


x(x-2y)^2 + y(x-y)^2 + (K+5/2)xy(x-y)
 = x^3 + (K-1/2)x^2・y - (K+1/2)xy^2 + y^3
 = (x+3.546455444685y)(x-0.53101・・・・y)^2
 ≧ 0,
(x/y)。 = {1 +√2 -√(2√2 -1)}/2
  = 0.531010056459569
0372132人目の素数さん
垢版 |
2020/03/22(日) 07:12:46.54ID:fYa2zo9P
9.神田
nを1以上の整数とする。
2n個の正の実数 x1,x2,・・・・,xn, y1,y2,・・・・,yn は
 x1 + x2 + ・・・・ + xn = 1,
をみたす。 1以上n以下の任意の整数の組(i,j)に対し
 x1 + ・・・・ + x_{i-1} ≧ y_j または 2 - x1 - ・・・・ - x_j ≧ y_i
となるとき
 x1・y1 + x2・y2 + ・・・・ + xn・yn ≦ 1
を示せ。
0373132人目の素数さん
垢版 |
2020/03/26(木) 01:44:21.04ID:zUlAmjt2
>>301
 x = max{a,b,c} で場合分けする方法もある・・・・

(i) 0≦x≦1 のとき
 a^2020 - a^2 +4 ≧ a^2018 + 3 > 3, etc.
∴ (左辺) > 27 ≧ (3xx)^3 ≧ (aa+bb+cc)^3.

(ii) x>1 のとき
 x^2020 - x^2 +4 > x^26 - x^2 +4 > x^24 +1 +1 +1 > 4 x^6,
∴ (左辺) ≧ 36 x^6 = (4/3)(3xx)^3 ≧ (4/3)(aa+bb+cc)^3.

http://suseum.jp/gq/question/3129  (クロニャンコさん-改)
0375132人目の素数さん
垢版 |
2020/03/27(金) 05:53:57.81ID:GzR1OrPK
>>324
問2
 c - a > 3 を示せ。
-----------------------------------------------------------------

 f(x) > 4x - abc (0<x<1)  ⇒  a < abc/4,
 f(x) < 4x -12 -abc (3<x<4) ⇒  c > 3 + abc/4,
もあるが・・・・
0376132人目の素数さん
垢版 |
2020/04/01(水) 10:30:00.97ID:3A39oS9Q
>>73
ベルトラン予想(チェビシェフの定理)によらないでも
初等的な論法によって証明できる。   (神戸市・公文氏)

数セミ増刊「数学の問題 第(3)集」日本評論社 (1988)
●107
0378132人目の素数さん
垢版 |
2020/04/03(金) 00:14:12.48ID:mgebV0rK
●107
(3) 自然数nは、n^(1/3) より小さいすべての自然数で割り切れるという。
このような最大のnは何でしょうか。

(4) 自然数nは、n^(1/4) より小さいすべての自然数で割り切れるという。
このような最大のnは何でしょうか。

(5) 自然数nは、n^(1/5) より小さいすべての自然数で割り切れるという。
このような最大のnは何でしょうか。
0379132人目の素数さん
垢版 |
2020/04/03(金) 00:33:15.61ID:mgebV0rK
(3) n = 420 = GCD{1,・・・,7}  [n^(1/3)] = 7.4888724

(4) n = 27720 = GCD{1,・・・,12}  [n^(1/4)] = 12.903226

(5) n = 720720 = GCD{1,・・・,16}   [n^(1/5)] = 14.844081
0380132人目の素数さん
垢版 |
2020/04/03(金) 00:36:13.96ID:mgebV0rK
まちがえた...orz

(3) n = 420 = LCM{1,・・・,7}  n^(1/3) = 7.4888724

(4) n = 27720 = LCM{1,・・・,12}  n^(1/4) = 12.903226

(5) n = 720720 = LCM{1,・・・,16}   n^(1/5) = 14.844081
0381132人目の素数さん
垢版 |
2020/04/08(水) 11:23:03.96ID:pDfrzDrp
(1) x^4 + x^3 - 2x + 1 > 0,
(2) x^4 + x^3 - 2x + 6/7 > 0,

高校数学の質問スレPart404.051〜068
0382132人目の素数さん
垢版 |
2020/04/08(水) 11:39:19.75ID:7I0d5fbg
>>381
(1)は見た瞬間にグラフの概形が頭に描かれたわ。
訓練された不等式ヲタとは、そういうものだ。
0385132人目の素数さん
垢版 |
2020/04/08(水) 13:59:31.73ID:7I0d5fbg
x,y>0に対して、
(x^x)*(y^y)*(Γ((x+y)/2))^2 ≦ Γ(x)*Γ(y)*((x+y)/2)^(x+y).
ここで、Γはガンマ関数
0386132人目の素数さん
垢版 |
2020/04/08(水) 23:51:37.17ID:pDfrzDrp
>>385
 f(x) = log{Γ(x)} - x・log(x),
 f '(x) = ψ(x) - log(x) - 1,
 f "(x) = ψ ' (x) - 1/x > 0,
ここに ψ(x) = Γ '(x)/Γ(x) ・・・・ digamma関数。

∴ f(x) は下に凸。
0387132人目の素数さん
垢版 |
2020/04/08(水) 23:57:56.91ID:pDfrzDrp
>>381
(2)
まず、高次の項を見て
(左辺)=(xx +x/2 -c)^2 +(2c-1/4)xx -(2-c)x + (6/7-cc),
とする。cは定数。

左辺は x = 0.607 の辺りで最小になるので |xx +x/2 -c| も小さいはず。
→ x=0.6 で xx +x/2 -c = 0 となるように c=0.66 とする。

(左辺)=(xx +x/2 -0.66)^2 + 1.07xx - 1.34x + 0.421542857
  ={(x-0.6)(x+1.1)}^2 + 1.07(x-67/107)^2 + 0.002010
  ≧ 0.002010
0388132人目の素数さん
垢版 |
2020/04/10(金) 03:26:14.29ID:IAsBrfBV
>>385

・E. Artin: "Entfuhrung in die Theorie der Gammafunktion",Hamburg (1931)

・高木貞治:「解析概論」改訂第三版, 岩波書店 (1961)
  第5章 §68. ガンマ函数

・E.アルチン「ガンマ関数入門」(はじめよう数学6), 日本評論社 (2002)
   p.126 2200円 上野健爾 [訳・解説]
 http://www.nippyo.co.jp/shop/book/1985.html
0389132人目の素数さん
垢版 |
2020/04/10(金) 12:07:36.86ID:HQzXTvXu
三角形の辺長 a,b,c および面積 S に対して、
√(aa+bb-4S) + √(aa+cc-4S) ≧ √(bb+cc-4S).
0390132人目の素数さん
垢版 |
2020/04/11(土) 14:55:32.07ID:jVXfLHUH
 BC = a, CA = b, AB = c としよう。
頂点Aから対辺BCに下した垂線(の延長線)上に、
 AD = BC = a
となる点Dをとると
 CD = √(aa+bb-2ab・sinC) = √(aa+bb-4S),
 BD = √(aa+cc-2ac・sinB) = √(aa+cc-4S),
で、どうする?
0393132人目の素数さん
垢版 |
2020/04/19(日) 05:23:49.74ID:Cq2k8yf8
>>391
 Tenma Inequality Contest
次を示せ。
1. x,y,z≧0、x+y+z=1 のとき
  7/9 ≦ (xyz+1)/(xy+yz+zx+1) ≦ 1,

2. x,y,z≧0、 xyz=1 のとき
 (y/x + z/y + x/z) + (x/y + y/z + z/x) ≧ (x+y+z) + (1/x + 1/y + 1/z) ≧ 6,

3. x,y,z>0, x+y+z=1 のとき
 1/{x(y+z)} + 1/{y(z+x)} + 1/{z(x+y)} ≧ 27/2 ≧ 1/(x^4 + y^4 + z^4 + xyz),

4. x,y,z≧0 のとき
 3(x^3+y^3+z^3 +1) + 4(xy+yz+zx) ≧ 9xyz + 4(x+y+z),
0394132人目の素数さん
垢版 |
2020/04/19(日) 05:49:57.17ID:Cq2k8yf8
>>393
1.
 s = x+y+z
 t = xy+yz+zx
 u = xyz
とおく。
左)
 u + 1 - (7/9)(t+1) = u + s^3 - (7/9)s(t+ss)
 = (2s^3 -7st+9u)/9
 ≧ {(s^3 -4st+9u) + s(ss-3t)} ≧ 0,
 等号成立は x=y=z=1/3,
右)
 (分母) - (分子) = (xy+yz+zx) - xyz
 = (x+y+z)(xy+yz+zx) - xyz
 = (x+y)(x+z)(z+x)
 ≧ 0,
 等号成立は {x,y,z} = {0,0,1}
0395132人目の素数さん
垢版 |
2020/04/19(日) 06:02:04.63ID:Cq2k8yf8
>>393
2.
xyz = G^3 とすると AM-GMで
 x+y+z ≧ 3G, 1/x+1/y+1/z ≧ 3/G,
(左辺) = (x+y+z)(1/x+1/y+1/z) -3
 ≧ 3(x+y+z)/G + 3G(1/x+1/y+1/z) -12
 ≧ (x+y+z)/G + G(1/x+1/y+1/z),


なお、対称式でなくても AM-GMで
 (1/3)(x/y+x/y+y/z) ≧ x/G, etc.
巡回的にたすと
 x/y + y/z + z/x ≧ (x+y+z)/G,
同様にして
 (1/3)(x/y+y/z+y/z) ≧ G/z, etc.
巡回的にたすと
 x/y + y/z + z/x ≧ (1/x+1/y+1/z)G,
は出る。

文献[9] 佐藤(訳) 朝倉書店 (2013) p.26 演習問題1.75
0396132人目の素数さん
垢版 |
2020/04/19(日) 06:22:15.43ID:Cq2k8yf8
>>393
3.
 1/(x(y+z))= 1/(x(s-x))={1/x + 1/(s-x)}/s,
(左辺)≧{1/x + 1/y + 1/z + 1/(s-x)+ 1/(s-y)+ 1/(s-z)}/s
 ≧{9/(x+y+z)}+ 9/(3s-x-y-z)}/s   (← AM-HM)
 = 9/(ss)+ 9/(2ss)
 = 27/(2ss),

x^4 +y^4 +z^4 + xyz
 ≧(x+y+z)(x^3+y^3+z^3)/3 + xyz
 = s(s^3 -3st +3u)/3 + u
 =(s^3 -3st +6u)/3   (s=1)
 =(2/9)(s^3 -4st +9u)+(1/27)s(ss-3t)+(2/27)s^3
 ≧(2/27)s^3,
∴(右辺)≦ 27/(2s^3),
0397132人目の素数さん
垢版 |
2020/04/19(日) 10:07:36.72ID:Cq2k8yf8
>>393
4.
(左辺)-(右辺)= 3s(ss-3t)+4t -4s +3
 ={(3s^3 -4s +3)(ss-3t) + t(2s-3)^2}/ss
 ≧ 0,

3s^3 -4s +3 = 3(s+4/3)(s-2/3)^2 + 11/9 ≧ 11/9.
0399132人目の素数さん
垢版 |
2020/04/21(火) 08:05:27.64ID:6SMLYdGW
>>398
〔問題5〕
nを2以上の整数とし、a_1, a_2, ・・・・, a_n を正の整数とする。
このとき、次の3つの条件をみたす正の整数 b_1, b_2, ・・・・, b_n が存在することを示せ。
(A) i = 1,2,・・・・,n に対して a_i≦b_i である。
(B) b_1, b_2, ・・・・, b_n をnで割った余りはすべて異なる。
(C) 不等式
  b_1 + b_2 + ・・・・+ b_n ≦ n((n-1)/2 +[(a_1+a_2+・・・・+a_n)/n]).
 が成り立つ。
 ただし、実数xに対してxを超えない最大の整数を[x]で表わす。

>>392
 そうかなぁ?
0400132人目の素数さん
垢版 |
2020/04/28(火) 06:57:59.84ID:A7QrgcmB
R^3上の(可測)集合A,Bに対して、
A+B:={x+y | x∈A,y∈B}とする.
このとき、
(A+Bの体積)^(1/3)≧(Aの体積)^(1/3)+(Bの体積)^(1/3)
を証明せよ.
0401132人目の素数さん
垢版 |
2020/04/28(火) 07:02:02.30ID:A7QrgcmB
>>400
ちなみに、A,Bを2次元平面上の図形として、
(A+Bの面積)^(1/2)≧(Aの面積)^(1/2)+(Bの面積)^(1/2)
となることは比較的簡単に証明出来ます
0402132人目の素数さん
垢版 |
2020/05/03(日) 10:38:33.83ID:u2nPgxPR
a,b,c>0, λ≧0
(a/b + b/c + c/a){a/(a+λb) + b/(b+λc) + c/(c+λa)} ≧ 9/(λ+1).

RMMかどこかで昔拾ったものだったか?

 ( ゚∀゚) プゥ
  ノヽノ) =3 'A`)ノ ヒャー
  くく   へヘノ
0403132人目の素数さん
垢版 |
2020/05/04(月) 16:31:00.11ID:jDRWX2Ph
3月の宿題で(1)のみ正解の数弱@shukudai_sujaku

昨年度の大学への数学(大数)での勝率は、

学コンBコースが 1/1 = 100% ,

宿題が 3/10 = 30% でした!

宿題の勝率が低すぎると思うので、

これからは一層精進していきたいです!

https://twitter.com/shukudai_sujaku
https://twitter.com/5chan_nel (5ch newer account)
0404132人目の素数さん
垢版 |
2020/05/05(火) 12:29:59.96ID:b2IqdVzK
3月の宿題で(1)のみ正解の数弱@shukudai_sujaku

昨年度の大学への数学(大数)での勝率は、

学コンBコースが 1/1 = 100% ,

宿題が 3/10 = 30% でした!

宿題の勝率が低すぎると思うので、

これからは一層精進していきたいです!

https://twitter.com/shukudai_sujaku
https://twitter.com/5chan_nel (5ch newer account)
0405132人目の素数さん
垢版 |
2020/05/05(火) 17:26:38.46ID:B6ZQqn9P
>>401
平面のときのも全然証明わかりません
証明もしくは出典教えてほしいです
0406132人目の素数さん
垢版 |
2020/05/05(火) 23:55:57.67ID:2A4Z2g/l
>>405
ヒントを言うとまずはA,B共に長方形の場合で示してください
ある有名不等式になります

どうしても答えが知りたければ
「Brunn–Minkowskiの不等式」でググってください
0407132人目の素数さん
垢版 |
2020/05/06(水) 00:23:27.36ID:1eDWV5m3
>>406
ありがとう
調べたらかなり有名みたいだね

もちろん最初に同じ向きの長方形の場合を考えてみたけど、そこから一般の場合にどう持っていくのかが謎すぎた
和をとった後の図形がどれくらい小さくなるかは元の図形の大域的な様子が必要そうに思えてさ
wikiぱっと見た感じでは凸包をとってよくて、さらに凸図形の場合は表面積の情報で抑えられるという感じなのかな
0408132人目の素数さん
垢版 |
2020/05/06(水) 00:35:55.88ID:sAc0zF0B
>>407
測度論で長方形から一般の可測集合に主張を拡張したい場合は「Dynkin's π-λ theorem」という便利な定理があります
0409132人目の素数さん
垢版 |
2020/05/06(水) 00:45:25.71ID:sAc0zF0B
適当にパラメータ付けした図形にBrunn–Minkowskiの不等式を適応すれば
大量に不等式を生成することが出来ます
0411132人目の素数さん
垢版 |
2020/05/08(金) 11:46:01.84ID:WmDpVhCu
3月の宿題で(1)のみ正解の数弱@shukudai_sujaku

昨年度の大学への数学(大数)での勝率は、

学コンBコースが 1/1 = 100% ,

宿題が 3/10 = 30% でした!

宿題の勝率が低すぎると思うので、

これからは一層精進していきたいです!

https://twitter.com/shukudai_sujaku
https://twitter.com/5chan_nel (5ch newer account)
0412132人目の素数さん
垢版 |
2020/05/08(金) 20:28:34.01ID:gGrwQusG
>>410

a,b,c>0 のとき、
 30sst ≦ 7s^4 + 9tt + 54su,
を示せ。
ここに s = a+b+c, t = ab+bc+ca, u = abc とおいた。


実数列{a_i},{b_j},{c_k}がある。以下の不等式が成り立つことを証明せよ。
(Σaa)(Σbc)^2 +(Σbb)(Σca)^2 +(Σcc)(Σab)^2 ≦(Σaa)(Σbb)(Σcc)+ 2(Σab)(Σbc)(Σca),


x1,x2,・・・,xn を実数とし、x=(x1,x2,・・・・,xn)と書く。
f(x)と g(x)は対称式である。
f(x)≦ g(x)が成り立ち、
等号成立が x = x_p =(y1,y2,・・・・,yn)に限るとしても
 f(x)≦ g(x_p)
が成り立たない例があることを示せ。


0 < x,y,z のとき、
 x/y + y/z + z/x ≧ (x+y+z)^2 /(xy+yz+zx),
を示せ。

5(改)
 1 < log([(e-2)x+2]/[(e-2)(x-1)+2]) + x・log(1+1/x)< f(x), (x>1)
ここに f(x)={x・log(x)+ log(x+1)- log(2Γ(x+1))}/(x-1),
 lim[x→1]f(x)= γ + 1/2,
 lim[x→∞]f(x)= 1,


∀x,y ∈[0,π/2)
 sin(x+y)+ tan(x)tan(y)≦ √{1+tan(x)^2}・√{1+tan(y)^2},
0413132人目の素数さん
垢版 |
2020/05/08(金) 20:45:55.20ID:gGrwQusG

 (右辺)= 7s^4 + 9tt + 54su
 = 30sst +(ss-3t)^2 + 6s(s^3 -4st+9u)
 = 30sst +(F_0)^2 + 6s・F_1
 ≧ 30sst,
 F_0 = ss -3t ≧ 0, F_1 = s^3 -4st +9u ≧ 0. (シューア)


 f(x)= Σ[i=1,n] (x_i)^2, g(x)= 2Σ[i=1,n] (x_i)^2,
 g(x_p)= g(o)= 0.
何か勘違いしてるかな?


コーシーで。
(分数形のものは Engel型のCauchy とか、Arthur Engelの最小原理と呼ぶらしい。)
文献[9] 佐藤(訳) 朝倉書店 (2013) p.44-45


 (左辺)^2 ≦{1 +|tan(x)tan(y)|}^2
 ≦ √{[1 +|tan(x)tan(y)|]^2 +[tan(x)±tan(y)]^2}/2
 ={1+tan(x)^2} {1+tan(y)^2},
或いは同じことだが
 (左辺)^2 ≦(1 +|tan(x)tan(y)|)^2
 ={cos(x)cos(y)± sin(x)sin(y)}^2 /{cos(x)cos(y)}^2
 = cos(x干y)^2 /{cos(x)cos(y)}^2  (←複号は適当な方をとる)
 ≦ 1/{cos(x)cos(y)}^2,
0414132人目の素数さん
垢版 |
2020/05/08(金) 23:59:46.30ID:gGrwQusG

 Σaa = AA, Σbb = BB, Σcc = CC,
 Σab = AB cosγ, Σbc = BC cosα, Σca = CA cosβ,
とおく。(0≦α,β,γ≦π)
球面三角不等式
 |α-β|≦ γ ≦ α+β,
より
 cos(α-β)≧ cosγ ≧ cos(α+β),
|cosγ - cosα・cosβ|≦|sinα・sinβ|,
2乗して整理すると
(cosα)^2 +(cosβ)^2 +(cosγ)^2 ≦ 1 + 2(cosα)(cosβ)(cosγ),
これに(ABC)^2 を掛ければ
 (Σaa)(Σbc)^2 +(Σbb)(Σca)^2 +(Σcc)(Σab)^2 ≦(Σaa)(Σbb)(Σcc)+ 2(Σab)(Σbc)(Σca).
0415132人目の素数さん
垢版 |
2020/05/09(土) 06:14:35.60ID:pHr5kdzK

x≒1 では
x・log(x)+ log(x+1)- log(2)=(3/2)(x-1)+(3/8)(x-1)^2 -(1/8)(x-1)^3 + ・・・
log{Γ(x+1)}=(1-γ)(x-1)+(ππ-6)/12・(x-1)^2 - 0.0673523(x-1)^3 + ・・・・
f(x)=(γ+1/2)+(7/8 - ππ/12)(x-1) - 0.0576477(x-1)^3 + ・・・・

x >>1 では
x・log(x)+ log(x+1)=(x+1)log(x)+ 1/x - 1/(2xx)+ 1/(3x^3)- ・・・・
log{2Γ(x+1)}=(x+1/2)log(x)-x +(1/2)log(8π)+ 1/(12x) - 1/(360x^3) + ・・・
f(x) ={x +(1/2)log(x)-(1/2)log(8π)+ 11/(12x)- 1/(2xx)+ 121/(360x^3)}/(x-1)
 → 1 (x→∞)

6(訂正)
 (左辺)^2 ≦{1 + |tan(x)tan(y)|}^2
 ≦{1 + |tan(x)tan(y)| })^2 +{ |tan(x)| - |tan(y)| }^2
 ={1 + tan(x)^2} {1 + tan(y)^2},
0417132人目の素数さん
垢版 |
2020/05/14(木) 01:49:21.16ID:431XMp+q
n, k は2以上の整数、Aはn×nエルミート行列のとき、
(det A)^(1/n) ≦ [{(tr A)^k - tr(A^k)}/(n^k - n)]^(1/k).
0418132人目の素数さん
垢版 |
2020/05/14(木) 08:31:45.43ID:w+h9h8DE
>>414

      | (a・a) (a・b) (a・c) |
RHS - LHS =|(b・a) (b・b) (b・c) |  (Grammian)
      | (c・a) (c・b) (c・c) |

  | a |
 =|b|| a b c | ≧ 0,
  | c |

桑野耕一「ラグランジュ恒等式とは何か」
  数学セミナー、連載(2006年4月号〜)

なお、上式は(|a||b||c|)^2 以下になる。(Hadamardの不等式)

高木:「解析概論」改訂第三版、岩波書店(1961)
 第2章 §26.極大・極小 [例2] p.72〜75
分かスレ459.588, 594, 759 など。
0420132人目の素数さん
垢版 |
2020/05/18(月) 21:35:49.20ID:0gqdjMjt
〔問題〕
x,y,zが非負実数全体を動くとき
 (x+y+z)^3 /(xyy+yzz+zxx+xyz)≧ 27/4,
を示せ。

例の問題...
0421132人目の素数さん
垢版 |
2020/05/18(月) 21:49:34.65ID:0gqdjMjt
>>420
(略証)
0 ≦ Min{x,y,z}= x としてもよい。
4(x+y+z)^3 - 27(xyy+yzz+zxx+xyz)
 = 9x(xx+yy+zz-xy-yz-zx) + (y+4z-5x)(x-2y+z)^2 ≧ 0,
等号成立は(x,y,z) = (1,1,1) (0,1,2) (1,2,0) (2,0,1)
[前スレ.014-020]
文献[8] 安藤:「不等式」数学書房 (2012)例題2.2.12(7)p.56
Inequalitybot [169]

(類題)
カナダMO-1995 A5
イギリスMO-2009 A4
Inequalitybot [61]
0422132人目の素数さん
垢版 |
2020/05/18(月) 23:15:09.32ID:0gqdjMjt
>>402

(1+λ)(a/b + b/c + c/a)≧(a/b + b/c + c/a)+ 3λ  (AM-GM)
 =(a+λb)/b +(b+λc)/c +(c+λa)/a,
∴ コーシーで
(1+λ)・LHS ≧{√(a/b)+ √(b/c)+ √(c/a)}^2
 ≧ 3^2       (AM-GM)
 = 9,
0423132人目の素数さん
垢版 |
2020/05/19(火) 01:32:34.01ID:G3uxUJZ9
>>421
巡回的なものを非巡回的な式に変形して示すの不思議
巡回的な変形でも示せるのか気になる
0425132人目の素数さん
垢版 |
2020/05/22(金) 07:21:06.00ID:y+ggBWMl
>>416
(略解)
Let x be the smallest odd number.
x + (x+2) + (x+4) ≦ 54,
     3x + 6 ≦ 54,
       x ≦ 16,
       x ≦ 15, (x is odd number)
∴ The largest value of the smallest nember is 15.
0427132人目の素数さん
垢版 |
2020/05/26(火) 22:40:14.24ID:jFtHfHtW
a,b,c>0、G=(abc)^(1/3) に対して
a/(a+2b+3) + b/(b+2c+3) + c/(c+2a+3) ≧ G/(1+G).

JBMO2020って何だよ?
0428132人目の素数さん
垢版 |
2020/05/27(水) 06:28:31.08ID:2I72JytV
>>427
{a(a+2b+3)+ b(b+2c+3)+ c(c+2a+3)}= (a+b+c)(a+b+c+3)= 9A(A+1),
を左辺に掛ける。 コーシーにより
9A(1+A)(左辺)≧(a+b+c)^2 = 9AA,
∴ (左辺)≧ A/(1+A)≧ G/(1+G),   (← AM-GM)

JBMO = Junior Balkan Mathematical Olympiad

なお、日本のは JJMO
0429132人目の素数さん
垢版 |
2020/05/27(水) 15:36:19.80ID:2I72JytV
>>427
1/(1+x)≧ 1-x より
a/(a+2b+3) = a/{(a+b+c+3) + (b-c)}
 = a/{(a+b+c+3) (1+x)}
 ≧ a(1-x)/(a+b+c+3)
 = a/(a+b+c+3) - (ab-ca)/(a+b+c+3)^2,
循環的にたす。
 (左辺)≧(a+b+c)/(a+b+c+3)≧ G/(G+1)),
0430132人目の素数さん
垢版 |
2020/05/27(水) 16:26:37.09ID:2I72JytV
>>266 のことでござるか・・・・
 JBMO2020 はバルカンMO (BMO) のジュニア版のこと >>428
じゃなく、ただのハンドル名かも・・・・
0431132人目の素数さん
垢版 |
2020/05/27(水) 18:38:24.11ID:2I72JytV
〔問題3〕改
a,b,c は正の実数で 1/a + 1/b + 1/c ≧ 3 であるとする。
 (a+1/b)^2 + (b+1/c)^2 + (c+1/a)^2 ≧ 3(a+b+c+1).
を証明せよ。 等号が成立つのはいつか?

Problem 3. (modified)
 Let a,b,c be positive real numbers such that 1/a+1/b+1/c ≧3.
Prove that
 (a+1/b)^2 + (b+1/c)^2 + (c+1/a)^2 ≧ 3(a+b+c+1).
When does equality hold ?
 JBMO-2014 P.3
0433132人目の素数さん
垢版 |
2020/05/30(土) 19:23:31.86ID:vR2Jo4eU
[例9-3] 改
 次の不等式をみたす整数a,b,cで、どれか1つは0でなく、
かつどの絶対値も100万を超えないものが存在することを示せ。
  |a + b√2 + c√3|< 10^(-12),

[第2章.274-276]
秋山 仁 + ピーター・フランクル 共著:
[完全攻略]数学オリンピック, p.47-48, 日本評論社 (1991/Nov)

注)鳩ノ巣原理では解けません。
0434132人目の素数さん
垢版 |
2020/05/31(日) 08:48:01.74ID:vwaCsVj1
二変数a,bの相加平均、相乗平均、調和平均の幾何学的な証明(直径a+bの円)がありますけど
三変数a,b,cの場合の幾何学的な証明はあるのでしょうか?相加平均は重心という自明な意味が
あるけど残りの二つの幾何的な意味がよくわからない。
0437132人目の素数さん
垢版 |
2020/05/31(日) 17:06:28.95ID:vwaCsVj1
球だとできるんですか?立方体で無理やり解釈することもできるけど一目瞭然では全然ないようなのしかできない。。

3辺がa,b,cの直方体の体積V=abc ,表面積S=2(ab+bc+ca) とすると
3辺の長さの相加平均≧直方体と同体積の立方体の1辺≧V/表面積の平均値
0438132人目の素数さん
垢版 |
2020/06/01(月) 03:27:29.95ID:LHxMDESI
>>433
97 -56√3 = 1/(97+56√3) = 0.005154776
99 -70√2 = 1/(99+70√2) = 0.005050634
辺々足して14で割る。
14 - 5√2 - 4√3 = 7.28957859×10^(-4) ・・・・ (1)
辺々引いて2で割る。
-1 + 35√2 - 28√3 = 5.207113×10^(-5) ・・・・ (2)
(2)×14 - (1)
-28 + 495√2 - 388√3 = 3.7957659×10^(-8) ・・・・ (3)
また、
127 + 138√2 -186√3 = 2.139967594×10^(-5) ・・・・ (4)
205 - 58√2 - 71√3 = 6.04497020×10^(-6) ・・・・ (5)

* 3.352882344113・・・・×10^(-13)まではあるらしい。
0440132人目の素数さん
垢版 |
2020/06/02(火) 03:56:28.89ID:TPydHgX/
〔問題404-627〕
0<x<y<1<x+y のとき
 {(1-x)(1-y)(x+y-1)(y-x)^2}/(x+y)^2
の最大値を求めるにはどうすればいいでしょう?

[高校数学の質問スレPart404.627,632,635]
0441132人目の素数さん
垢版 |
2020/06/02(火) 04:18:56.72ID:TPydHgX/
〔問題404-634〕
q>1 を定数とする。
x^q + y^q + z^q = 1 を満たす正の実数x,y,zであって
(x+1)(y+1)(z+1) を最大にするものを求めよ。

[高校数学の質問スレPart404.634,639,648]
0444132人目の素数さん
垢版 |
2020/06/02(火) 12:22:08.87ID:TPydHgX/
問題1
 三角形ABCとその内部の点Pは、AB=7、AC=8、PB=1、PC=4 を満たす。
∠BAC と ∠BPC の二等分線が平行であるときの、BCの長さを求めよ。

(解答例)
題意より、点Pは△ABCの垂心となる。
 AP⊥BC、BP⊥CA、CP⊥AB
二等分線の方向をx軸とすれば傾きは
 AP -1/5、 BC 5
 BP -3、 CA 1/3
 CP 3、 AB -1/3
よって
 A(0,0) B(21/√10, -7/√10) C(24/√10, 8/√10) P(20/√10, -4/√10)
長さは
 BC = 3√(13/5) = 4.83735
 AP = 4√(13/5) = 6.4498
0445132人目の素数さん
垢版 |
2020/06/02(火) 14:08:28.88ID:TPydHgX/
問題2
 nを4以上の整数とする。
ある正n角形の各頂点にはある頂点から反時計回りで1からnの整数が
書かれている。
この正n角形にn-3本の対角線(辺は含まない)を どの二つの対角線も
交わらないように取ると、正n角形はn-2個の三角形に分けられる。
(証明不要)
これらn-2個の三角形それぞれの得点をその三角形の3頂点に書かれた
整数の和とする。
n-2個の三角形の得点の総和として考えられる最大の値を求めよ。

(解答例)
Σ[k=1,n] (点 'k' を共有する三角形の数)
= Σ[k=1,n] (点 'k' を端点とする対角線の数+1)
∴ 点 'n' を端点とする対角線n-3本をとれば 2n(n-2)  (最大)

逆に 点 '1' を端点とする対角線n-3本を取れば (n+3)(n-2) (最小)
0446132人目の素数さん
垢版 |
2020/06/02(火) 14:27:03.25ID:TPydHgX/
問題3
 以下の等式を満たす正の整数の組 (a,b,c) を全て求めよ。
   a^(bc) + b^(ca) = c^(ab)

(解答例)
 a^(bc) < c^(ab) より a^c < c^a,
 b^(ac) < c^(ab) より b^c < c^b, 
∴ a^(1/a), b^(1/b) < c^(1/c),
一方
 1 < n^(1/n) < ・・・・ < 5^(1/5) < 4^(1/4) = 2^(1/2) < 3^(1/3),
と比べて
(a,b,c) = (1,1,2) (1,2,3) (2,1,3)
0447132人目の素数さん
垢版 |
2020/06/02(火) 17:47:39.85ID:TPydHgX/
問題4
 内接円を持つ四角形ABCDの辺 AB, BC, CD, DA 上に
それぞれ P, Q, R, S をとり、線分PRとQSの交点をKとする。
四角形 APKS, BQKP, CRKQ に内接円が存在するとき、
四角形 DSKR にも内接円が存在することを示せ。

(解答例)
 内接円をもつ ⇔ 2組の対辺の和が等しい。
だけでは解けぬ。どうするか?
0448132人目の素数さん
垢版 |
2020/06/02(火) 18:21:12.25ID:TPydHgX/
問題5
 ある2以上の整数dは、ちょうどk個の正の約数 d_1 < d_2 < ・・・・・ < d_k を持ち、
任意のk個の正の実数 x_1, x_2, ・・・・・, x_k に対して以下の不等式を満たす。
このようなdを全て求めよ。

(√x_1 + √x_2 + ・・・・・ + √x_k + 1)^(2d)
  ≦ (k+1)^(2d)・Π[i=1,k] {(x_i^d_i + k)/(k+1)}^d_(k+1-i)
  ≦ 2^(2d-k)・Π[i=1,k] (x_i^d + k^d_(k+1-i)),

ガラパゴス不等式と名付けたい・・・・
0449132人目の素数さん
垢版 |
2020/06/03(水) 14:14:42.78ID:Vj2o+qIA
[第7章.946]
 i-j=k をみたすn-k項と i-j=n-k をみたすk項 の計n項で Jensen する。
 n項の引数の和はkとなる。
Σ[i-j=k] f(x_i - x_j) + Σ[i-j=n-k] f(1 - x_i + x_j) ≧ n f(k/n),
k=1,2,・・・・,n-1 でたす。

--------------------------------------------------------
蛇足だが・・・・
 f(a) ≦ n ∫[a-1/2n, a+1/2n] f(x) dx,
より
 Σ[k=1,n-1] f(k/n) ≦ n∫[1/2n, 1-1/2n] f(x) dx ≦ (n-1)∫[0,1] f(x) dx,

http://suseum.jp/gq/question/2724
0450132人目の素数さん
垢版 |
2020/06/04(木) 03:52:52.71ID:UZmO2K4a
>>440
s = x+y, t=xy とおくと
 0 < t < 1 < s < 2,
16(1-x)(1-y)(x-y)^2 = 16(1-s+t)(ss-4t)  (← tの2次式)
 = (2-s)^4 - {(2+s)^2 -8t -8}^2   (← 平方完成)
 ≦ (2-s)^4,
より
(与式)≦ (s-1)(2-s)^4 /(16ss)
 = (2/√3 -1)^3 - g(s)(s-2/√3)^2 /(16ss)
 ≦ (2/√3 -1)^3
 = 0.003702332976 = M,

等号は s = 2/√3 = 1.1547 のとき。

10+M = 10 + (2/√3 -1)^3
 = (2/√3 +1)^3
 = 1/{3(2/√3 -1)}^3
 = 1/(27M),

M = 1/{27(10+M)} < 1/270 = 0.00370370・・・・

g(s) = {(√3)(2-s)^3 + (3√3 -4)(2-s)^2 + 4(3√3 -5)(2-s) + 8(7-4√3)}/(√3)
 > 8(7-4√3)/√3
 = 0.331615  (s<2)
∵ 5/3 < √3 < 7/4,
0451132人目の素数さん
垢版 |
2020/06/04(木) 04:14:53.02ID:UZmO2K4a
>>441
 q乗平均Q ≧ 相加平均A
より
 (x+1)(y+1) = (A+1)^2 - (1/4)(x-y)^2
  ≦ (A+1)^2
  ≦ (Q+1)^2,
∴ (x+1)(y+1)(z+1) ≦ (Q+1)(Q+1)(z+1),
∴ もし最大値があるとすれば、それは x=y=z に限る。
0452132人目の素数さん
垢版 |
2020/06/04(木) 17:35:49.34ID:UZmO2K4a
>444
△ABCの垂心をHとおく。
辺BCに関してHと対称な点をDとおくと、
∠D = ∠BHC = 180゚ - ∠A,
∴ Dは△ABCの外接円上にある。
 OA = OD
∴ ∠OAH = ∠ODH  ・・・・ (1)

△ABCを中点三角形とするような大三角形△A'B'C'を考える。
その垂心は三角形ABCの外心Oである。
相似関係より、
 ∠BAO=∠CAH, ∠CAH=∠BAO,
∴ ∠A の二等分線は ∠OAHの二等分線。
 ∠BDO=∠CDH, ∠CDH=∠BDO,
∴ ∠D の二等分線は ∠ODHの二等分線。
(1) より
 ∠BHC の二等分線 // ∠Aの二等分線
0453132人目の素数さん
垢版 |
2020/06/06(土) 04:26:40.25ID:klZxi4yn
[AMM, Problem 12154]
Let r_a , r_b , and r_c be the exradii of a triangle with circumradius R and inradius r. Prove
 r_a/(r_b + r_c) + r_b/(r_c + r_a) + r_c/(r_a + r_b) ≧ 2 - r/R.
0455132人目の素数さん
垢版 |
2020/06/08(月) 04:37:40.73ID:4nsS10XA
>>438
97-56√3 = (2-√3)^4 = 1/(2+√3)^4,
99-70√2 = (√2 -1)^6 = 1/(1+√2)^6,
より
-28 +495√2 -388√3 = {-(√2 -1)^12 +(2-√3)^8}/28, ・・・・ (3)
0456132人目の素数さん
垢版 |
2020/06/09(火) 10:39:14.84ID:oCR5MqlE
38419 -13895√2 -10836√3 = 9.489944×10^(-9)  ・・・・ (6)
1920 -42258√2 +33395√3 = 4.066451×10^(-10)  ・・・・ (7)

(4)×2 - (5)×7
 -1181 +682√2 +125√3 = 4.84560485×10^(-7)  ・・・・ (8)

(6)×4 - (3)
 153704 - 56075√2 -42956√3 = 2.11768032×10^(-12)  ・・・・ (9)
0457132人目の素数さん
垢版 |
2020/06/15(月) 09:49:34.02ID:m4MzqaBi
>>372
 (1,n) から 1 ≧ y1
 (i,i) から 2-S_i ≧ yi,
 (n,n) から 1 ≧ yn,
 (i,j) i<j から
  S_(i-1) ≧ yj または 2-S_j ≧ yi  ・・・・ これが難解

なお (1,2) 〜 (1,n-1) と (i,j) i>j は不要

 S_k = x0 + x1 + x2 + ・・・・・ + xk,
0458132人目の素数さん
垢版 |
2020/06/28(日) 15:44:40.60ID:OH7XqlAJ
(1) 円周率πに対して、3.1<π<3.2を示せ
(2)ネイピア数eに対して、2.7<e<2.8を示せ
0459132人目の素数さん
垢版 |
2020/06/29(月) 17:05:05.68ID:4ejNywyM
(1)
>>100 で θ=π/6 とおくと
 18/(2 + 2 + √3) < π < 2(1/2 + 1/2 + 1/√3),
 3.140237343 < π < 3.15470054

(2)
特に x=1 のとき、剰余項を入れて書けば
  e = 1 + 1/1! + 1/2! + ・・・・ + 1/n! + R_(n+1)   (11)
   R_(n+1) = e^θ/(n+1)! < 3/(n+1)!
今(11)を用いて1/n!を計算して行けば、n=4 までは右図のようになる。
それらを加えてeの近似値を得るが、剰余項 R_5 < 1/40 だから
 2 + 17/24 < e < 2 + 11/15
 2.7083333 < e < 2.7333333

高木:「解析概論」改訂第三版、岩波書店 (1961)
 第2章 微分法、§25.Taylorの公式、p.66
0460132人目の素数さん
垢版 |
2020/06/29(月) 19:07:55.63ID:4ejNywyM
(1)
Simpson の方法 は(3)の応用である。
h=(b-a)/2n と置いて y_(2i-1) に隣る二つの区間に関する積分∫f(x)dx の
近似値として(3)のように
   (h/3){y_(2i-2) + 4y_(2i-1) + y_(2i)}
を取って i=1,2,・・・・,n に亘って総計すれば
 ∫[a,b] f(x)dx
  ≒ (h/3){y_0 +4y_1 +2y_2 +4y_3 + ・・・・ + 2y_(2n-2) + 4y_(2n-1) + y_(2n)},
これが Simpson の公式である。              ・・・・ (5)
 もしも(4)によって剰余項をも取るならば、総計して
 R = -{n(h^5)/90}f^(4)(ξ) = -{(b-a)(h^4)/180}f^(4)(ξ),
これは Simpsonの公式の誤差の限界を与える。
 一例として π/4 = ∫[0,1] 1/(1+x^2) dx からπの近似値を計算してみよう。
n=5 とすれば h=0.1
 π/4 = (0.1/3)(1/1.00 + 4/1.01 + 2/1.04 + 4/1.09 + 2/1.16 + 4/1.25
         + 2/1.36 + 4/1.49 + 2/1.64 + 4/1.81 + 1/2.00) + R
   = (0.1/3)・23.5619446 + R
   = 0.7853981535 + R
∴ π = 3.141592614 + 4R

 -1.333×10^(-5) < R < 1.667×10^(-6)
 {実際は R = 9.91264×10^(-9)}

高木:「解析概論」改訂第三版、岩波書店 (1961)
 第3章 積分法、§38.定積分の近似計算 p.127-128
 (貞治先生も筆の誤り?)
0461132人目の素数さん
垢版 |
2020/07/02(木) 07:33:44.87ID:VISrmZkI
u,v,w∈R^n
(||v||^2 ||w||^2 - (v,w)^2) ||u||^2 ≧ ||(w,u)v - (v,u)w||^2.

これは有名な不等式なん?
0462132人目の素数さん
垢版 |
2020/07/02(木) 17:35:21.65ID:ceNKIuAv
A_(i,j) = v_i w_j - v_j w_i (交代テンソル)とおくと
 成分は 1≦i<j≦n をわたります。
与式は
 ||A||^2 |u|^2 ≧ |(A・u)|^2 = (スカラー三重積)^2
で、コーシーの不等式です。
3次元の場合、右辺は(u,v,wが作る平行六面体の体積)^2 です。

・幾何学的解釈
 ∠(v,w) = a, ∠(w,u) = b, ∠(u,v) = c
とおくと与式は
 1 + 2cos(a)cos(b)cos(c) - cos(a)^2 - cos(b)^2 - cos(c)^2 ≧ 0,
すなわち
 4 sin((a+b+c)/2) sin((-a+b+c)/2) sin((a-b+c)/2) sin((a+b-c)/2) ≧ 0,
なので、角{a,b,c}の三角不等式です。

・参考
{u,v,w}がなす球面三角形の面積をSとすると
 {4 cos(a/2) cos(b/2) cos(c/2) sin(S/2)}^2
 = 4 sin((a+b+c)/2) sin((-a+b+c)/2) sin((a-b+c)/2) sin((a+b-c)/2)
 = 1 - cos(a)^2 - cos(b)^2 - cos(c)^2 + 2cos(a)cos(b)cos(c)
 = |1,cos(c),cos(b)|
  |cos(c),1,cos(a)|
  |cos(b),cos(a),1|
これはヘロンの公式の球面版と考えられます。(カニョリの式)
0463132人目の素数さん
垢版 |
2020/07/02(木) 18:26:18.89ID:ceNKIuAv
(訂正)
与式は
 ||A||^2 |u|^2 ≧ |(A・u)|^2
で、コーシーの不等式です。
3次元の場合、 ||A|| はv,wが作る平行4辺形の面積です。

・・・・・

>>462 の行列式の各行に |u|, |v|, |w|, 各列に |u|, |v|, |w| を掛けて元に戻せば
|(u, u) (u, v) (u, w)|
|(v, u) (v, v) (v, w)|
|(w, u) (w, v) (w, w)|
の形(Grammian)になり、
| u'|
|v'||u, v, w|
|w'|
= (スカラー三重積)^2
= (u,v,wが作る平行六面体の体積)^2
でした。
0466132人目の素数さん
垢版 |
2020/07/18(土) 02:28:47.60ID:xiJ191gm
>>465
この手の同次数の不等式の係数というのは分母を払えば
(N個の係数1の単項式)≧(N個の係数1の単項式)
と書けるようになっていますよね?
このタイプ(特に巡回や対称の場合)の不等式に関して
どのようなとき不等式が成立するかの一般論ってあるんでしょうか

昔、考えたのは対称の場合で
指数のタイプ
(n,0,…,0)、(n-1,1,0,…,0)、…、(1,1,…,1)
に関してヤング図形的な半順序付けをしたとき
上の条件を満たすような係数になっていて、左辺の各単項に右辺の単項式への順序を下げる1対1対応があるとき不等式は成立するというものでした(これは確か証明できた)
これを使うと相加相乗などはすぐに示せます

逆に同次対称で上の係数条件を満たすような不等式はこの左右の順序を下げる1対1対応が存在するか?も考えたのですが確かこれには反例がありました
0469132人目の素数さん
垢版 |
2020/07/19(日) 19:28:26.69ID:WIR0lTu1
>>465
aa=A, bb=B, cc=C とおく。
(左辺) - (aa+bb+cc)^3
 ≧ 4(A^3 + B^3 + C^3) + 15ABC - (A+B+C)^3
 = 3{A(A-B)(A-C) + B(B-C)(B-A) + C(C-A)(C-B)}
 = 3F_1(A,B,C)
 ≧ 0,           (Schur-1)

∴ (左辺) ≧ (aa+bb+cc)^3 ≧ (1/27)(a+b+c)^6.

 (1+1+1)(aa+bb+cc) ≧ (a+b+c)^2  (コーシー) から。
0470132人目の素数さん
垢版 |
2020/07/21(火) 17:57:10.28ID:Q73vct+1
>>465
a^3 =A', b^3 =B', c^3 =C' とおく。
(左辺) = 4(a^6 + b^6 + c^6) + 5(a^5・b + b^5・c + c^5・a)
 ≧ (A'+B'+C'){(4/3)(A'+B'+C') +5abc}  (← 補題)
 = (A'+B'+C'){(1/3)(a+b+c)^3 + F_1(a,b,c)}
 ≧ (1/3)(A'+B'+C')(a+b+c)^3
 ≧ (1/3){(aa+bb+cc)(a+b+c)}^2,   (← コーシー)

〔補題〕
 a^5・b + b^5・c + c^5・a ≧ abc(a^3+b^3+c^3),
(略証)
 a^5・b + b^5・c + c^5・a
  ≧ (a^3+b^3+c^3)^2 /(a/b+b/c+c/a)  (← コーシー)
  = abc(a^3+b^3+c^3)^2 /(aac+bba+ccb)
  ≧ abc(a^3+b^3+c^3),      (← チェビシェフ)

あるいは
 (16a^5・b + b^5・c + 4c^5・a)/21 ≧ a^4・bc,  (← AM-GM)
巡回的にたす。                 (終)
0471132人目の素数さん
垢版 |
2020/07/22(水) 07:47:23.22ID:U4xy9LSi
〔演習問題1.90〕
a,b,c を非負実数とする。このとき、次を証明せよ。
 aa+bb+cc + 2abc + 1 ≧ aa+bb+cc + 3(abc)^(2/3)
  ≧ aa+bb+cc + 9abc/(a+b+c)
  ≧ 2(ab+bc+ca),

[9] 佐藤(訳), 朝倉書店 (2013) p.41
0472132人目の素数さん
垢版 |
2020/07/22(水) 07:49:09.73ID:U4xy9LSi
(略証)
左・中は AM-GM で出る。
右は通分して
 (a+b+c)(aa+bb+cc-2ab-2bc+2ca) + 9abc
 = a(a-b)(a-c) + b(b-c)(b-a) + c(c-a)(c-b)
 = F_1(a,b,c) ≧ 0,      (Schur-1)
0473132人目の素数さん
垢版 |
2020/07/22(水) 14:59:40.78ID:cBVWOvpo
a,b,c,d>0
a + (ab)^(1/2) + (abc)^(1/3) ≦ (4/3)*(a+b+c)
a + (ab)^(1/2) + (abc)^(1/3) + (abcd)^(1/4) ≦ (143/100)*(a+b+c+d)

Slovenia 2012 ( ゚∀゚) ウヒョッ!
0474132人目の素数さん
垢版 |
2020/07/23(木) 14:32:02.08ID:cdENLWJx
右辺の係数を λ_n とおくと
 λ_1 = 1.0
 λ_2 =(1+√2)/2 = 1.20710678118655 (a = (1+√2)^2・b)
 λ_3 = 4/3 = 1.333333333  (a = 4b = 16c)
 λ_4 = 1.42084438540961  (a = bp = cq = dr)
 ・・・・
[第8章.972-990]

[前スレ.041] あたりの【Kiran Kedlaya】はこれの改良版(?)
0475132人目の素数さん
垢版 |
2020/07/23(木) 15:03:09.65ID:cdENLWJx
>>470
〔補題〕
 a^{n+1}・b + b^{n+1}・c + c^{n+1}・a ≧ abc (a^{n-1} + b^{n-1} + c^{n-1}),

(略証)
 n=0 のとき等号成立
 n=1 のとき AM-GM
 n≧2 のとき
 a^{n-3} /b + b^{n-3} /c + c^{n-3} /a
 = (a^{n-2}・c + b^{n-2}・a + c^{n-2}・b) /(abc)
 ≦ (a^{n-1} + b^{n-1} + c^{n-1}) /(abc),
よって
 (左辺) ≧ (a^{n-1}+b^{n-1}+c^{n-1})^2 /(a^{n-3}/b + b^{n-3}/c + c^{n-3}/a)
                  (← コーシー)
 ≧ abc(a^{n-1}+b^{n-1}+c^{n-1}).

(別法)
 AM-GMで
 [nn・a^{n+1}・b + b^{n+1}・c + n c^{n+1}・a] /(nn+n+1) ≧ a^n・bc,
巡回的にたす。
0476132人目の素数さん
垢版 |
2020/07/23(木) 20:57:22.44ID:sHXJwhgv
異なる4つの実数が任意に与えられたとき
そこから |ab+1|>|a-b| を満たす異なる実数a,bが選べることを示せ。
0478132人目の素数さん
垢版 |
2020/07/25(土) 10:36:55.68ID:g3fpMEvS
>>476

[高校数学の質問スレPart405] から。

4つの実数をx_iとする。(i=1〜4)
θ_i = arctan(x_i) (-π/2 < θ_i < π/2) とおく。
4つから上手く2つを選ぶと
 |θi - θj| ≦π/4 または |θi-θj±π| ≦π/4,
となる。
tanの加法公式
 tan(α-β) = (tanα-tanβ)/(1+tanα・tanβ),
より
 1 ≧ |tan(θi - θj)| = |(x_i-x_j)/(1+x_i・x_j)|,
よって x_i, x_j は条件を満たす。
等号成立は {x_i} がπ/4ずつ異なるとき。 [895,948-949]

例:tan(±22.5゚) = ±(√2 -1), tan(±67.5゚) = ±(√2 +1),
0479132人目の素数さん
垢版 |
2020/08/05(水) 03:24:43.09ID:ud0wEpwG
三角形ABCの外接円の半径Rと内接円の半径rに対して、
2/R ≦ (1/a)*sec(A/2) + (1/b)*sec(B/2) + (1/c)*sec(C/2) ≦ 1/r.

[AMM, Prob12168] ( ゚∀゚)ウヒョッ!
0481132人目の素数さん
垢版 |
2020/08/11(火) 16:49:54.71ID:sLooAqcf
〔Inequalitybot 98〕
aa ≦ 1, aa+bb ≦ 5, aa+bb+cc ≦ 14, aa+bb+cc+dd ≦ 30 のとき
   a+b+c+d ≦ 10
を示せ。   Hungary-Israel binational 2007, 1日目, 問2
0482132人目の素数さん
垢版 |
2020/08/11(火) 17:00:55.87ID:sLooAqcf
コーシーにより、
 (a+b+c+d)^2 /(1+2+3+4) ≦ aa + bb/2 + cc/3 + dd/4
 = (1-1/2)aa + (1/2-1/3)(aa+bb) + (1/3-1/4)(aa+bb+cc) + (1/4)(aa+bb+cc+dd)
 ≦ (1-1/2) + (1/2-1/3)・5 + (1/3-1/4)・14 + (1/4)・30
 = 10,
等号成立は (a,b,c,d) = (1,2,3,4)
0483132人目の素数さん
垢版 |
2020/08/11(火) 22:11:26.18ID:sSrJdUio
a,b,c∈R,
a^2 + b^2 + c^2 > 0,
-1/2 ≦ Σ[cyc] ab/(a^2 + b^2 + 3c^2) ≦ 3/5.

[不明] ( ゚∀゚)ウヒョッ!
0487132人目の素数さん
垢版 |
2020/08/14(金) 14:32:03.90ID:Vqud894y
>>484
a,b,c≧0 に対し
 a^3 + b^3 + c^3 ≧ abb + bcc + caa,
を示せ。
--------------------------------------
 差積 (a-b)(b-c)(c-a) の符号は正にも負にもなるから、
このままではマズイ。
(ついでに言えば、符号も変)

0 ≦ {(a+2b)(a-b)^2 + (b+2c)(b-c)^2 + (c+2a)(c-a)^2}/3
= a^3 + b^3 + c^3 -abb -bcc -caa,

とやるか又は AM-GM で

(a^3 + 2b^3)/3 - abb = (1/3)(a+2b)(a-b)^2 ≧ 0  >>485
を循環的にたす。
0489132人目の素数さん
垢版 |
2020/08/18(火) 21:30:28.99ID:ymr8iYI5
「入試数学の純粋な難問」
0 ≦ x,y,z ≦ 1 のとき
 (x+y+z)/3 + √{x(1-x)+y(1-y)+z(1-z)} ≦ 3/2
を示せ。
--------------------------------------------------------
(x+y+z)/3 = A とおく。
x(1-x) = (3/2)(3/8 - x/3) - (x - 3/4)^2 ≦ (3/2)(3/8 - x/3),
より
x(1-x) + y(1-y) + z(1-z) ≦ (3/2)(9/8 - A)
 = (3/2 - A)^2 - (3/4 - A)^2 ≦ (3/2 - A)^2,
よって
√{x(1-x) + y(1-y) + z(1-z)} ≦ 3/2 - A,
0492132人目の素数さん
垢版 |
2020/08/26(水) 23:23:01.07ID:oXbdk8QE
a,b,c>0
$\frac{a^3+b^3}{ \sqrt{a^2-ab+b^2} } + \frac{b^3+c^3}{ \sqrt{b^2-bc+c^2} } + \frac{c^3+a^3}{ \sqrt{c^2-ca+a^2} } \geq 2(a^2+b^2+c^2)$

2020 China Norther MO ( ゚∀゚)ウヒョッ!
0496132人目の素数さん
垢版 |
2020/09/03(木) 00:51:04.79ID:PGJ1gE8Y
(a+i)(b+i)(c+i) = (abc -a-b-c) + (ab+bc+ca-1)i,
(a-i)(b-i)(c-i) = (abc -a-b-c) - (ab+bc+ca-1)i,
辺々掛ける。
0497132人目の素数さん
垢版 |
2020/09/03(木) 01:07:54.34ID:PGJ1gE8Y
>>492
a,b,c>0 のとき
 (a^3+b^3)/√(aa-ab+bb) + (b^3+c^3)/√(bb-bc+cc) + (c^3+a^3)/√(cc-ca+aa) ≧ 2(a^2 + b^2 + c^2),

(略証)
コーシーで
 (x^3+y^3)/√(xx-xy+yy) = √{(x^3+y^3)(x+y)} ≧ x^2 + y^2,
巡回的にたす。
0499132人目の素数さん
垢版 |
2020/09/03(木) 13:48:56.02ID:PGJ1gE8Y
>>495
実数でやるなら
 a+b+c = s, ab+bc+ca = t, abc = u,
とおく。
 (左辺) = (abc)^2 + ((ab)^2 + (bc)^2 + (ca)^2) + (a^2 + b^2 + c^2) + 1
 = uu + (tt-2su) + (ss-2t) + 1
 = (uu -2su +ss) + (tt -2t +1)
 = (u-s)^2 + (t-1)^2,
0500132人目の素数さん
垢版 |
2020/09/03(木) 14:48:46.83ID:PGJ1gE8Y
>>493
(上)
〔問題214〕
自然数n∈Nを固定する。
i=1,2,・・・・・,2n に対して |x_i| ≦ 1 の値をとるとき
 Σ[1≦r<s≦2n] (s-r-n) x_r x_s
の取り得る最大の値を求めよ。
 IMO Shortlist 2015 A-3
 Inequalitybot [214]


(中)
△ABCにおいて、
 F = {(sinA)^2 + 2(sinB)^2 + 3(sinC)^2} / {(sinA)(sinB)(sinC)}
とおく。
(1) △ABCの3辺の長さを BC=a, CA=b, AB=c とおき、
さらに△ABCの面積をSとする。
F を a,b,c,S で表わせ。
(2) Fの最小値を求めよ。

(下)
Problem 26
正の実数 a,b,c が ab+bc+ca=3 をみたすとき、
 a(bb+cc)/(aa+bc) + b(cc+aa)/(bb+ca) + c(aa+bb)/(cc+ab) ≧ 3
が成立することを示せ。
0501132人目の素数さん
垢版 |
2020/09/03(木) 16:26:15.63ID:PGJ1gE8Y
(中)
(1) 正弦定理
 sin(A) = a/2R, sin(B) = b/2R, sin(C) = c/2R,

 S = abc/4R,
より
 F = 2R(aa+2bb+3cc)/abc = (aa+2bb+3cc)/2S,

(2)
ところで 面積S は a,b,c の関数である。(ヘロンの公式)
 (aa+2bb+3cc)^2 - 11・16SS
 = (aa+2bb+3cc)^2 - 11{2(ab)^2 + 2(bc)^2 + 2(ca)^2 -a^4 -b^4 -c^4}
 = (3・4・5){(bb/4-cc/3)^2 + 2(cc/3-aa/5)^2 + 3(aa/5-bb/4)^2}
 ≧ 0,
 aa+2bb+3cc ≧ (4√11)S,
∴ F ≧ 2√11 = 6.63325
等号成立は a:b:c = √5:√4:√3 のとき。
0502132人目の素数さん
垢版 |
2020/09/13(日) 23:58:13.39ID:JpJgDqA9
a,b,c,d > 0
\sqrt[3]{ab} + \sqrt[3]{cd} ≦ \sqrt[3]{(a+c+d)(a+c+d)}

あばばばばばば
  ∩___∩         
  |ノ   ヽ/⌒)    あびゃば
  /⌒)(゚) (゚) /       あびゃあばばば
 / / (_●)ミ /         ∩――、
( ヽ |∪| /         /(゚)ヽ _ ヽ
 \  ヽノ /         / (● (゚) |つ
  /    /         | (入_ノ ミ
 |    /         | (_/  ノ
 | /\ \         \___ノ゙ー-、
 | /  ) )          /\    _ \
 (_ノ  ( \        (⌒O /\   (_ノ
     \_)        \ノ   /  、  )0
0504132人目の素数さん
垢版 |
2020/09/14(月) 04:14:30.54ID:MMq0bu8b
何かおかしい、何となくそんな気がした。
TVに映る試合は俺とは全く縁もゆかりもない県同士の戦いだが、負けてる方をなんとな〜く応援している気分でいると、これまたなんとなくそろそろハルヒが騒ぎ出すような気がした。
0505132人目の素数さん
垢版 |
2020/09/15(火) 07:33:01.17ID:bL5lP9LW
>>483
等号成立条件だけ。
 最小値: {a,b,c} = {-1,0,1}
 最大値: {a,b,c} = {1,1,1} {1,1,2/3}
0506132人目の素数さん
垢版 |
2020/09/15(火) 21:41:55.44ID:oug42vb/
うむ
0508132人目の素数さん
垢版 |
2020/09/21(月) 22:00:12.84ID:uwUcrYFn
>>483>>505
等号成立条件は、たぶんこうぢゃなゐかな? ( ゚∀゚)ウヒョッ!

(a,b,c,d) = (t, kt, (1+ 1/k)t, k(k+1)t), ただし k, t > 0 とする。
0510132人目の素数さん
垢版 |
2020/09/23(水) 14:21:02.58ID:qMQmLmqf
xが0以上のとき 5x^3-3x+1>0

微分法で簡単に示せるですが
不等式エキスパートの人なら巧みな多項式変形とかで示せるですか?
0511132人目の素数さん
垢版 |
2020/09/23(水) 21:07:53.46ID:63e1O9oo
x√5 = X とおけば
 5x^3 - 3x + 1 > 5x^3 - 3x + (2/√5)
 = (X^3 - 3X + 2) /√5
 = (X+2)(X-1)^2 /√5
 ≧ 0,

ただし、x=1/√5 で極小になることを
微分などの方法で知る必要がある…
0514132人目の素数さん
垢版 |
2020/09/23(水) 22:35:07.36ID:qMQmLmqf
x=0のときは明らかなのでx>0として 5x^2 + 1/x > 3 を言えばよいが
相加相乗で左辺≧3*(5/4)^(1/3) >3 。

式変形だけで、例えば
x^16 - x + 1 = (x^8-1/2)^2+(x^4-1/2)^2+(x^2-1/2)^2+(x-1/2)^2
みたいな感じの巧みな変形でいけないものでしょうか。
0515132人目の素数さん
垢版 |
2020/09/24(木) 10:06:27.00ID:qc+lGULo
>>512
 X^3 + 1 + 1 ≧ 3X,
は 相加相乗平均(AM-GM) と思ってもいいし、
コーシー
 (X^3 + 1 + 1)(1 + X^3 + 1)(1 + 1 + X^3)
 ≧ (X + X + X)^3
 = (3X)^3,
の3乗根と思ってもいい。
0516132人目の素数さん
垢版 |
2020/09/24(木) 10:21:36.43ID:3a+g1aMq
>>514-515
abc3数の相加相乗平均は、
(a+b+c)(a^2+b^2+c^2-ab-bc-ba)
= (a+b+c){(a-b)^2+(b-c)^2+(c-a)^2}
と変形できるので、a=X・b=1・c=1を代入して……って>>511と同じ式変形やないかーい笑

y=f(x) とした関数は非負だと x=1/√5 で極小値かつ最小値をとる。
極小値だけ移動した g(x)=f(x)-極小値 を考える→
x軸に接する
→(x-1/√5)^2 または (X-1)^2を因数にもつ
という考えと同じ。
0517132人目の素数さん
垢版 |
2020/09/24(木) 22:14:11.72ID:shPxNCvG
>>516
少し話が逸れるんだけど4次や5次の相加相乗にも同じように直接平方完成する変形があるんでしょうか?
0518◆iMgX3HS3iA
垢版 |
2020/09/25(金) 01:49:19.72ID:a6NySAb6
>>517
> 少し話が逸れるんだけど4次や5次の相加相乗にも同じように直接平方完成する変形があるんでしょうか?
なんと出来るらしい。
https://mathoverflow.net/questions/279969/wanted-positivity-certificate-for-the-am-gm-inequality-in-low-dimension

AMとGMの差は非負多項式で表すことが可能 https://gyazo.com/0e13cfb59b28c529dd5adbfed354bd16
具体的な恒等式(5次) https://gyazo.com/58b89593fc30a48f70ab35cee68d31e5
具体的な恒等式(2次・3次・4次)https://gyazo.com/4726111c57e1863fca1b9fcd64678b23

アドルフ・フルヴィッツによる1891年の論文
Hurwitz, A. (1891). Ueber den Vergleich des arithmetischen und des geometrischen Mittels. Journal für die reine und angewandte Mathematik, 108, 266-268. https://link.springer.com/chapter/10.1007/978-3-0348-4160-3_35
nが奇数の場合を示したらしい: 藤原和将・小澤徹(応用物理)による2014年の論文 Fujiwara, Kazumasa, and Tohru Ozawa. Identities for the Difference between the Arithmetic and Geometric Means, (2014).
http://m-hikari.com/ijma/ijma-2014/ijma-29-32-2014/ozawaIJMA29-32-2014.pdf
nが偶数の場合を示しているらしい: ハーディ・リトルウッド・ポリア『不等式』第2章

なおこの問題は、ヒルベルトの第17問題(いつも正の有理式は平方和で表せる)の特殊な場合でもある。

最初に見付けたのページは医師でアマチュア数学者の佐藤郁郎によるコラムだったが、いかんせん読みにくく参考までに。(NGなのでURL貼らない)
因数分解の算法(その11)
因数分解の算法(その14)
因数分解の算法(その18)
0519132人目の素数さん
垢版 |
2020/09/25(金) 02:15:38.40ID:Bm3x9keW
>>518
めっちゃ詳しくありがとうございます!
まさか本当に出来るとは驚きです
第17問題のwikiを読んだ感じでは非負な斉次多項式に対して一般にこういうことは出来ないようですね
この不可能性はモデル理論的な話があるようでこれも面白そうです
0521132人目の素数さん
垢版 |
2020/09/25(金) 13:45:47.22ID:C/C9yJEj
(x_1)^n, ・・・・・, (x_n)^n の相加平均をA, 相乗平均をG,
兩n = n(A^n - G^n) = Σ x^n - nΠ x,
とおく。
兩2(a,b) = aa +bb -2ab = (a-b)^2,

兩3(a,b,c) = a^3 +b^3 +c^3 - 3abc
 = (a+b+c){(a-b)^2 + (b-c)^2 + (c-a)^2}/2,

兩4(a,b,c,d) = a^4 +b^4 +c^4 +d^4 - 4abcd
 = (aa-bb)^2 + (cc-dd)^2 + 2(ab-cd)^2
 = (aa-cc)^2 + (bb-dd)^2 + 2(ac-bd)^2
 = (aa-dd)^2 + (bb-cc)^2 + 2(ad-bc)^2,

兩5(a,b,c,d,e)
 = a^5 + b^5 + c^5 + d^5 + e^5 -5abcde
 = (a-b)(a^4 -b^4)/4 + (a-c)(a^4 -c^4)/4 +
 ・・・・・ + (d-e)(d^4 -e^4)/4
 + a兩4(b,c,d,e)/4
 + b兩4(c,d,e,a)/4
 + c兩4(d,e,a,b)/4
 + d兩4(e,a,b,c)/4
 + e兩4(a,b,c,d)/4,

* (x-y)(x^4-y^4) = (x+y)(xx+yy)(x-y)^2 ≧ 0,
0522132人目の素数さん
垢版 |
2020/09/26(土) 22:14:27.50ID:lnmePYpg
>>521
なるほど、5次の場合は
1/4(x+y)(x^2+y^2)(x-y)^2
=1/6(x^3+y^3)(x-y)^2+1/12(x+y)(x^2-y^2)^2
を利用すると>>518の藤原小澤の表示と一致するのか
藤原小澤の論文は流し読みしたけどテクすぎて全然わからん
表示の仕方の自由度高そうだし何か行列式的な表示とか対称式の空間上の作用素みたいなのを見つけて綺麗に示せないもんかね

>>519
訂正
てっきり不可能性がモデル理論的に分かると思っていたけど今日モデル理論の本借りて見てみたら肯定的な証明が書かれていた
17問題が肯定的なのか否定的なのか混乱してきた…
0523132人目の素数さん
垢版 |
2020/09/26(土) 22:49:49.44ID:yIQAC3t7
元の十七問題はΣ多項式^2でかけるか?でそれは誰かの反例が出た
後に実閉体まで話広げるとΣ実閉包の元^2でかける事が証明された(Artin)
永田先生の可換体論の5章に証明がある
0524132人目の素数さん
垢版 |
2020/09/26(土) 23:07:14.49ID:lnmePYpg
自分の読んでる「幾何学的モデル理論入門」(最近、改訂版が出たばかりらしい)に実閉体の第17問題が肯定的に解けることを利用して有理数体の場合も証明できるかのように書いてるように見えるんです…
0528132人目の素数さん
垢版 |
2020/09/26(土) 23:49:36.06ID:yIQAC3t7
>>526
thx
確か上がってる反例はΣ整式^2では表せない例でΣ有理式^2では表せるんだったかな?
Artinの定理の正確なステートメント覚えてない(かつ永田先生の本が現在部屋のどこにあるかわからん)のでわかんないけどΣ有理式^2で表すのは有理係数でもいけるのかもしれない

 任意の実閉包の中で0以上 →Σ有理式^2で表示できる

だったかも
確か右が言えてない場合に標数0の加法的付値体て0未満になる構造が存在する事示してそれを上手いこと微調整して通常のRの中で0以下に出来る事を示すんだったような
0529132人目の素数さん
垢版 |
2020/09/26(土) 23:56:47.06ID:lnmePYpg
あー、有理式なら肯定的という可能性があるんですね
ありがとうございます
0531132人目の素数さん
垢版 |
2020/09/27(日) 00:49:47.04ID:KH9c7ePZ
あ、全ての有理数に対して非負な多項式で考えるから奇数次の場合は全ての変数を平方にしておかないといけなくて藤原小澤の結果の変数を平方にするだけか
0532132人目の素数さん
垢版 |
2020/09/27(日) 01:09:34.88ID:KH9c7ePZ
いや、そうなってくると藤原小澤も必要なくて古典的なフルヴィッツの形で示せてるのか
無知すぎてスマン
0533132人目の素数さん
垢版 |
2020/09/27(日) 01:15:18.97ID:T1BBTchP
有理式使っていいならAM-GMは簡単でしょ?
いわゆる2冪でやっといて減らす作戦でいける
0534132人目の素数さん
垢版 |
2020/09/27(日) 14:36:59.49ID:N42SrDUa
>>521
兩n(a_1, a_2, ・・・・, a_n)
 = (a_1)^2 + (a_2)^n + ・・・・ + (a_n)^n - n(a_1)(a_2)・・・・(a_n)
 = Σ[i<j] (a_i - a_j)[(a_i)^{n-1} - (a_j)^{n-1}] /(n-1)
 + Σ[i=1,n] a_i (Σ[j≠i] (a_j)^{n-1} - (n-1)Π[j≠i] a_j) /(n-1)
 = Σ[i<j] (a_i - a_j)[(a_i)^{n-1} - (a_j)^{n-1}] /(n-1)
 + Σ[i=1,n] a_i 兩{n-1}(i以外) /(n-1),
0538132人目の素数さん
垢版 |
2020/09/28(月) 01:14:12.03ID:VdFe70Zi
>>535
数列 {a_n}n∈N と {b_n}n∈N が |a_n|≦1, |b_n|≦1 (∀n∈N) を満たす時、
次を示せ。
| Π[i=1,n] a_i - Π[k=1,n] b_k | ≦ Σ[j=1,n] |a_j - b_j| (∀n∈N)

(略証)
Π[i=1,n] a_i - Π[k=1,n] b_k
 = Σ[j=1,n] (Π[i=1,j-1] a_i) (a_j - b_j) (Π[k=j+1,n] b_k),
三角不等式により
(左辺) ≦ Σ[j=1,n] (Π[i=1,j-1] |a_i|) |a_j - b_j| (Π[k=j+1,n] |b_k|)
 ≦ Σ[j=1,n] |a_j - b_j|
 = (右辺),
0539132人目の素数さん
垢版 |
2020/10/06(火) 20:34:14.40ID:CqXEEU8P
〔問題944〕
a,b,c は相異なる正の数で、√a + √b + √c = 1 を満たす。
 f(x,y) = log(y/x) / (1/x - 1/y),
に対して、
 f(a,b) + f(b,c) + f(c,a) ≦ 1/3
を示せ。

高校数学の質問スレPart407 - 944
0541132人目の素数さん
垢版 |
2020/10/13(火) 14:51:16.00ID:Aceyovpj
>>539
0<x,y, x≠y のとき
f(x,y) = log(y/x)/(1/x - 1/y)
 = log(y/x)/(√(y/x) - √(x/y))
 = 2t/(e^t - e^{-t})・√(xy)
 = t/sinh(t)・√(xy)
 ≦ √(xy),
 等号成立は x=y のとき。

(左辺) = f(a,b) + f(b,c) + f(c,a)
 ≦ √(ab) + √(bc) + √(ca)
 ≦ (1/3)(a+b+c + 2√(ab) + 2√(bc) + 2√(ca))
 = (1/3)(√a + √b + √c)^2,
 等号成立は a=b=c のとき。
0544132人目の素数さん
垢版 |
2020/10/14(水) 04:12:15.55ID:PHtzabu1
JP346.
 両辺に ab(a+b) >0 を掛けて通分すると
ab(a+b)(左辺 - 右辺) = (a-b)^2 {(a-b)^2 + (4 - k/4)ab},
 (4 - k/4) ≧ 0,
 k ≦ 16,
0546132人目の素数さん
垢版 |
2020/10/14(水) 20:10:00.68ID:PHtzabu1
JP347.
基本対称式を
 s = a+b+c, t = ab+bc+ca, u = abc,
とおく。
(a+b)(b+c)(c+a) = st-u を掛けて通分すると
2(a+b)^2・(a+c)^2 + 2(b+c)^2・(b+a)^2 + 2(c+a)^2・(c+b)^2
 = 2(ss-t)^2 + 8su
 = s^4 + (5/3)tt + s(s^3 -4st+9u) + (tt-3su)/3
 ≧ s^4 + (5/3)tt,
∴ (左辺) ≧ {s^4 + (5/3)tt}/st = (s^3)/t + 5t/(3s),

JP348.
 a/b=x, b/c=y, c/a=z とおくと
 x^3 + y^3 + z^3 ≧ 3xyz = 3,   (← AM-GM)
 (x^4+y^4+z^4)(1+1+1) ≧ (x^2+y^2+z^2)^2, (←コーシー)
辺々掛ける。
0547132人目の素数さん
垢版 |
2020/10/14(水) 22:07:28.25ID:OfAfCbWz
>>545
346はkについての一次式だから
与式⇔k≦4次式÷4次式
になる(割る時、除数の符号わそこまで難しくない)
分子も分母も(a-b)^2で割り切れる
0548132人目の素数さん
垢版 |
2020/10/15(木) 17:59:01.64ID:2j40wcqC
JP350.
 (a+b+c)^2 ≦ 3(aa+bb+cc),
x ≧ 1/√3 のとき
{1 + 3x(1-x)}^2 - (4-3xx) = 3(3xx-1)(x-1)^2 ≧ 0,
√(4-3xx) + 3x(x-1) ≦ 1,
x=a,b,c でたす。

JP351.
ABCが鈍角凾フときは
 Πcos(・) ≦ 0,
 (左辺) ≧ 0 ≧ (右辺),
で成立するから以下では、ABCは鋭角Δとする。
 sin(A) = x, sin(B) = y, sin(C) = z
とおくと
 xyz > 0,
一方、題意より
 xx+yy+zz = 1 - 2xyz,
 xyz ≦ 1/8,
左辺に加法公式
 sin(A)sin(B) = cos(A)cos(B) - cos(A+B) = xy + z,
を入れれば
 (左辺) - (右辺) = (xy+z)(yz+x)(zx+y) - 4xyz + 5(xyz)^2
 = ・・・・,
 がんばれ

JP352.
△不等式で
 |a+b+c| + |a-b| + |a-c| ≧ |(a+b+c) + (a-b) + (a-c)| = 3|a|,
巡回的にたすと
 3|a+b+c| + 2(|a-b|+|b-c|+|c-a|) ≧ 3(|a|+|b|+|c|),
0549132人目の素数さん
垢版 |
2020/10/15(木) 19:59:49.04ID:2j40wcqC
UP346
 (左辺) = 1 + (x-1)exp(arctan(x))√(1+xx),  ←可積分
∴ x=1

UP347.
上の式は
0 = |x|^2 /m + |y|^2 /n - |x+y|^2 /(m+n)
 = xx' /m + yy' /n - (x+y)(x+y)' /(m+n),
mn(m+n) を掛けて通分する。
0 = nnxx' + mmyy' - mn(xy'+x'y)
 = (nx - my) (nx - my)'
 = |nx - my|^2
∴ x = (m/n)y,
これを下の式に入れて
 y = 1+2i,

UP534.
 (1/n)Σ[k=1,n-1] (k/n)sin(kπ/n)
 → ∫[0,1] x・sin(πx) dx   (n→∞)
 = [ sin(πx)/π^2 - x・cos(πx)/π ](x=0,1)
 = 1/π,

(与式) ≒ (1 + π/n)^n
 = {(1+π/n)^(n/π)}^π
 → e^π,  (n→∞)

UP359.
Ω = ∫[0,π/2] θ・cosθ/[(sinθ)^3 + (cosθ)^3] dθ
 = (π/36){π√3 + log(97/8 + 7√3)}
  - (1/144){ψ'(5/12) - ψ'(11/12)}
 = 0.71907287245537291248414214
ここに
ψ'(x) = {log(Γ(x))}" = {Γ'(x)/Γ(x)}'
 = Σ[k=0,∞] 1/(x+k)^2  … トリガンマ函数
0550132人目の素数さん
垢版 |
2020/10/16(金) 05:20:43.69ID:QJC/WS82
JP351.
 cos(A) = x, cos(B) = y, cos(C) = z
とおくと、
に修正…

SP349.
 s = -log(sin(a)) >0,  c = -log(cos(a)) >0,
とおく。
(左辺) = (e^{-s})^{√(c/s)} + (e^{-c})^{√(s/c)}
 = 2e^{-√(cs)}
 ≦ 2e^{-log(2)/2}    (*)
 = 2e^{log(1/√2)}
 = √2,
(*) a が端(0,π/2) に近づくとき cs は急に大きくなる。

SP351.
 (1+t)^{1/3} + (1-t)^{1/3}
 = 2/[(1+t)^{2/3} - ((1+t)(1-t))^{1/3} + (1-t)^{2/3})
 ≦ 2,

UP358.
|∫[a,b] e^{ix}/x dx | ≦ ∫[a,b] 1/x dx = log(b/a),
両辺を2乗する。

UP360.
 x = y = 1/3,
0552132人目の素数さん
垢版 |
2020/10/16(金) 19:37:27.70ID:QJC/WS82
SP353.
与式を辺々引くと
 λ(x-y) = √(λλyy-1) - √(λλxx-1)
  = λλ(yy-xx)/{√(λλyy-1) + √(λλxx-1)},
もし x-y≠0 ならば
 1 = - λ(x+y)/{√(λλyy-1) + √(λλxx-1)}
 < 0, (矛盾)
∴ x = y = z = 2/(λ√3),
0553132人目の素数さん
垢版 |
2020/10/16(金) 20:49:16.21ID:QJC/WS82
SP353. (別法)
t≧1 で f(t) = t + √(tt-1) は単調増加
与式より
 f(λx) = f(λy) = f(λz),
 λx = λy = λz,
λ>0 より
 x = y = z,
0554132人目の素数さん
垢版 |
2020/10/17(土) 13:08:39.01ID:Ml1qOBSK
UP346.
 f(x) e^{arctan(x)} = ∫ (2xx) e^{arctan(x)} / √(1+xx) dx
とおく。
 f '(x) + f(x)/(1+xx) = 2xx/√(1+xx),
 f '(x)√(1+xx) + f(x)/√(1+xx) = 2xx,
ここで
 f(x) = g(x)√(1+xx),
とおくと
 (1+xx)g '(x) + (1+x)g(x) = 2xx,
g(x) が n次多項式とすると
 g(x) = ax^n + ・・・
(左辺)= (n+1)ax^{n+1} + …
∴ n=1,
 (左辺) = 2ax^2 + g(1)(1+x),
 a=1, g(1)=0,
 g(x) = x-1,
 f(x) = (x-1)√(1+xx),
0555132人目の素数さん
垢版 |
2020/10/18(日) 13:13:01.21ID:ZEBeZlNg
SP354.
 log(x^{xy}・y^{yz}・z^{zx})
 = log(x^{xy}) + log(y^{yz}) + log(z^{zx})
 = y・log(x^x) + z・log(y^y) + x・log(z^z)
 ≦ y・(x^x - 1) + z・(y^y - 1) + x・(z^z - 1)
 = (y・x^x + z・y^y + x・z^z) - (x+y+z),

*) e^t ≧ 1+t より log(u) ≦ u-1,
0556132人目の素数さん
垢版 |
2020/10/18(日) 19:36:39.48ID:ZEBeZlNg
SP358.
コーシーで
 {(y+1)+(z+1)+(x+1)} {(z+1)(x+1)+(y+1)}{x^3/[(y+1)(z+1)] + cyclic}
 ≧ (x+y+z)^3
 = s^3
よって
 (左辺) ≧ 4s^3 /(s+3)^2 + 3
 = s{(2s/(s+3))^2 + (s+3)/2s + (s+3)/2s - 1}
 = s(3-1)       (← AM-GM)
 = 2s
 ≧ (右辺),
等号は s=3, x=y=z=1 のとき。
0557132人目の素数さん
垢版 |
2020/10/18(日) 21:26:39.70ID:ZEBeZlNg
SP358.
コーシーで
 { …… } {(z+1)+(x+1) + (y+1)}{ …… }
 ≧ (x+y+z)^3

 ≧ s(3-1)       (← AM-GM)

JP360.
 tan(x)^2/{tan(x)^3+cot(x)} + cot(x)^2/{cot(x)^3+tan(x)}
  - 2/{tan(x)^2 +cot(x)^2}
 = Σ {tan(x) + cot(x) -2}/{tan(x)^2 + cot(x)^2}
 = X / (XX+4X+2)
 ≦ 1/(4+2√2),           (*)
ここに X = tan(x) + cot(x) -2 ≧ 0,
∴ 0 ≦ (左辺) - (右辺) ≦ 3/(4+2√2),

*) (XX+4X+2) - (4+2√2)X = (X-√2)^2 ≧ 0,
等号成立は X = √2, sin(2x) = 2 - √2,
0559132人目の素数さん
垢版 |
2020/10/20(火) 20:12:27.86ID:J8I4fsGY
〔問題558〕
正の実数 x,y,z が xyz=1 を満たすとき、以下を示せ。
x/(1+y+z)^3 + y/(1+z+x)^3 + z/(1+x+y)^3 ≧ 1/9 ≧ 1/(√xy + √yz + √zx)^2,
0562132人目の素数さん
垢版 |
2020/12/03(木) 06:23:28.55ID:qlrP4DQI
a,b,c > 1/2 のときに、aa+bb+cc+ab+bc+ca-a-b-c ≧0 を証明したい。
左辺を平方完成して、残り物 ab+bc+ca-3/4 にAM-GMする以外にハァハァできそうな方法はないかな?
0563132人目の素数さん
垢版 |
2020/12/03(木) 06:24:46.82ID:qlrP4DQI
>>558
左辺の分母の1を(abc)^(1/3)に変えて同次にするんだろうと思うけど、そこで手が止まっている…
0564132人目の素数さん
垢版 |
2020/12/06(日) 02:09:47.83ID:KT/cOuDT
>>562
 (左辺) = {(a+b)^2 + (b+c)^2 + (c+a)^2}/2 - (a+b+c)
   = {(a+b)(a+b-1) + (b+c)(b+c-1) + (c+a)(c+a-1)}/2
  > 0,
とか
 (左辺) = {4(a+b+c)^2 + (a-b)^2 + (b-c)^2 + (c-a)^2}/6 - (a+b+c)
 ≧ (2/3)ss - s       (s=a+b+c)
 = (2/3)s(s - 3/2)
 > 0,
とか色々あるけど、単に
 a' = a - 1/2 > 0, b' = b - 1/2 > 0, c' = c - 1/2 > 0,
でいい希ガス…
0566132人目の素数さん
垢版 |
2020/12/14(月) 01:35:46.45ID:gLda82Cm
a,b>0は定数とする. 0<s,t<1のとき
st/(as+bt)+(1-s)(1-t)/{a(1-s)+b(1-t)}の最大値を求めよ.
0567132人目の素数さん
垢版 |
2020/12/18(金) 06:19:31.83ID:DAoaiwdi
1/(a+b) - (与式) = ab(s-t)^2 /[(a+b)(as+bt){a(1-s)+b(1-t}] ≧ 0,
等号成立は s=t のとき。
0568132人目の素数さん
垢版 |
2020/12/18(金) 13:22:00.83ID:DAoaiwdi
コーシーで
 st/(as+bt) ≦ (at+bs)/(a+b)^2,
 (1-s)(1-t)/{a(1-s)+b(1-t)} ≦ {a(1-t)+b(1-s)}/(a+b)^2,
辺々たす。
0569132人目の素数さん
垢版 |
2020/12/19(土) 21:22:13.58ID:iw6DTiTj
単位円に内接する正n角形のn個の頂点からの距離の和が最小になる点とその最小値を求めよ。
0570132人目の素数さん
垢版 |
2020/12/20(日) 20:55:16.29ID:QYPKWpxY
頂点A_k の極座標を (1, 2kπ/n) 点Pの極座標を (r, θ) とおく。
第二余弦定理より
PA_k = √{1 - 2r・cos(2kπ/n - θ) + rr} ≧ 1 - r・cos(2kπ/n - θ)
 等号成立は r=0 のとき
また
 Σ[k=1,n] cos(2kπ/n - θ) = Σ[k=1,n] {sin((2k+1)π/n - θ) - sin((2k-1)π/n - θ)}/{2sin(π/n)}
 = {sin((2n+1)π/n - θ) - sin(π/n - θ)}/{2sin(π/n)}
 = 0,
∴ Σ[k=1,n] PA_k ≧ n,
 等号成立は P=O のとき。

・nが偶数のとき (n=2m)
三角不等式より
 PA_k + PA_{m+k} ≧ A_k A_{m+k} = 2,
 等号成立は P が線分 A_k A_{m+k} 上にあるとき。(← 円の直径)
∴  Σ[k=1,n] PA_k = Σ[k=1,m] (PA_k + PA_{m+k}) ≧ Σ[k=1,m] 2 = 2m = n,
 等号成立は P=O のとき。
0571132人目の素数さん
垢版 |
2020/12/20(日) 23:01:24.01ID:QYPKWpxY
ヴェクトルの内積を使えば
OA_k = 1 より

PA_k ≧ ↑PA_k・↑OA_k = (↑OA_k - ↑OP)・↑OA_k
  = 1 - ↑OP・↑OA_k,

∴ Σ[k=1,n] PA_k ≧ n - ↑OP・{Σ[k=1,n] ↑OA_k} = n,
等号成立は ↑OP = o.
0572132人目の素数さん
垢版 |
2021/01/01(金) 08:32:42.45ID:NURKUP5N
      ∧_∧
     ( ´Д` )  新年あけまして
     /     ヽ
     し、__X__,ノJ

      /´⌒⌒ヽ
    l⌒    ⌒l  おめでとうございます。
   ⊂ (   ) ⊃
      V ̄V

正の数 a,b,c に対して
(a^2021 -a^3 +3)(b^2021 -b^3 +3)(c^2021 -c^3 +3) > (a+b+c)e,
e = 2.71828… は自然対数の底
>>294
0574132人目の素数さん
垢版 |
2021/01/03(日) 06:38:55.14ID:N51mYuOL
eが出てきても eじゃない…

(左辺) ≧ 2.7199579587(a+b+c)
等号は a=b=c = 0.9968783547581 のとき。
0575132人目の素数さん
垢版 |
2021/01/03(日) 16:12:00.62ID:+lbXmv47
すべての自然数nについて
Σ_{k=1}^n (k^(1/2)-1)≧(n/2)*((n/2)^(1/2)-1)
が成り立つことをしめせ
0576132人目の素数さん
垢版 |
2021/01/03(日) 20:22:26.62ID:N51mYuOL
k≧2 について
 (k-1)^3 - k(k-3/2)^2 = (3/4)k - 1 > 0,  (AM-GM)
 (k-1)^{3/2} > (k - 3/2)√k = k^{3/2} - (3/2)√k,
 √k > (2/3)(k^{3/2} - (k-1)^{3/2}),
(左辺) = Σ_{k=2}^{n} (√k - 1)
  > (2/3)(n^{3/2} - 1) - (n-1)
  = (1/3)(2√n +1)(√n - 1)^2
  > (n/2)(√(n/2) - 1),
0577132人目の素数さん
垢版 |
2021/01/03(日) 22:48:34.64ID:N51mYuOL
積分を使えば簡単だが…
√k > ∫_{k-1}^{k} (√x)dx = (2/3)(k^{3/2} - (k-1)^{3/2}),
0578132人目の素数さん
垢版 |
2021/01/04(月) 20:59:39.27ID:OQ8TTvGy
最後は
 (左辺) > n((2/3)√n - 1)
  > (n/2)(√(n/2) + √(n/3) - 2)  (← 補題)
  ≧ (n/2)(√(n/2) - 1)     (n≧3)
 n=1, n=2 は明らか。

〔補題〕
 1/√2 + 1/√3 < 4/3.
(略証)
コーシーで
 (1/√2 + 1/√3)^2 ≦ (1+1)(1/2+1/3) = 5/3 < 16/9 = (4/3)^2,
0580132人目の素数さん
垢版 |
2021/01/06(水) 18:50:24.31ID:ZXZ11nuc
n が被ってしまった。。。。m 等にすべきか。
>>577 から
 Σ_{k=2}^{n} √k > √2 + ∫_[2}^{n} (√x) dx
  = √2 + (2/3)(n√n - 2√2)
  > (2/3)n√n - 1/2,        (n≧2)
0584132人目の素数さん
垢版 |
2021/01/10(日) 02:37:24.49ID:k4Y9uhcW
〔問題〕
A, B, and C are non-negative real numbers. Prove that
3(A^4 + B^4 + C^4) ≧ (A^2 + B^2 + C^2)(AB + BC + CA) + (A^2 - B^2)^2 + (B^2 - C^2)^2 + (C^2 - A^2)^2
≧ (A^2 + B^2 + C^2)^2,

BMO-2016 Azerbaijan (改)
[数学オリンピック31.018]
0586132人目の素数さん
垢版 |
2021/01/27(水) 16:42:45.22ID:knjIwEAx
〔問題〕
凾フ各辺の長さを a,b,c とするとき、外接円の半径Rは
 (1/3)√(aa+bb+cc) ≦ R
  ≦ (1/(6√3)){a(b+c)/(b+c-a) + b(c+a)/(c+a-b) + c(a+b)/(a+b-c)},

佐藤淳郎(訳)「美しい不等式の世界」朝倉書店 (2013)
 (左) Leibnizの不等式 (定理2.4.5) p.88-89
 (右) 演習問題 2.57(改) p.94
0589132人目の素数さん
垢版 |
2021/02/16(火) 19:11:23.08ID:+p1DAQAM
底面を凸四角形ABCDとする四角錐V-ABCDについて、
すべての辺に接し、かつ中心が底面上にあるような球が存在するとする。
このとき、
 VA+VB+VC+VD ≦ AB+BC+CD+DA

[大数2021−2月宿題]
0590132人目の素数さん
垢版 |
2021/02/16(火) 20:08:40.42ID:ItHwm5kd
球の半径を1とし、球の中心をO、∠OAB、∠OBC、∠OCD、∠ODAをa,b,c,dとすれば
AB+BC+CD+DA=2(cot a+cot b +cot c + cot d)、
VA+VB+VC+VD=cot a + tan a +cot b + tan b +cot c + tan c +cot d + tan d
∴ AB+BC+CD+DA-(VA+VB+VC+VD)
=cot a - tan a +cot b - tan b +cot c - tan c +cot d -tan d
以下a+b+c+d=π/2とcot x-tan xの凸性
0594132人目の素数さん
垢版 |
2021/02/17(水) 00:22:20.07ID:FhoAKrRQ
>>590
三行目では, VA=cot a + tan a になるってことですか?
tan a の出所がわからないです。
0596132人目の素数さん
垢版 |
2021/02/18(木) 15:24:16.44ID:gfQsTfgF
内接円をもつ四角形ABCDで、内接円の中心を I とするとき

(IA+ID)^2+(IB+IC)^2 ≦ (AB+CD)^2
0597132人目の素数さん
垢版 |
2021/02/23(火) 15:53:16.27ID:mfVhACbJ
>>593
cos(π) = - 1,
cos(2π/3) = - 1/2,
cos(2π/5) = (√5 -1)/4 = 1/(2φ) = 0.309017
cos(2π/7) = 0.6234898
Π[p≧11] cos(2π/p) > Σ[p≧11] (1 - 2ππ/p^2)
  > 1 - Σ[p≧11] 2ππ/p^2
  > 1 - (2ππ)Σ[k≧5] 1/(2k+1)^2
  = 1 - (2ππ)(ππ/8 - 1 - 1/9 - 1/25 - 1/49 - 1/81)
  = 1 - (2ππ)・0.0498356
  = 0.01628475
∴ (与式) > 0

なお
 Π[p≧7] cos(2π/p) = 0.3338
 Π[p≧11] cos(2π/p) = 0.5354
(与式) ≒ 0.05158
0598132人目の素数さん
垢版 |
2021/02/23(火) 16:29:58.81ID:mfVhACbJ
Π[p≧11] cos(2π/p) > Π[p≧11] cos(2π/(p-1))
 > Π[n=5,∞] cos(π/n)
 = (2√2)Π[n=3,∞] cos(π/n)
 = (2√2)・0.114942        (*)
 = 0.3251052

*) 数セミ増刊「数学の問題」第2集, 日本評論社 (1978)
 ●117 によれば
 Π[n=3,∞] cos(π/n) = 0.114942044853296…
0599132人目の素数さん
垢版 |
2021/03/03(水) 14:15:42.03ID:SY070HAY
(2)
 |cosθ| ≦ 1/2  ⇔  cos(2θ) ≦ -1/2,
 |cosθ| ≧ 1/2  ⇔  cos(2θ) ≧ -1/2,

(4)
 |cosθ| ≦ cos(72)  ⇔  cos(4θ) ≧ cos(72),
 |cosθ| ≧ cos(72)  ⇔  cos(4θ) ≦ cos(72),

[分かスレ466-119]
0600132人目の素数さん
垢版 |
2021/03/03(水) 14:37:41.92ID:SY070HAY
(2)
 cos(2θ) + 1/2 = 2 [(cosθ)^2 - 1/4],

(4)
 cos(4θ) - cos(72) = 8 [cos(18)^2 - (cosθ)^2] [cos(72)^2 - (cosθ)^2]

∴ |cosθ| ≦ cos(72)  ⇒  cos(4θ) ≧ cos(72),

[分かスレ466-129]
0601132人目の素数さん
垢版 |
2021/03/06(土) 19:39:49.08ID:dHW5XVEt
(4)
 |cosθ| ≦ cos(72)  ⇒  cos(4θ) ≧ cos(72),
 |cosθ| ≧ cos(72)  ←  cos(4θ) ≦ cos(72),
0602132人目の素数さん
垢版 |
2021/03/06(土) 19:43:07.41ID:dHW5XVEt
〔問題157〕
x>0, y>0, z>0 ならば
 (x+y)^z + (y+z)^x + (z+x)^y >2,

[分かスレ466-157, 178]
0603132人目の素数さん
垢版 |
2021/03/06(土) 19:53:41.41ID:VMjWPceO
>>178
そのわかすれの178のレスでx+y,y+z,z+xのうち1以上のものが少なくとも一つあるとしてるけど、それらのケースに帰着できるわけじゃないよね?
最小値はx=y=z=0.184付近だし
単にすぐに除外していいケース述べてるだけだよね?
0604132人目の素数さん
垢版 |
2021/03/07(日) 06:25:23.53ID:gfZuqlK8
全部1未満のとき
 0 < x, y, z < 1.
 f(z) = a^(1-z) (a>0) は下に凸だから
 f(z) < f(0)(1-z) + f(1)z,  (0<z<1)
 a^(1-z) < a(1-z) + z < a+z,
 a^z > a/(a+z),   … ベルヌーイの不等式
∴ (x+y)^z > (x+y)/(x+y+z),
 巡回的にたす。
0606132人目の素数さん
垢版 |
2021/03/10(水) 06:18:44.76ID:dMP4wwTf
〔問題596〕
正の実数 a,b,c が a+b+c = 1 を満たすとき
 (1/a - a)(1/b - b)(1/c - c) ≧ (3 - 1/3)^3,
 等号成立は a=b=c = 1/3.
を示せ。

[高校数学の質問スレ410-596,599,610]
0607132人目の素数さん
垢版 |
2021/03/10(水) 22:08:55.30ID:NWVdDcAf
「任意の自然数nに対して、n<2^nが成立」
これを色々な方法で証明せよ

よくある証明方法は帰納法、二項定理の利用、微分によるなどあるが、それ以外もあるんかな
0608132人目の素数さん
垢版 |
2021/03/11(木) 06:14:13.22ID:JY2ui+vd
帰納法
 2^n = 2^{n-1} + 2^{n-1} > (n-1) + 1 = n,
あるいは
 2^n = 2^{n-1} + 2^{n-2} + ・・・・ + 2 + 1 + 1 ≧ n + 1,
           (n+1)項

a_1, a_2, ・・・・, a_n ≧ 0 のとき
(1+a_1)(1+a_2)・・・・(1+a_n) = 1 + s_1 + ・・・・ + s_n ≧ 1 + s_1,
  s_k は k次の基本対称式
  s_1 = a_1 + a_2 + ・・・・ + a_n,
より
 2^n ≧ 1 + n
0609132人目の素数さん
垢版 |
2021/03/11(木) 12:04:01.19ID:qBeEcW7U
俺が考えていた証明

n(1/n-1/2^n)=1-n/2^n
=(Σ1/2^i)-n/2^n
>(Σ1/2^i)-(1/2+…+1/2^n)>0
よりn>0だから
1/n-1/2^n>0⇔n<2^n
0612132人目の素数さん
垢版 |
2021/03/17(水) 08:27:58.15ID:Rkkg81B/
>>606
a+b+c = 1 より
 G = (abc)^{1/3} ≦ 1/3,     (AM-GM)

 1/y - y = (1+y)・(1/y - 1),
より
(1/a - 1)(1/b - 1)(1/c - 1) = (1-a -b -c)/(abc) + (1/a + 1/b + 1/c) - 1
 = 1/a + 1/b + 1/c - 1
 ≧ 3/G - 1
 ≧ 2(1/G + 1)   (G≦1/3)
 = 2(1/3G + 1/3G + 1/3G + 1)
 ≧ (2/(3G)^{1/4})^3,

(1+a)(1+b)(1+c) ≧ (1+G)^3  (コーシー)
 = (1/27)(3G+1+1+1)(1+3G+1+1)(1+1+3G+1)
 ≧ ((4/3)(3G)^{1/4})^3,

辺々掛けて (左辺) ≧ (8/3)^3.
0613132人目の素数さん
垢版 |
2021/03/17(水) 08:34:06.62ID:Rkkg81B/
〔問題3204〕
a≧b≧c≧d≧0 のとき
 (a+2b) (aa+bb) ≦ (a+b)^3
 (a+2b+3c) (aa+bb+cc) ≦ (a+b+c)^3,
 (a+2b+3c+4d) (aa+bb+cc+dd) ≦ (a+b+c+d)^3,

注) 5文字の場合は aa(b-d-2e) が出て来ます…orz

すうじあむ
 http://suseum.jp/gq/question/3204
0615132人目の素数さん
垢版 |
2021/04/08(木) 19:31:57.99ID:jAHOCp/v
〔例2.4.6〕
三角形の辺の長さを a,b,c, 面積を凾ニすると
  ≦ (3/4)abc/√(aa+bb+cc),

佐藤(訳), 文献9, 朝倉書店 (2013)  p.89
0616132人目の素数さん
垢版 |
2021/04/08(木) 20:11:29.99ID:jAHOCp/v
(略証)
  = (1/4)√{4(aabb+bbcc+ccaa) - (aa+bb+cc)^2}  (Heron)
  = (1/4)√{4(xy+yz+zx) - (x+y+z)^2}
  ≦ (1/4)√{9xyz/(x+y+z)}      (Schur-1)
  = (3/4)abc/√(aa+bb+cc),

* Schur-1
 F_1(x,y,z) = (x+y+z)^3 - 4(x+y+z)(xy+yz+zx) + 9xyz
  = x(x-y)(x-z) + y(y-z)(y-x) + z(z-x)(z-y) ≧ 0,
0618132人目の素数さん
垢版 |
2021/04/25(日) 04:42:31.91ID:We8cr6tt
〔問題〕
正の実数 a,b,c について、次が成り立つことを示せ。
 {aa(b+c)+4}/(a+2)^3 + {bb(c+a)+4}/(b+2)^3 + {cc(a+b)+4}/(c+2)^3 ≧ 2/3.
等号成立は (a,b,c) = (1,1,1) のとき
0619132人目の素数さん
垢版 |
2021/04/25(日) 05:46:51.43ID:We8cr6tt
(略証)
{aa(b+c) + 2 + 2} / (a+1+1)^3
 ≧ 1/ {(1+1+1)[a/(b+c) + 1/2 + 1/2)]}  (← コーシー)
 = (b+c) / {3(a+b+c)},
巡回的にたす。
0621132人目の素数さん
垢版 |
2021/04/25(日) 11:03:19.13ID:YeL676w3
>>619
作問者の天真(Twitter:@bon_miss_tenma)です
こんなにあっさり解かれるとは思ってませんでしたw
ついでに620さんの問題も僕のだったりします、是非挑戦してください!
0622132人目の素数さん
垢版 |
2021/04/25(日) 19:29:46.14ID:We8cr6tt
〔問題620〕
正の実数 a,b,c が aa+bb+cc=3 を満たすとき、次を示せ。
 (2a+1)/(b+c+1)^3 + (2b+1)/(c+a+1)^3 + (2c+1)/(a+b+1)^3 ≧ 1/3,
等号成立は (a,b,c)=(1,1,1) のとき。

(略解)
 (左辺) ≧ (b+c+1)/(b+c+1)^3 + (c+a+1)/(c+a+1)^3 + (a+b+1)/(a+b+1)^3
  = 1/(b+c+1)^2 + 1/(c+a+1)^2 + 1/(a+b+1)^2  (← チェビシェフ)
  ≧ 9/{(b+c+1)^2 + (c+a+1)^2 + (a+b+1)^2}  (← AM-HM / コーシー)
  ≧ 3/{(bb+cc+1) + (cc+aa+1) + (aa+b+1)}
  = 3/{2(aa+bb+cc)+3}
  = 1/3,      (← 題意)
0623132人目の素数さん
垢版 |
2021/04/26(月) 02:25:57.94ID:y9M7sTQu
(補足)
チェビシェフで
(a+1/2)/(b+c+1)^3 + (b+1/2)/(c+a+1)^3 - (a+1/2)/(c+a+1)^3 - (b+1/2)/(b+c+1)^3
 = (a-b) {1/(b+c+1)^3 - 1/(c+a+1)^3}
 ≧ 0,
循環的にたすと
 (左辺) - 1/(b+c+1)^2 - 1/(c+a+1)^2 - 1/(a+b+1)^2 ≧ 0,
0624132人目の素数さん
垢版 |
2021/04/28(水) 04:23:14.87ID:B9p/ERZg
>>618
>>620

〔問題34〕
a,b,c > 0 のとき
 (a(b+c)+1)/(b+c+1)^2 + (b(c+a)+1)/(c+a+1)^2 + (c(a+b)+1)/(a+b+1)^2 ≧ 1,
 Inequalitybot [34] ☆5
 JMO-2010 問4

Inequalitybot も問題番号で検索できるようになってます。
0625132人目の素数さん
垢版 |
2021/04/28(水) 07:27:45.63ID:B9p/ERZg
>>583
〔問題48〕
a,b,c >0 のとき
 (a^5-a^2+3)(b^5-b^2+3)(c^5-c^2+3) ≧ (a+b+c)^3

 USAMO-2004, Q5
 Inequalitybot [48] ☆6
0626132人目の素数さん
垢版 |
2021/04/28(水) 16:58:32.60ID:B9p/ERZg
>>613
 (a+b)^3 - (a+2b)(aa+bb) = aab + (2a-b)bb ≧ 0,
 (a+b+c)^3 - (a+2b+3c)(aa+bb+cc) = aab + (2a-b)bb + (2a+b-2c)cc + 6abc ≧ 0,
 (a+b+c+d)^3 - (a+2b+3c+4d)(aa+bb+cc+dd)
= aa(b-d) + (2a-b-d)bb + (2a+b-2c-d)cc + (2a+b-3d)dd + 6(abc+abd+acd+bcd) ≧ 0,
0627132人目の素数さん
垢版 |
2021/04/28(水) 17:05:44.79ID:B9p/ERZg
これと以下を組み合わせた問題があった。
〔補題〕
a+b+c+… = 1 のとき
  (a^a)(b^b)… ≦ (aa+bb+…),
(略証)
a+b+c+… = s とおく。
y=log(x) は上に凸だから Jensen で
 a・log(a) + b・log(b) + ・・・・ ≦ s・log((aa+bb+・・・・)/s)
 (a^a)(b^b)… ≦ {(aa+bb+…)/s}^s,
s=1 とおく。
0628132人目の素数さん
垢版 |
2021/04/28(水) 19:31:51.11ID:B9p/ERZg
〔問題〕
 tan(1/2) > cos(1).
これの証明はどうすれば出来ますか?

 高校数学の質問スレ411- 028, 936
0629132人目の素数さん
垢版 |
2021/04/28(水) 23:28:33.43ID:Tu1Xrn91
t = tan(1/2)とおいて
tan(1/2)-cos(t)=(t^3+t^2+t-1)/(t^2+1)
なのでコレが+を言えば良い
tan(1/2)=0.546302.....
t^3+t^2+t-1は単調増大で0になるのはt=0.543689....
とりあえず5次までマクローリン展開して
tan(1/2)
>1/2+(1/3)(1/2)^3+(2/15)/(1/2)^5=131/240=0.54583333......
0630132人目の素数さん
垢版 |
2021/04/29(木) 02:03:41.02ID:mxa1BnUU
>>628
θ = 1/2 とおいて
tanθ - cos(2θ) = tanθ - 1 + 2(sinθ)^2
 = tanθ - 3/2 + {1/2 + 2(sinθ)^2}
 ≧ tanθ - 3/2 + 2sinθ  (AM-GM)
 = tanθ + 2sinθ - 3θ
 ≧ 0,   (Snellius-Huygensの式)
0631132人目の素数さん
垢版 |
2021/04/29(木) 02:45:36.56ID:NbeeKPJA
このスネル・ホイヘンスの不等式、以前からどうやって見つけたのか気になってるヤツだ
0636132人目の素数さん
垢版 |
2021/05/05(水) 05:39:50.22ID:16g2LNeV
〔簡易版〕
a,b,c>0 に対して
 (a+b+c)^3 ≧ 27abc{(aa+bb+cc)/(ab+bc+ca)}^(2/3).

(略証)
(a+b+c)^6 = {(aa+bb+cc) + (ab+bc+ca) + (ab+bc+ca)}^3
  ≧ 27(aa+bb+cc)(ab+bc+ca)^2,    (AM-GM)
2/3 乗して
(a+b+c)^4 ≧ 9(ab+bc+ca)^2 {(aa+bb+cc)/(ab+bc+ca)}^(2/3)
  ≧ 27(a+b+c)abc {(aa+bb+cc)/(ab+bc+ca)}^(2/3),

元の問題は解けぬwww
0637132人目の素数さん
垢版 |
2021/05/05(水) 05:41:42.08ID:16g2LNeV
〔問題3.85〕
実数a,b,cに対して
 (aa+2)(bb+2)(cc+2) ≧ 3(a+b+c)^2,

 APMO-2004 A5.改
 文献[9] 佐藤(訳)、朝倉書店 (2013) 問題3.85 p.140
 Inequalitybot [20] ☆8
 [高校数学の質問スレ412−029,036,040]
0638132人目の素数さん
垢版 |
2021/05/05(水) 05:49:27.56ID:16g2LNeV
(解1)
(aa+2)(bb+2)(cc+2) - 3(a+b+c)^2
 = (1/3){(aa+5)(bc-1)^2 + (bb+5)(ca-1)^2 + (cc+5)(ab-1)^2
  + (ab+bc+ca-3)^2 + (a-b)^2 + (b-c)^2 + (c-a)^2}
 ≧0

(解2)
 (aa+2)(bb+2)(cc+2) - 3(a+b+c)^2
 = aa + bb + cc + 2abc + 1 - 2(ab+bc+ca)
 + (abc-1)^2
 + 2(ab-1)^2 + 2(bc-1)^2 + 2(ca-1)^2,

 文献[9] の演習問題1.90 (ii) p.41-42 に帰着する。
〔問題1.90〕(ii)
a,b,c を非負実数とする。このとき
 aa + bb + cc + 2abc + 1 ≧ 2(ab+bc+ca),
0640132人目の素数さん
垢版 |
2021/05/06(木) 05:59:11.34ID:Vi6k/Ft1
>>638
〔例題2.1.11〕
(7) a,b,c が非負実数のとき
 aa + bb + cc + 2abc + 1 ≧ 2(ab+bc+ca),


文献[8] 安藤, 数学書房 (2012) p.36
0641132人目の素数さん
垢版 |
2021/05/30(日) 05:50:26.45ID:EIfW8DuI
>602
{x+y, y+z, z+x} のうち1以上のものが

・2個以上のときは 明らか。
・1個以下のときは 1 > y+z, z+x より 0 < x, y, z < 1  >>604
0642132人目の素数さん
垢版 |
2021/05/30(日) 06:56:34.75ID:EIfW8DuI
>>631
マクローリン展開
 sinθ = θ - (1/3!)θ^3 + (1/5!)θ^5 - (1/7!)θ^7 + (1/9!)θ^9 - …
 tanθ = θ + (1/3)θ^3 + (2/15)θ^5 + (17/315)θ^7 + (62/2835)θ^9 + …
から思い付いたのかも。

>>100 にもあるよ。
 H = θ - (1/180)θ^5 - (1/1512)θ^7 - (1/25920)θ^9 - …
 G = θ + (1/45)θ^5 + (4/567)θ^7 + (1/405)θ^9 + …
 A = θ + (1/20)θ^5 + (1/56)θ^7 + (7/960)θ^9 + …
 A + H - 2G = (1/324)θ^7 + (1/432)θ^9 - …
 AH/GG = (2cosθ+1)/{(2+cosθ)(cosθ)^(1/3)}
    = 1 + (1/324)θ^6 + (1/648)θ^8 + …
0643132人目の素数さん
垢版 |
2021/05/30(日) 08:38:14.15ID:EIfW8DuI
ついでに…
s>0, t>0 とし
 A = (s+s+t)/3,
 G = (sst)^(1/3)
 H = 3st/(s+t+t),
とおくと
 H < G < A,
 AH > GG,   (0<s<t)
 A+H > 2G,   (0<s<t)
(略証)
 AH = (s+s+t)st/(s+t+t),
 G^3 = sst,
より
 (AH)^3 - G^6 = tt {t(s+s+t)^3 - s(s+t+t)^3}{s/(s+t+t)}^3
  = tt(s+t){(t-s)s/(s+t+t)}^3 > 0,
∴ AH > GG,

 (A+H)/2 = (ss+7st+tt)/[3(s+t+t)],
 G^3 = sst,
より
 {(A+H)/2}^3 - G^3 = {(t-s)^3 + 27stt}{(t-s)/[3(s+t+t)]}^3 > 0,
∴ A+H > 2G,
0644132人目の素数さん
垢版 |
2021/05/30(日) 18:54:47.33ID:SMLQU2Ye
>>642
テイラー展開は、あまり時代に合わんような気もする。まあ古くから、特殊な場合だけや結果だけ知られているということがよくあるのと、詳しくないので結論付けられない。

ホイヘンスによる証明があったわ。
円の大きさの発見 : 1654年ホイヘンスによる円周率の計算
https://www2.tsuda.ac.jp/suukeiken/math/suugakushi/sympo27/27_tanuma.pdf
(近似)式自体は、15世紀のニコラウス・クザーヌスまで遡れるらしい。

グレゴリーやニュートンが17世紀後半にべき級数展開したらしいから、ホイヘンスは知らないような気もする。代数計算得意じゃないとキツイし。
0646132人目の素数さん
垢版 |
2021/06/08(火) 05:51:17.44ID:Hilnv+E/
5.Sは3次元座標空間の有限個の点の集合である。
S_x, S_y, S_z はそれぞれ、Sの点の yz-平面, zx-平面, xy-平面への正射影からなる点の集合である。
次を証明せよ。
  | S |^2 ≦ |S_x|・|S_y|・|S_z|
ここに | A | は有限集合Aの要素の個数である。
0647132人目の素数さん
垢版 |
2021/06/08(火) 05:57:08.83ID:Hilnv+E/
>>280

f(x)は下に凸な関数とする。自然数nに対して不等式
 nΣ[k=0,n] f(2k) > (n+1)Σ[k=1,n] f(2k-1)
を示せ。

[面白スレ36.256-260]
0648132人目の素数さん
垢版 |
2021/06/08(火) 20:03:58.37ID:Hilnv+E/
>>645 >>646
z値の集合を {z1, …, zi, …, zn} とする。
S, Sy, Sx の点を z値で分類する。
S, Sy, Sx の点のうち z=zi をみたすものの個数を |Li|, ai, bi とする。

(1) |Li| ≦ ai・bi,

(2) |S| = |L1| + … + |Li| + … + |Ln|,

(3) |Sy| = a1 + … + ai + … + an,
  |Sx| = b1 + … + bi + … + bn,

(4) |Li| ≦ |Sz|,

(1) と (4) を掛けて
 |Li|^2 ≦ (ai・bi) |Sz|,
 |Li| ≦ √(ai・bi) √|Sz|,    ・・・・ (5)

(2), (5) より
|S|^2 ≦ {√(a1・b1) + … + √(ai・bi) + … + √(an・bn)}^2・|Sz|
  ≦ (a1 + … + ai + … + an)(b1 + … + bi + … + bn)|Sz|   コーシー
  = |Sy| |Sx| |Sz|,

http://www.youtube.com/watch?v=IzitrvYnNkc 11:08,
0652132人目の素数さん
垢版 |
2021/06/15(火) 20:42:53.54ID:iDQ7MEu/
>>650
0<k≦3 ゆえ x^(3/k) は下に凸。 x=1 で接線を曳いて、
 (3-k) + k・x^(3/k) ≧ 3x,
(左辺) - (右辺) ≧ aa+bb+cc - 2(ab+bc+ca) + 3(abc)^(2/3)
 ≧ aa+bb+cc - 2(ab+bc+ca) + 9abc/(a+b+c)    (AM-GM)
 = F1(a,b,c)/(a+b+c)
 ≧ 0,

*) Schurの不等式
F1(a,b,c) = a(a-b)(a-c) + b(b-c)(b-a) + c(c-a)(c-b)
 = (a+b+c)^3 - 4(a+b+c)(ab+bc+ca) + 9abc ≧ 0.
0658132人目の素数さん
垢版 |
2021/07/25(日) 04:46:50.40ID:0rv1EuHc
(略解)
 t = ab+bc+ca < 3,
と仮定すると
 u = abc < 1,   (AM-GM)
となり題意に反する。
∴ 3 ≦ t < 4,
∴ s = a+b+c ≧ tt/3 ≧ 3,

(s-t)(ss+st+tt - 4t)
 = (4-t)(t-3)(t+3) + (s^3-4st+9u)
 = (4-t)(t-3)(t+3) + F1(a,b,c)
 ≧ 0,   (← Schur-1)
∴ s-t ≧ 0,
 [面白スレ37.704] にもあった。
0660132人目の素数さん
垢版 |
2021/07/25(日) 05:08:13.86ID:0rv1EuHc
〔類題184〕
a,b,c>0, a+b+c+abc=4 のとき a+b+c≧ab+bc+ca,

大数宿題 2010-Q7
[不等式スレ7.114-115,160]
Inequalitybot [184] ☆7
0661132人目の素数さん
垢版 |
2021/07/25(日) 05:12:18.46ID:0rv1EuHc
(略解)
s = a+b+c < 3 と仮定すると
 u = abc < 1   (AM-GM)
となり題意に反する。
∴ 3 ≦ s < 4.

4s(s-t) = (4-s)(s-3)(s+3) + 9(4-s-u) + (s^3 -4st +9u)
  = (4-s)(s-3)(s+3) + 9(4-s-u) + F1(a,b,c)
  ≧0,     (← Schur-1)
∴ s-t ≧ 0.
0662132人目の素数さん
垢版 |
2021/07/25(日) 06:48:26.36ID:0rv1EuHc
>>655
〔問題2.〕
 任意の実数 x1, x2, ・・・・, xn に対して、不等式
     Σ[i=1,n] Σ[j=1,n] √|xi-xj| ≦ Σ[i=1,n] Σ[j=1,n] √|xi+xj|,
が成り立つことを示せ。
0663132人目の素数さん
垢版 |
2021/07/25(日) 11:16:51.60ID:236oCq6r
実質極値がa=b=cの時でしかもそれが未定定数法で簡単に求まるやつはなんかもひとつやな
0665132人目の素数さん
垢版 |
2021/07/27(火) 19:25:05.18ID:gRrHTwn5
>>655
こんな良い不等式がまだ残ってるとは

これルートなくても成り立ちそうだけど、その場合は簡単に示せたりする?
0666132人目の素数さん
垢版 |
2021/07/29(木) 01:08:01.44ID:gaBM8HMZ
複素数 z (0≦arg(z) < 2π) に対して、
   |z-1| < |z| - 1 + |z|*arg(z).
( ゚∀゚) ウヒョッ!
0668132人目の素数さん
垢版 |
2021/07/29(木) 21:13:16.52ID:a7gJLkin
a_1≧a_2≧…≧a_n>0かつa_1+a_2+…+a_n=1のとき
a_1+2a_2+…na_nのとりうる値の範囲を求めよ.
0669132人目の素数さん
垢版 |
2021/07/30(金) 06:22:01.50ID:oWjQc2j0
f(a) = Σ[k=1,n] k・a_k とおく。
f(1, 0, …, 0) = 1 (最小)
f(1/n, 1/n, …, 1/n) = (n+1)/2 (最大)

(略証)
f(a) - 1 = (a_1+a_2+…+a_n - 1) + Σ[k=2,n] (k-1) a_k ≧ 0,
(n+1)/2 - f(a) = Σ[k=1,n] ((n+1)/2 - k) a_k
   = Σ[k'=1,n] (k' - (n+1)/2) a_{n+1-k'}
   = (1/2)Σ[k=1,n] ((n+1)/2-k) (a_k - a_{n+1-k})  (←同符号)
   ≧ 0,
0671132人目の素数さん
垢版 |
2021/07/30(金) 14:02:27.84ID:oWjQc2j0
>>669
(n+1)/2 - f(a) = ((n+1)/2) (1 - a_1 - a_2 - … - a_n)
      + (1/2) Σ[k=1,n-1] k(n-k) (a_k - a_{k+1})
     ≧ 0,
の方がいいか…
0675132人目の素数さん
垢版 |
2021/09/04(土) 18:23:48.83ID:KMsJe/e+
a, b, c が0以上かつ a^2 + b^2 + c^2 = 1 を満たすとき,
(a+bーc)^n + (b+c-a)^n + (c+a-b)^n (n は3以上の整数) 
の最大値と最小値を求めよ.
0676132人目の素数さん
垢版 |
2021/09/04(土) 21:02:15.78ID:HGuBdRDo
最大値 2^{n/2}
  a = 0, b = c = 1/√2 など。 (x=√2, y=z=0, etc.)
最小値 (1/3)^{n/2 - 1}
  a = b = c = 1/√3,  (x=y=z=1/√3)

x = b+c-a, y = c+a-b, z = a+b-c とおくと
1 = aa + bb + cc
 = {(x+y)^2 + (y+z)^2 + (z+x)^2}/4
 = {(x+y+z)/√3}^2 + (1/4){(x-y)/√2}^2 + (1/4){(x+y-2z)/√6}^2,
回転楕円体 (どら焼き形)
 短軸:1  (1,1,1)方向  
 長軸:2  それと垂直方向
0677132人目の素数さん
垢版 |
2021/09/04(土) 23:49:24.75ID:KMsJe/e+
>> 676

もう少し具体的に
0678132人目の素数さん
垢版 |
2021/09/05(日) 02:00:08.90ID:HFxHmzMl
a=u^2,b=v^2,c=w^2
束縛
C = u^4+v^4+w^4-1
評価関数
S = (v^2+w^2-u^2)^n+(w^2+u^2-v^2)^n+(u^2+v^2-w^2)^n

s = 2n((v^2+w^2-u^2)^(n-1)+(w^2+u^2-v^2)^(n-1)+(u^2+v^2-w^2)^(n-1))とおいて
dC=4(u^3,v^3,w^3)
dS=s(u,v,w)
s≠0により
dSがdCで張られる
⇔vw(v^2-w^2)=wu(w^2u^2)=uv(u^2-v^2)=0
⇔u^2=v^2=w^2 or u^2=v^2 & w=0 or u=v=0 or...
0679132人目の素数さん
垢版 |
2021/09/05(日) 06:18:37.20ID:Zhg5gGCb
専門的過ぎてついていけない
数オリの高校生の理解できる解法でお願いします
0680132人目の素数さん
垢版 |
2021/09/05(日) 10:01:38.24ID:HFxHmzMl
数学の問題は進んだテクニック使っても全然簡単にならず、実は中学生でも理解できるような話の方が楽に解ける時がある
数オリとかの問題とかそういう問題のオンパレードだし、ピーターフランクルとかそんな問題大好きの人もいっぱいいる
しかしそれは進んだ数学を勉強しないでいい理由になどにはならないし、ましてや逆に言えば、進んだテクニック使えば楽に解ける問題をいつまでもいつまでもそういう”初頭数学縛り”をかけて解くのは単なる“自己満”でしか無い
不等式の話を本当に極めるなら未定乗数法は絶対避けては通れない
0681132人目の素数さん
垢版 |
2021/09/05(日) 12:53:40.46ID:LDbpAA38
grad(f(u,v,w)) = ∇f = (∂f/∂u, ∂f/∂v, ∂f/∂w)
s1 = 2n{-(v^2+w^2-u^2)^(n-1) + (w^2+u^2-v^2)^(n-1) + (u^2+v^2-w^2)^(n-1)},
s2 = 2n{(v^2+w^2-u^2)^(n-1) - (w^2+u^2-v^2)^(n-1) + (u^2+v^2-w^2)^(n-1)},
s3 = 2n{(v^2+w^2-u^2)^(n-1) + (w^2+u^2-v^2)^(n-1) - (u^2+v^2-w^2)^(n-1)},
とおくと
 grad(C) = ∇C = 4(u^3, v^3, w^3)
 grad(S) = ∇S = (s1・u, s2・v, s3・w)
       = s(u, v, w)
ここから ついていけない…
0682132人目の素数さん
垢版 |
2021/09/06(月) 18:13:36.65ID:eC9BaMcK
問題[2]
 a_n = (1 + 1/n)^n, b_n = (1 + 1/n)^(n+1)  (nは正の整数)
とおくとき、nが増加するとa_nは増加し、b_nは減少することを証明せよ。
 (数学検定 2011年秋, 1級 2次 問題[2] の一部)
* 作問者は AM-GM を活用する解答を期待していたが…

〔補題258〕               >>258
 (1) (1 + 1/n)^(n+1/2) は単調減少でeに収束
 (2) n! < n^(n+1/2) / e^(n-1),
 (3) (2n)! / n! < (√2)(4n/e)^n,

>>263
Σ[k=1,n] (1/((k+1)(k!)^2))^(1/k) ≒ 1.99877613 - ee/n + 64.5/nn - …
0685132人目の素数さん
垢版 |
2021/09/15(水) 17:00:11.30ID:cOPYG12B
f(t)=1/(n+e^t)、F(t1,‥) = Σf(ti)とおく
f(t)はt≧lognで下に凸かつt≦lognで上に凸
全てtiがlognより小さい領域ではti=0のときFは最大値1
そうでない領域でΣti=0かつF(ti)>1が存在すれば
t1 =(n-1)c, ti=-c (i≧2,t1>logn)
であるtiで存在する
e^t=uとおいて
F(ti)-1
= 1/(n-1+u^(n-1) + (n-1)/(n-1+1/u)-1
= 1/(n-1+u^(n-1) - 1/((n-1)u+1)
しかしu≧1において
n-1+u^(n-1)≧(n-1)u+1
であるから矛盾
0686132人目の素数さん
垢版 |
2021/09/16(木) 05:07:13.88ID:Sn49tAbo
背理法で…
不等式が成り立たないとする。すなわち、
 Σ[k=1,n] 1/(n-1+x_k) >1,
であると仮定する。このとき
1/(n-1+x_i) > 1 - Σ[k≠i] 1/(n-1+x_k)
 = (1/(n-1))Σ[k≠i] x_k /(n-1+x_k)
 ≧ ( Π[k≠i] x_k /(n-1+x_k) )^{1/(n-1)}, (AM-GM)
となる。i=1,…,n で掛けて
 Π[i=1,n] 1/(n-1+x_i) > Π[k=1,n] x_k /(n-1+x_k),
となるが、これは 1 > Π[k=1,n] x_k を意味するので矛盾である。

ルーマニアMO-1999,
文献[9], 佐藤(訳), 朝倉書店(2013), 問題3.35 p.131
Inequalitybot [109]
0687132人目の素数さん
垢版 |
2021/09/16(木) 05:13:59.10ID:Sn49tAbo
〔類題〕
 x_1, x_2, …, x_n >0 が Σ[k=1,n] 1/(n-1+x_k) = 1 を満たすとする。
このとき
   Π[k=1,n] x_k ≧ 1,
を証明せよ。

文献[9], 佐藤(訳), 朝倉書店(2013), 問題1.46改 p.14
0688132人目の素数さん
垢版 |
2021/09/20(月) 10:34:26.70ID:YMP5Sl+4
(1)
z,w∈C、|z|=|w|=1 のとき、
|z+1| + |w+1| + |zw+1| ≧ 2

(2)
a,b,c∈C に対して、
|a| + |b| + |c| ≦ |a+b-c| + |b+c-a| + |c+a-b|

( ゚∀゚) ウヒョッ!
0689132人目の素数さん
垢版 |
2021/09/20(月) 22:52:38.35ID:lYkiWXwV
(1)は簡単やな
x^をxの複素共役として
|z+1|+|w+1|+|zw+1|
=|z+1|+|w^+1|+|z+w^|
なので|a|+|b|+|c|=1のとき
|b+c|+|c+a|+|a+b|≧2
を示せば良い
b = c exp(2iA), c = exp(2iB), a = exp(2iC), A+B+C=π
となる非負実数A,B,Cがとれるとしてよくこのとき
|b+c|+|c+a|+|a+b|
=2(cosA+cosB+cosC)
であるからcos(x)の凸性により(A,B,C)=(-π,π,π),(π,-π,π),(π,π,-π)のとき最小値2
0690132人目の素数さん
垢版 |
2021/09/20(月) 23:02:12.47ID:lYkiWXwV
(2)は力技で
sを複素定数としC^3の領域
R={ .. | a + b + c = 2s }
におけるS=2( |s-a| + |s-b| + |s-c| ) - ( |a| + |b| + | c| )の最小値が0以上であることを示せば良い
それには全微分できない領域で非負、全微分可能な極値で非負を言えば十分
s=0であればS=|a|+|b|+|c|となり自明だからs≠0とする

i) a=0のとき
S=2(|b+c|/2 + |b-c|/2 × 2) - (|b|+|c|)
=|b+c|/2-|b|+|b+c|/2-|c|+|b-c|
≧-|b-c|/2 × 2 + |b-c| = 0

(ii) a=s のとき
このときs=a=b+cより
S=2(|b|+|c|)-(|b+c|+|b|+|c|)
=|b|+|c|-|b+c|≧0

(iii) a=bのとき
このときs=a+c/2より
S=2(|c/2|+|c/2|+|a-c/2|)-(|a|×2+|c/2|)
=|c|+|2a-c|-|2a|≧0

(iv)a,b,cが同一直線上のとき
a,b,cは実数としてよくSをaの関数として見たときlim[a→±∞]S=∞だから極値だけ考えればよく、極値をとるのはa=s,0の場合のみであるから既出の場合に還元される

(v)その他の場合
Sは全微分可能でありz^を複素共役としてe(z)=z/|z|とおけば
dS = -2(e(s-a)^da + e(s-a)da^+ e(s-b)^db +e(s-b)db^+ e(s-c)^dc + e(s-c)dc^)-(e(a)^da+e(a)da^+e(b)^db+e(b)db^+e(c)^dc+e(c)dc^)
でありコレがda+db+dcの複素定数倍であるから
2e(s-a)+e(a)=2e(s-b)+e(b)=2e(s-c)+e(c)=0
である
よってa,b,cが同一直線上となるので既出のケースに還元される
0691132人目の素数さん
垢版 |
2021/09/21(火) 12:09:47.62ID:AENcTZtD
>>689
(1)
|a| = |b| = |c| = 1 のとき
 |b+c| + |c+a| + |a+b| ≧ 2,
ですか。

>>690
(2) は簡単やな。Ravi変換で
 b+c-a = p,
 c+a-b = q,
 a+b-c = r,
とおけば
(左辺) = |a| + |b| + |c|
 = |q+r|/2 + |r+p|/2 + |p+q|/2
 ≦ |p| + |q| + |r|.
0692132人目の素数さん
垢版 |
2021/09/21(火) 12:22:40.65ID:IIHpCqtI
あれ?
その方法最初に考えてダメと思ったんやけど勘違いしたかな?
まぁ複素係数の微分形式の復習になったからいいけど
0693132人目の素数さん
垢版 |
2021/09/21(火) 20:58:30.74ID:AENcTZtD
>>689
 C ≧ π/2 の場合 (鈍角) は
 |b+c| + |c+a| + |a+b|
 = 2(|cosA| + |cosB| + |cosC|)
 ≧ 2(cosA + cosB)
 ≧ 2(1 + cos(A+B))   (凸性)
 ≧ 2,        (A+B≦π/2)
ですね。あるいは
 cosA + cosB + cosC
 = 1 + 4sin(A/2)sin(B/2)sin(C/2)   (A+B+C=π)
 ≧ 1,
0694132人目の素数さん
垢版 |
2021/09/22(水) 19:57:40.85ID:K2h4cEAP
>>688
(1)は簡単やな
 |z+1| + |w^+1| + |z+w^|
 ≧ |(z+1) + (w^+1) - (z+w^)|
 = 2,

 |b+c| + |c+a| + |a+b|
 ≧ |-(b+c) + (c+a) + (a+b)|
 = 2|a|,
同様にして
 |b+c| + |c+a| + |a+b| ≧ 2 Max{|a|,|b|,|c|}

(2)は簡単やな  >>691
0695132人目の素数さん
垢版 |
2021/09/22(水) 20:21:10.20ID:miCnVfcc
>>694
なんでそういう書き方するん?
それ読んだ相手がどういう気持ちになるか考えられへんの?
0697132人目の素数さん
垢版 |
2021/09/25(土) 11:52:48.15ID:S56dxsDJ
1/n^3 = n/n^4
 < n/(nn-1/4)^2
 = {(n+1/2)^2 - (n-1/2)^2}/{2(nn-1/4)^2}
 = (1/2){1/(n-1/2)^2 - 1/(n+1/2)^2}
∴ Σ[n=2,∞] 1/n^3 < 2/9 = 0.222222
ぢゃ出ない・・・・orz
0698132人目の素数さん
垢版 |
2021/09/28(火) 04:04:41.51ID:xRhStcay
c>0 かつ z,w∈C のとき、
|z+w|^2 ≦ (1+c)|z|^2 + (1 + 1/c)|w|^2.

昔やったかも? ( ゚∀゚)
0699132人目の素数さん
垢版 |
2021/09/28(火) 12:43:23.72ID:ED+tdwHx
|z+w|^2 ≦ (|z| + |w|)^2     (三角不等式)
  = |z|^2 + |w|^2 + 2|z||w|
  = |z|^2 + |w|^2 + c|z|^2 + (1/c)|w|^2 - (|z|√c - |w|/√c)^2
  ≦ |z|^2 + |w|^2 + c|z|^2 + (1/c)|w|^2   (GM-AM)
  = (1+c)|z|^2 + (1+1/c)|w|^2.
等号成立は w = cz のとき。
0700132人目の素数さん
垢版 |
2021/09/28(火) 12:56:38.75ID:ED+tdwHx
>>696
 φ = (1+√5)/2 = 1.618034… とおく。 (黄金比)
1/n^3 = n/n^4
 ≦ n/{n^4 - (2/φ-1)^2・(nn-4)}    (n≧2)
 = n/{(nn -2 +4/φ)^2 - nn}
 = n/{(nn -n -2 +4/φ)(nn +n -2 +4/φ)}
 = (1/2){1/(nn-n-2 +4/φ) - 1/(nn+n-2 +4/φ)},
∴ Σ[n=2,∞] 1/n^3 < φ/8 = (1+√5)/16,
0702132人目の素数さん
垢版 |
2021/09/29(水) 09:17:12.31ID:lKJ2KBeg
>>696
1/n^3 = n/n^4
 < n/{n^4 - (√10 -3)^2(nn-9)}    (n≧3)
 = n/{(nn -9 +3√10)^2 - n^2}
 = n/{(nn -n -9 +3√10)(nn +n -9 +3√10)}
 = (1/2){1/(nn -n -9 +3√10) - 1/(nn +n -9 +3√10)},
∴ Σ[n=2,∞] 1/n^3 < 1/8 + (1+√10)/54 = 0.2020792
0705132人目の素数さん
垢版 |
2021/09/30(木) 06:51:58.59ID:hn+yThHP
arg(a) = arg(b) = arg(c) かつ |a|, |b|,| c| が三角不等式を満たす。
ただし arg(0) は任意の値に等しいとする。

∵ arg(p) = arg(q) = arg(r).
0706132人目の素数さん
垢版 |
2021/09/30(木) 22:46:47.82ID:jz/TtT2s
Σ(k=1~2n)2nCk×(1/(n-1)^2)^k>2/(n-1)

↑これ成り立ちそうなんだけど証明浮かばん
0707132人目の素数さん
垢版 |
2021/09/30(木) 23:26:03.13ID:yVhW4Ory
とりあえずt = 1/(n-1)とおいてt=0の近傍では左辺-右辺は
4 t^2 + (13 t^3)/3 + (11 t^4)/3 + (8 t^5)/5 - (7 t^6)/90 + O(t^7)
(テイラー級数)
だそうな
0708132人目の素数さん
垢版 |
2021/10/01(金) 06:04:21.62ID:y+GdRVMF
初項は 2n/(n-1)^2 > 2/(n-1),
あとの項も >0,

なお、二項公式から
 Σ(k=1〜2n) C(2n,k) (t^2)^k = (1+t^2)^{2n} - 1.
0709132人目の素数さん
垢版 |
2021/10/18(月) 20:54:00.80ID:gvNZ1Lh7
0<a<b,A=(a+b)/2 , G=(ab)^(1/2), H=2ab/(a+b), I(a,b)≡a(a+3b)/(3a+b) とおくと
H≦I(a,b)≦G≦I(b.a)≦A 
が成立するんですけどI(a,b)の意味というか簡単な解釈みたいのってあるのでしょうか?
証明はただ計算すればいいんですけど、、
0710132人目の素数さん
垢版 |
2021/10/19(火) 18:46:50.39ID:OvSIJGC7
分かりませんね。

 I(a,b) = (A+b)/(1+b/H),
 I(b,a) = (A+a)/(1+a/H),
なので
 f(x) = (A+x)/(1+x/H),
とおいてみますか。
これはxの一次分数式で単調減少です。
 f(∞) = H,
 f(b) = I(a,b),
 f(H) = (A+H)/2,
 f(G) = √(AH) = G,
 f(A) = 2AH/(A+H),
 f(a) = I(b,a),
 f(0) = A,
0711132人目の素数さん
垢版 |
2021/10/19(火) 19:59:53.99ID:G5c9BaN8
>>709

AとHの調和平均を J(a,b)=2AH/(A+H)=4ab(a+b)/(a^2+b^2+6ab) とすると

H≦I(a,b)≦J(a,b)≦G≦ab/J(a,b)≦I(b,a)≦A も成立
0714132人目の素数さん
垢版 |
2021/10/20(水) 19:32:20.22ID:Xr1EP2wS
 f(x) = (A+x)/(1+x/H) ⇔ (f-H):(A-f)=H:x (H,Aの内分比をxで定める)
 g(x) = (b+x)/(1+x/a) ⇔ (g-a):(b-g)=a:x (a,bの内分比をxで定める)

 f(-A) = 0        g(-b) = 0
 f(-b) = a        g(∞) = a
 f(∞) = H        g(b) = H
 f(b) = I(a,b)      g(A) = I(a,b)
 f(A) = 2AH/(A+H)  g(I(b,a)) = 2AH/(A+H)
 f(G) = √(AH) = G  g(G) = √(AH) = G
 f(H) = (A+H)/2    g(I(a,b)) =(A+H)/2
 f(a) = I(b,a)      g(H) = I(b,a)
 f(0) = A        g(a) = A
 f(-a) = b        g(0) = b
 f(-H) = ∞       g(-a) =∞
0715132人目の素数さん
垢版 |
2021/10/24(日) 18:45:19.81ID:P60+HIiU
久々に来たまだあったんだ
aopsは計算機でSOSするのが多く手だと難しいのが多いね
あとは巡回式とか、巡回式じゃない非対称とかも最近多い
不等式を解くテクニックとかは飽和かな?
0719132人目の素数さん
垢版 |
2021/11/02(火) 05:33:41.97ID:te4HpQwE
解1.
A(1/3,0) B(3,0) P(x,y) とおくと
 (与式) = 3(AP/BP)

(与式) = k (<3) は
 AP:BP = k:3
なる アポロニウスの円である。
直径の両端は 内分点と外分点
 (3-8/(3-k),0) と (3-8/(3+k),0)
閉円盤 xx+yy≦1 と共有点をもつことから
 3-8/(3-k) ≧ -1 または 3-8/(3+k) ≦ 1,
∴ k ≦ 1,

解2.
|z-3|^2 - |3z-1|^2
 = (z-3)(z~-3) - (3z-1)(3z~-1)
 = 8(1-zz~)
 = 8(1-|z|^2),
あるいは
|r・e^(iθ)-3|^2 - |3r・e^(iθ)-1|^2
 = (rr -6r・cosθ +9) - (9rr - 6r・cosθ +1)
 = 8(1-rr),
よって
 |z|≦1  ⇔  |(3z-1)/(z-3)| ≦ 1,
0720132人目の素数さん
垢版 |
2021/11/02(火) 06:43:06.22ID:te4HpQwE
〔補題〕
d(X) がある空間における距離ならば
 D(X) = (1/2)log{h + d(X)^2},   h≧4/3
も距離。

[面白スレ39.399,402,404]
0724132人目の素数さん
垢版 |
2021/11/06(土) 15:34:08.46ID:QOJe0Sk2
 |z| ≦ 1,
 |z-1| ≦ |z| + 1 ≦ 2,
 |z-2| ≦ |z| + 2 ≦ 3,
 ……
辺々掛けて
 |z(z-1)(z-2)…(z-n)| ≦ (n+1)!,
等号は z=-1 のとき。
0727132人目の素数さん
垢版 |
2021/11/15(月) 18:53:19.04ID:EETFS5Z+
凄くはないよ
その設定で作問してる
等号成立条件が一致してるとか出来過ぎ
汎用性がない解法
0728132人目の素数さん
垢版 |
2021/11/20(土) 01:53:37.23ID:ecvBNjJu
〔問題471〕
任意の自然数nに対して、次を示せ。
 1/n > 1/(n+1) + 1/(n+1)(n+2) + 1/(n+1)(n+2)(n+3) + ……

[高校数学の質問スレPart414.471]
0731132人目の素数さん
垢版 |
2021/11/21(日) 20:28:37.29ID:myOhL9Wf
右辺は {n!e} になるらしい。

高校生なら等比級数
 1/n = 1/(n+1) + 1/(n+1)^2 + 1/(n+1)^3 + ……
を使うだろうけど...
0732132人目の素数さん
垢版 |
2021/11/21(日) 21:24:18.50ID:FQCqRacp
>>731
そのまんまじゃないか。全く気付かずガックリ。そして>>729となるのも知ってる人は知っているヤツよなあ(積分がある値になることを知っていれば、勘で逆算できる可能性がある)
0733132人目の素数さん
垢版 |
2021/11/22(月) 05:31:37.87ID:ezmXDy6Q
>>729
 被積分関数は x≒1 で急増するから、そこで精度が必要。
しかし x=1 で上から接するのは、e^{1-x} が下に凸なので難しい。
そこで e^{x-1} ≧ x と下から接すれば
 e^{1-x} ≦ 1/x,
∴ (与式) < ∫[0,1] x^{n-1} dx = 1/n を得る。
0734132人目の素数さん
垢版 |
2021/11/24(水) 06:01:42.38ID:JOGGpS/y
〔補題292〕
a,b,c,d > 0 のとき
 a/b + b/c + c/d + d/a ≧ 8(ac+bd)/((a+c)(b+d)),
等号成立は a=c, b=d のとき。

[分かスレ471.279,292]
0735132人目の素数さん
垢版 |
2021/11/24(水) 23:12:24.17ID:4mZSj8jn
>>734

a/b + b/c + c/d + d/a
≧ 2√{(ac)/(bd)}+ 2√{(bd)/(ac)}
= 2(ac + bd) / √(abcd)
≧ 8(ac + bd) / {(a + c)(b + d)}
0737132人目の素数さん
垢版 |
2021/11/27(土) 12:53:00.48ID:HxEDg/nu
〔掛谷の定理〕
 a_n z^n - a_(n-1) z^(n-1) + …… + (-1)^(n-1) a_1 z + (-1)^n a_0 = 0 (a_k>0)
の根は m ≦ |z| ≦ M をみたす。
ここに m = min{a0/a1, a1/a2, …, a(n-1)/a_n}
   M = Max{a1/a0, a2/a1, …, a(n-1)/a_n}

高橋正明 著「複素数」科学新興社モノグラフ13. (1972)
高橋正明 著「複素数」改訂版, 科学新興新社モノグラフ9. (1998)

すべて実根のときは、ニュートンの不等式から
  a(k-1)/a_k ≦ a_k/a(k+1),
 m = a_0/a_1 ≦ …… ≦ a(n-1)/a_n = M,

∴ a_0/a_1 ≦ z ≦ a(n-1)/a_n,
0739132人目の素数さん
垢版 |
2021/11/28(日) 11:20:03.28ID:MWTbmNPN
>>737
(訂正スマソ)
  M = Max{a0/a1, a1/a2, …, a(n-1)/a_n}

元の形は
 0 < a_0 < a_1 < … < a(n-1) < a_n のとき |z| <1,
 a_0 > a_1 > … > a(n-1) > a_n > 0 のとき |z| >1,
0741132人目の素数さん
垢版 |
2021/12/20(月) 09:14:39.04ID:d28R2ON3
>>740
これは大学の解析の問題やな
普通の高校生には手も足も出ないだろな
東大の問題よりセンスがあってカッコ良いわ
0743132人目の素数さん
垢版 |
2021/12/20(月) 11:21:02.50ID:NKcN+ZAk
全然手抜きじゃないだろ
手抜きというのは東大の円周率の評価式や加法定理の証明問題をいうんだよ
0744132人目の素数さん
垢版 |
2021/12/20(月) 14:52:43.21ID:HFnFoVGH
数学できるヤツいらんから、大学で扱ってる問題拾ってきて出してるだけ。

東大のは採点が大変なんだから手抜きにはならん。(想定解答をもとに採点するが別解が出てくれればまた採点官で検討して反映させるので)
0746132人目の素数さん
垢版 |
2022/01/07(金) 20:39:08.21ID:uuJtvVgV
a,b,c>0,
ab+bc+ca=12,
√(ab) + √(bc) + √(ca) + 32/(abc) ≧10.

( ゚∀゚) プケラッチョ
0748132人目の素数さん
垢版 |
2022/01/11(火) 20:20:46.43ID:V9PX5rXA
>>746
条件式の相加相乗から8≧abcなので
√(ab)+√(bc)+√(ca)+16/(abc)+16/(abc)≧5(256/(abc))^(1/5)≧10
0751132人目の素数さん
垢版 |
2022/03/12(土) 20:51:17.19ID:+WWCnbfX
複素関数の本を読んでいたら、ハルナックの不等式が出てきた。春泣く不等式
0752132人目の素数さん
垢版 |
2022/04/13(水) 20:44:36.01ID:on0g0jTO
z∈C に対して |z(4-z)|<1 を解きたいんですけど、どうやればいいんでせうか?
0756132人目の素数さん
垢版 |
2022/04/14(木) 00:17:50.63ID:UD9sHCgS
ごめん違う問題だったな。平方完成しててきとうにやればええやろ。
0759132人目の素数さん
垢版 |
2022/04/14(木) 01:09:54.51ID:wrO3la+m
>>758
最後の方にある

y = sqrt(-x^2 + sqrt(16 x^2 - 64 x + 65) + 4 x - 8)



y = -sqrt(-x^2 + sqrt(16 x^2 - 64 x + 65) + 4 x - 8)

の間やろ
0760132人目の素数さん
垢版 |
2022/04/14(木) 02:27:28.97ID:UD9sHCgS
>>758
スケールおかしいだけで、円の内部やぞ。
|z(4-z)|=|z||z-4|だから原点0とzのキョリと4とzのキョリの積、それが1未満。0付近か4付近だと小さくなる。
0762132人目の素数さん
垢版 |
2022/04/14(木) 04:05:38.21ID:RdL/Dtg3
zの存在領域は、0の近傍と4の近傍に2つある。
境界線は次式で表されて、楕円っぽい形で、円にはならない。
y=-sqrt{-xx+4x-8+√(16xx-64x+65)}
y= sqrt{-xx+4x-8+√(16xx-64x+65)}.
0763132人目の素数さん
垢版 |
2022/04/14(木) 12:44:16.36ID:l1Bgx+ly
ごめんボーッとしてて頭おかしくなってたわ。
因数分解して(二次式)(二次式)<0にできるんだから楕円2つの内部だ。楕円のようなカタチの二次式は楕円しかないので。

>>760
>>758
一行目完全にムシ、それ以外は正しい。
0765132人目の素数さん
垢版 |
2022/04/14(木) 18:34:50.58ID:RdL/Dtg3
>>763
正確にはだえんじゃないよね。
y=-sqrt{-xx+4x-8+√(16xx-64x+65)}
で言うと、左半分が少し広がっている。
0766132人目の素数さん
垢版 |
2022/04/14(木) 18:47:04.72ID:RdL/Dtg3
もともとは、冪級数
 S(z) = Σ[n=0 to ∞] {z(4-z)}^n
について、
(1) S(z) の収束する領域が2つの分離した領域であることを示せ。
(2) S(z) を最大限に解析接続せよ。
という問題。

 y=-sqrt{-xx+4x-8+√(16xx-64x+65)}
 y= sqrt{-xx+4x-8+√(16xx-64x+65)}.

これって微分して増減表調べればグラフの概形が分かるだろうけどダルイ。

(1)を、z = re^(iθ)、4-z = se^{ i (φ+)π} とおいて、極形式で考える方法は挫折。
0767132人目の素数さん
垢版 |
2022/04/14(木) 20:37:32.40ID:UD9sHCgS
>>764
x・yの二次式って意味だった。

>>765
ホンマや、図示と違って楕円じゃないな。。。ちゃんとした図示じゃなかったってだけか。
てっきり因数分解=0とできるもんだと誤解したわ。
0772132人目の素数さん
垢版 |
2022/04/19(火) 04:42:19.83ID:pmf8vt90
USAMO第六問

nは2以上の整数
実数列x_1≧x_2≧…≧x_n,y_1≧y_2≧…≧y_n があり、次式を満たしている
・0=Σx_i=Σy_i
・1=Σ(x_i^2)=Σ(y_i^2)
このとき
Σ(x_i*y_i-x_i*y_(n+i-1))≧2/√(n-1)
を示せ
0776132人目の素数さん
垢版 |
2022/04/25(月) 17:46:55.78ID:EdwWMLBZ
これな
https://i.imgur.com/jg0aj9g.jpg
シンプルに見える問題でも複雑な数になる好例
0777132人目の素数さん
垢版 |
2022/04/25(月) 17:53:23.58ID:EdwWMLBZ
同じくシンプルだが複雑な定数になる例、
あやな(@suugaku1)の不等式
-M≦sinx+sin2x≦M
のMも偶然だが1.76くらいになる
0778132人目の素数さん
垢版 |
2022/04/25(月) 20:00:08.42ID:q7yGPC6B
ウクライナMO2021

a≧b≧c>0のとき
(a^2+b^2)/(a+b)+(b^2+c^2)/(b+c)+(c^2+a^2)/(c+a)≧a+b+c+(a-c)^2/(a+b+c)
0780132人目の素数さん
垢版 |
2022/04/26(火) 00:34:34.71ID:NvtQufOW
代数関数の値域の上限、下限は係数体の代数的拡大の元
決定するためのアルゴリズムも見つかってる
0783132人目の素数さん
垢版 |
2022/05/01(日) 13:38:24.60ID:69yOeg50
>>775 >>776
nを4から正の偶数に一般化してみた

任意の相異なる実数a,b,cに対し
(a/(b-c))^n + (b/(c-a))^n + (c/(a-b))^n ≧ (2-k)/(2k-k^2)^(n/2)
ここにkは -1+x+2x^n = 0 の正の根

n≧4で等号成立は以下の6組
a:b:c = k±√(2k-k^2) : k±k√(2k-k^2) : -k(1-k)
a:b:c = k±k√(2k-k^2) : k±√(2k-k^2) : -k(1-k)
a:b:c = k±√(2k-k^2) : -(-k±√(2k-k^2)) : 2
0786132人目の素数さん
垢版 |
2022/07/20(水) 02:52:11.46ID:83qPeNBR
w∈C、|w| < 1/2 に対して、|e^w -1| < |w|e^{1/2}.

w∈Rのときは分かるけど、複素数のときはどうやって証明するんでしょうか?
0787132人目の素数さん
垢版 |
2022/07/20(水) 08:22:42.62ID:d+g2kWua
f(z) = eᶻ-1とおけば最大値の原理から|z|<1/2に対して

|eᶻ-1|/|z| < max { |eᶻ-1|/z ; |z| = 1/2 }
= 2((e^(1/2)-1) (z = 1/2のとき最大値)
= 1.2974425414
≦ e^(1/2) = 1.6487212707
0789132人目の素数さん
垢版 |
2022/07/20(水) 13:45:48.58ID:rsT8xIwC
>>787
ごめん、やっぱ分かってなかった。
境界 |z| = 1/2 で最大値をとることまでは分かったけど、z=1/2で最大となるのはなぜですか?

|z| = 1/2のときに、|(e^z -1)/z| = 2|e^z -1| までは分かるけど、
2|e^z -1| ≦ 2(e^{1/2} -1) はなぜ?
0790132人目の素数さん
垢版 |
2022/07/20(水) 15:00:00.10ID:kuOeuKMZ
|e^w-1|
=|w^1/1!+w^2/2!+w^3/3!+...|
<=|w|(1/1!+|w|/2!+|w|^2/3!+...)
<=|w|(1/1!+(1/2)/2!+(1/2)^2/3!+...)
=|w|(e^(1/2)-1)/(1/2).
0792132人目の素数さん
垢版 |
2022/08/04(木) 02:20:43.56ID:2zMvcFob
単位円上の5点を取り10本の線分で結ぶと凸包の五角形が、11個の部分に分割される
中央の小五角形の面積をT、小五角形と辺を共有する5つの三角形の面積の総和をSとする
S + 2Tの最大値を求めよ
0804132人目の素数さん
垢版 |
2022/08/17(水) 23:57:56.28ID:VYryJHFS
>>800
計算しなおしたら、4/3でした。 すまそ。

z∈C、|z|=1 に対して、
|e^z - z^3 - z - i| ≦ e + (√2) - 4/3.
0807132人目の素数さん
垢版 |
2022/09/09(金) 10:08:28.12ID:BXbMv5wz
ほー、スネルとホイヘンスといえば光学か
確かに光学(の何かに)使えそうな見た目してんな>>806
0808132人目の素数さん
垢版 |
2022/09/09(金) 15:27:50.14ID:b56r1F4Z
十分小さな z∈C に対して、|z|/2 ≦ |log(1+z)| ≦2|z|.
これはどうやって証明するのでせうか?
0809132人目の素数さん
垢版 |
2022/09/09(金) 15:37:28.25ID:iQiE86Vx
マクローリン展開じゃないの?
| log(1+z) - z | ≦ Σ[k≧2] |zᵏ|/k
≦ Σ[k≧2] |zᵏ|/2
= |z|²/(1-|z|)
0816132人目の素数さん
垢版 |
2022/09/27(火) 00:51:21.62ID:3dnqmVEF
| (exp(z)-1-z)/z |
= | Σ[ k≧2 ] zᵏ⁻¹/k! |
≦ | Σ[ k≧2 ] | z Iᵏ⁻¹/k!
≦ | Σ[ k≧2 ] 1/k!
= exp(1) -2
≦ 1/2+1/6(1/(1-1/4))
= 13/18
< 3/4
0819132人目の素数さん
垢版 |
2023/03/03(金) 04:38:08.86ID:bTCAa9Qt
ちらっと立ち読みしただけなんだが、数蝉のNoteに、階乗のAM-GMが載っていたよ、
0833132人目の素数さん
垢版 |
2023/05/21(日) 12:08:26.23ID:LagPnPNw
exp(x)>x
0944132人目の素数さん
垢版 |
2023/05/22(月) 23:34:24.40ID:uY+yy519
0+0=0と分配律により
∀a、a+0=aでありa=0とすると
0+0=0
(0+0)a=0a、0a+0a=0a
∴0a=0
0949132人目の素数さん
垢版 |
2023/05/22(月) 23:39:24.43ID:uY+yy519
群Gに置いて
aa⁻¹=e
これはaの逆元がa⁻¹
a⁻¹の逆元がaであることを示す
0962132人目の素数さん
垢版 |
2023/05/22(月) 23:57:37.64ID:uY+yy519
Rに関してae=ea=aでありR*に制限しても単位元eは同じものが同じ役割を持つ。すなわち単位元は存在する。
10011001
垢版 |
Over 1000Thread
このスレッドは1000を超えました。
新しいスレッドを立ててください。
life time: 1616日 2時間 48分 56秒
10021002
垢版 |
Over 1000Thread
5ちゃんねるの運営はプレミアム会員の皆さまに支えられています。
運営にご協力お願いいたします。


───────────────────
《プレミアム会員の主な特典》
★ 5ちゃんねる専用ブラウザからの広告除去
★ 5ちゃんねるの過去ログを取得
★ 書き込み規制の緩和
───────────────────

会員登録には個人情報は一切必要ありません。
月300円から匿名でご購入いただけます。

▼ プレミアム会員登録はこちら ▼
https://premium.5ch.net/

▼ 浪人ログインはこちら ▼
https://login.5ch.net/login.php
レス数が1000を超えています。これ以上書き込みはできません。

ニューススポーツなんでも実況